You are on page 1of 206

150 Challenges For Mathletes

Along with INMO and TST Problems

Pranav M. Sawant
Piyush K. Jha
Anshuman Shukla
Contents
Preface 3

Acknowledgments 4

Topics of Olympiad Math 5

I 150 Challenges For Mathletes ……………………………… 9

A. Solved Challenges……………………………………………….. 10

B. Unsolved Challenges………………………………………….… 20

II INMO Problems .…………………………..……………………… 44

III Team Selection Test Problems ………………………………. 101

IV Solutions of Solved Problems .………………………………. 181

About the Authors 202

Recommended Resources for Olympiad Mathematics 204


Preface
Olympiad Mathematics is significantly different from what a normal math student would study
in standard math courses such as calculus, pre-calculus, algebra etc. This makes it challenging
but at the same time, it is a lot of fun indeed! Mathematics, as you all know, is the essence of
science, without which nearly every field of higher study would be impenetrable. Mathematics
is, in its way, the poetry of logical ideas.

This book is intended to be a problem-solving book in mathematics. In the current edition, there
are around 150 problems, crafted by me and my team, at the level of the IOQM (Indian
Olympiad Qualifier in Mathematics) / AIME (American Invitational Mathematics Examination)
although certain questions involve concepts regularly used in exams such as the USAMO (United
States of America Mathematical Olympiad) and the INMO (Indian National Mathematical
Olympiad). However we have included certain questions of calculus and real analysis as well.
These problems are typically at the level of PUTNAM

We have not classified the problems according to the difficulty level as we know that difficulty is
a subjective concept and problems that may be hard for some might be very easy for others.
Likewise, we have not added the questions topic-wise as most of these questions contain
concepts from multiple topics and hence require a good level of analysis and problem-solving
abilities across different areas.

In the current edition of the book, we have 50 problems with their solutions (labeled as solved
problems) and 100 unsolved problems, left as an exercise for the readers. In future editions, I
will be looking to add more problems and give hints and solutions for them. If you need any
hints or solutions to the unsolved problems, feel free to shoot me an email anytime at
pranavcoder2005@gmail.com. At the end of this book, there is a special section for Indian
students as well, although it is a good resource for all mathletes. It contains the previous year's
INMO (1986-2022) and Indian Team Selection Test (TST) Problems (2001-2019). I have also given
certain handouts and book suggestions at the end. Constructive criticism is always welcome and
feel free to let me know if you find any errors in this edition. Hope you enjoy this book!

Pranav Sawant
India
Acknowledgments

This book is a small excursion in my mathematics journey and there is no measure of length
when it comes to mentioning who helped me in this journey. A special thanks to Scribe T for
helping me with LaTeX and a huge thank you to the editor of this book Anshuman Shukla for
the wonderful editing and cover page design. I would also like to thank all my mathematics
teachers: Paresh Kokney, Chandrakant Choubey, Prabha Verma, Yury Ustinovskiy, Abhishek
Das, Alok Kumar, Prashant Jain, Nikhil Nagaria, Valsamma Varghese, Mini Santhosh and of
course, my grandmother and grandfather, without whom I probably would have never loved
mathematics. And how can I forget my mom and dad who have supported me throughout the
way.

A big Thank you to all the institutions and persons who provided me invaluable knowledge and
supported me through this journey. Also, a big thank you to all of the AOPS users and members,
without you, we would never have a brilliant collection of problems and their elegant solutions
on AOPS. Pardon me if I forgot someone!

The rights to Indian National Mathematics Olympiad problems and Indian Team Selection Test
problems are exclusively held by Homi Bhabha Centre For Science Education (HBCSE) and
Mathematics Teachers' Association (India) (MTA(I)).
Topics of Olympiad Math

Number Theory
● Euclidean Division Algorithm and Properties of GCD
● Fundamental Theorem of Arithmetic
● Sum of divisors and number of divisors
● Chicken McNugget Theorem
● Bezout’s Theorem
● Congruence Modulo
● Chinese Remainder Theorem and solving basic congruences
● Fermat’s Little Theorem, Euler’s Totient Function and Euler’s Totient Theorem
● Wilson’s Theorem and Lucas’s Theorem
● Order of an Element
● Primitive Roots
● Quadratic congruences
● Quadratic residues, quadratic reciprocity, Legendre’s symbol, Euler’s criterion
● Cyclotomic Polynomials
● Lagrange’s Theorem for Polynomials
● Diophantine equations
● p-adic valuation and Legendre’s formula
● Lifting the Exponent Lemma (LTE)
● Hensel’s Lemma
● Pell’s equation and its properties
● Fermat’s last theorem, Catalan Conjecture and Pythagorean triplets
● Bertrand’s Postulate
● Zsigmondy’s Theorem

Algebra
● Algebraic Identities
● Fundamental Theorem of Algebra
● Polynomial Division and Synthetic Division
● Veita’s Relations
● Factor Theorem and Remainder Theorem
● Rational Root Theorem
● Binomial Theorem and Multinomial Theorem
● Complex Numbers
● Brahmagupta Identity
● Euler’s Four-Square Identity
● Sequences and Series
● Fibonacci Sequence and its properties
● Lagrange Interpolation
● Symmetric Polynomials
● Chebyshev Polynomials
● Rouche’s Theorem
● Intermediate Value Theorem, Lagrange’s Mean Value Theorem, Rolle’s Theorem, Taylor
and Maclaurin series.
● Irreducibility Criterions: Gauss’s Lemma, Eisenstien’s and Extended Eisenstien’s
Irreducibility criterion, Cohn’s irreducibility criterion and Perron’s Irreducibility criterion.
● Infinite Descent and Vieta’s Root Jumping

Functional Equations
● Domain, co-domain, range, injectivity, surjectivity, bijectivity, involution functions,
additive, multiplicative, periodic and cyclic functions
● Cauchy’s Functional Equations
● Jensen’s Functional Equation
● Monotonicity and continuity
● Polynomial Functional Equations

Inequalities
● Triangle Inequalities
● QM-AM-GM-HM
● Weighted Means
● Jensen’s Inequality
● Cauchy Schwarz and Titu’s Inequality
● Rearrangement Inequality
● Muirhead’s inequality
● Holder’s inequality
● Minkowski Inequality
● Isoperimetric inequalities
● Chebyshev’s Inequality
● Schur’s Inequality
● Karamata’s Inequality
● Ravi Substitution
● Lagrange Multipliers

Geometry
● Congruence and Similarity of Triangles
● Angle Chasing, length chasing and trig bashing
● Cyclic Quadrilaterals
● Centroid, circumcentre, incentre, orthocentre, incircles and excircles
● Tangents, Power of a Point and Radical Axis
● Incentre-Excentre Lemma
● Nine-point circle
● Homotheties
● Ceva’s Theorem and Menelaus Theorem
● Simson and Symmedian Lines
● Midpoint of altitudes
● Isogonal/Isotonic conjugates
● Cartesian and Barycentric Coordinates
● Curvilinear and Mixtilinear Incircles
● Special Points; HM Point, Fermat Point, Isodynamic Point, Bevan Point
● Inversion and angle conservation in inversion
● Overlays and orthogonal circles
● Inversion distance formula
● Cross Ratio
● Harmonic Bundles and Quadrilaterals
● Apollonian Circles
● Pascal’s Theorem and Projective Transformations
● Polars/Poles and Bruhcard’s Theorem
● Brianchon’s Theorem
● Spiral Symmetry
● Miquel’s Theorem and Miquel Points
● Gauss-Bodenmiller Theorem
● Moving Points and Circle Tangency through Homothety
● Cayley-Bacharach Theorem
● Napoleon’s Theorem
● Sawayama-Thebault Theorem

Combinatorics
● Set Theory, relations and Cartesian Product, image and preimage, composition,
cardinality, De-Morgan’s Laws, Venn Diagrams and Syllogism
● Basic counting and Fundamental Theorem of Counting
● Circular Permutations, Selection and Division of Objects, Arrangements and
Derangements
● Bijections
● Double Counting
● Recursion
● Principle of Inclusion and Exclusion (PIE)
● Pigeonhole Principle
● Permutation groups, Burnside Counting Lemma and Polya’s Theorem
● Hall’s Marriage Theorem
● Dilworth’s Lemma
● Pascal’s Triangle and Pascal’s Identity
● Hockey-Stick Identity
● Vandermonde’s Identity
● Bayes Theorem
● Invariants and monovariants
● Generating Functions
● Game Theory

Graph Theory
● Graphs, edges, vertices, faces, adjacent, incident, degree, path, cycle, length of path and
cycle, walk, connect/disconnect, tree, forest, Hamiltonian path/cycle, Eulerian circuit,
complete, planar, bipartite, k-partite graphs.
● Handshake Lemma
● Euler’s Formula for Planar Graphs
● Kuratowski’s Theorem
● Dirac’s Theorem
● Ore’s Theorem
● Cayley’s Theorem
● Turan’s Lemma
● Four color theorem
● Ramsey Theory
● Zarankiewicz’s Lemma
150 Challenges For Mathletes
Solved Challenges

Problem 1. Let f ( x ) = x3 + ax2 + bx + c and g( x ) = x3 + bx2 + cx + a, where a, b, c


are integers with c ̸= 0. Suppose that the following conditions hold: f (1) = 0, the
roots of g( x ) = 0 are the squares of the roots of f ( x ) = 0.
Find the value of a2023 + b2023 + c2023 .

Problem 2.
if α, β, γ are roots of the equation x3 + 2x2 + 3x + 1 = 0 Find last four digits of
α35005 + β35005 + γ35005

Problem 3.
In △ ABC, let D be the foot of the altitude from A to BC. Construct squares ABWX
and CAYZ outside △ ABC. Let M be the midpoint of XY and P be the intersection of
BZ and CW. Prove that M, A, P, and D are collinear.

Problem 4.
q p √
Define a function g : N → R Such that g( x ) = 4x + 4x+1 + 4x+2 + .... Find the
sum of last 4 digits in the decimal representation of g(2023).

Problem 5.
Compute the number of ordered quadruples (w, x, y, z) of complex numbers (not
necessarily non-real) such that the following system is satisfied:
wxyz = 1
wxy2 + wx2 z + w2 yz + xyz2 = 2
wx2 y + w2 y2 + w2 xz + xy2 z + x2 z2 + ywz2 = −3
w2 xy + x2 yz + wy2 z + wxz2 = −1
10
Solved Challenges

Problem 6. The positive reals x, y, z satisfy the relations

x2 + xy + y2 = 1
y2 + yz + z2 = 2
z2 + zx + x2 = 3.

m−n p
The value of y2 can be expressed uniquely as q , where m, n, p, q are positive
integers such that p is not divisible by the square of any prime and no prime dividing
q divides both m and n. Compute m + n + p + q

p √
Problem 7. Solve the equation p + q2 + r = s2 + t in prime numbers.

Problem 8. F ind all functions f : R → R, satisfying the condition


f ( x f (y) + f ( x )) = 2 f ( x ) + xy
for any real x and y.

Problem 9. How many f : A → A are there satisfying f ( f ( a)) = a for every


a ∈ A = {1, 2, 3, 4, 5, 6, 7}?

21 times
z }| {
Problem 10. W hat is the least positive integer n such that f ( f (. . . f (n))) = 2013

where f ( x ) = x + 1 + ⌊ x ⌋? (⌊ a⌋ denotes the greatest integer not exceeding the real
number a.)

11
Solved Challenges

Problem 11. How many triples of positive integers ( a, b, c) are there such that
a! + b3 = 18 + c3 ?

Problem 12. Let x, y, z be real numbers such that

x + y + z = 2, xy + yz + zx = 1

Find the maximum possible value of x − y.

Problem 13. Let φ(n) be the number of positive integers less than n that are
relatively prime to n, where n is a positive integer. Find all pairs of positive integers
(m, n) such that
2n + (n − φ(n) − 1)! = nm + 1.

Problem 14. L et z1 , z2 , z3 be nonzero complex numbers and pairwise distinct, hav-


ing the property that (z1 + z2 )3 = (z2 + z3 )3 = (z3 + z1 )3 . Show that |z1 − z2 | =
| z2 − z3 | = | z3 − z1 | .

Problem 15. Show:

Z π/2
8
< sin(sinx )dx < 1
9 0

12
Solved Challenges

Problem 16. Let a ∈ (1, ∞) and a countinuous function f : [0, ∞) −→ R having the
property:
lim x f ( x ) ∈ R.
x →∞
R ∞ f (x) Ra 
a) Show that the integral 1 x dx and the limit limt→∞ t 1 f x t dx both exist, are
finite and equal. R a dx
b) Calculate limt→∞ t 1 1+ xt
.

Problem 17. For any integer n ≥ 2 denote by An the set of solutions of the equation
jxk jxk jxk
x= + +···+ .
2 3 n
Determine the set A2 ∪ A3 .

Problem 18. Find all injective functions f : Z → Z that satisfy: | f ( x ) − f (y)| ≤


| x − y| ,for any x, y ∈ Z.

Problem 19. Determine continuous functions f : R → R such that


 Rb Rb
a2 + ab + b2 f ( x ) dx = 3 x2 f ( x ) dx, for every a, b ∈ R .
a a

R1 n
Problem 20. Calculate: lim 0 e x dx
n→∞

13
Solved Challenges

Problem 21. Let the matrices of order 2 with the real elements A and B so that
AB = A2 B2 − ( AB)2 and det ( B) = 2. Calculate det ( A + 2B) − det ( B + 2A).

Problem 22. Find the minimum number of perfect cubes such that their sum is
equal to 346346 .

Problem 23. Prove that for any integers a, b, the equation 2abx4 − a2 x2 − b2 − 1 = 0
has no integer roots.

Problem 24. ABCD is a cyclic convex quadrilateral whose diagonals meet at X. The
circle ( AXD ) cuts CD again at V and the circle ( BXC ) cuts AB again at U, such that D
lies strictly between C and V and B lies strictly between A and U. Let P ∈ AB ∩ CD.
If M is the intersection point of the tangents to U and V at (UPV ) and T is the
second intersection of circles (UPV ) and ( PAC ), prove that ∠ PTM = 90o .

Problem 25. Let ABC be a triangle. Consider the circle ω B internally tangent to
the sides BC and BA, and to the circumcircle of the triangle ABC, let P be the point
of contact of the two circles. Similarly, consider the circle ωC internally tangent to
the sides CB and CA, and to the circumcircle of the triangle ABC, let Q be the point
of contact of the two circles. Show that the incentre of the triangle ABC lies on the
segment PQ if and only if AB + AC = 3BC.

14
Solved Challenges
4 x −cos2 x 4 x −sin2 x
Problem 26. Solve the following equation 2sin − 2cos = cos 2x

Problem 27. Let a, b, c, d ∈ N ∗ such that the equation

x2 − ( a2 + b2 + c2 + d2 + 1) x + ab + bc + cd + da = 0

has an integer solution. Prove that the other solution is integer too and both solutions
are perfect squares.

Problem 28. Solve the equation

2 +x
2023x + log2023 x = 2023x+1

Problem 29. In a triangle ABC, where a = BC, b = CA and c = AB, it is known


that: a + b − c = 2 and 2ab − c2 = 4. Prove that ABC is an equilateral triangle.

Problem 30. Prove that the number 1010 can’t be written as the product of two
natural numbers which do not contain the digit "0" in their decimal representation.

Problem 31. Show that for every natural n > 1 we have: (n − 1)2 | nn−1 − 1

15
Solved Challenges

Problem 32. Let a, b, c > 0 the sides of a right triangle. Find all real x for which
a x > b x + c x , with a is the longest side.

Problem 33. (a) Show that for every n ∈ N there is exactly one x ∈ R + so that
x n + x n+1 = 1. Call this xn . (b) Find lim xn .
n→+∞

Problem 34. Show that for p > 1 we have

1 p + 2 p + ... + (n − 1) p + n p + (n − 1) p + ... + 2 p + 1 p
lim = +∞
n→+∞ n2
Find the limit if p = 1.

Problem 35. An acute triangle ABC (AB > AC) has circumcenter O, but D is
the midpoint of BC. Circle with diameter AD intersects sides AB and AC in E and F
respectively. On segment EF pick a point M so that DM ∥ AO. Prove that triangles
ABD and FDM are similar.

∞   
n−r+1 r−1
Problem 36. I f 1 ≤ r ≤ n are integers, prove the identity: ∑ d d−1
=
d =1
 
n
.
r

16
Solved Challenges
1 1 1 1
Problem 37. Show that for every positive integer n, 2 2 · 4 4 · 8 8 · ... · (2n ) 2n < 4.

p
3
√ p
3

Problem 38. Find all non-negative integer x for which 13 + x+ 13 − x is
an integer.

Problem 39. Determine all functions f : R → R such that:

f (max { x, y} + min { f ( x ), f (y)}) = x + y


for all real x, y ∈ R

Problem 40. Find all real quadruples ( a, b, c, d) satisfying the system of equations


 ab + cd = 6

ac + bd = 3

 ad + bc = 2

a + b + c + d = 6.

sin 3x +sin 3y+sin 3z


Problem 41. If tan( x + y + z) = cos 3x +cos 3y+cos 3z , compute difference between
maximum and minimum value of R = cos( x + y + z)(cos 3x + cos 3y + cos 3z) +
sin( x + y + z)(sin 3x + sin 3y + sin 3z)

17
Solved Challenges

Problem 42. Given that x2 + y2 = 1 and 4xy 2x2 − 1 = 1. If the largest possible
p √
value of x that satisfies these equations can be expressed as 12 a + b. Find a + b.

Problem 43. For how many rational numbers p is the area of the triangle formed
by the intercepts and vertex of f ( x ) = − x2 + 4px − p + 1 an integer?

Problem 44. Compute


Z A
1 1
lim A x , dx.
A→+∞ A 1

Problem 45. a, b, c, d ∈ R, solve the system of equations:


 3

 a +b = c

 b3 + c = d


 c3 + d = a
 3
d +a=b

Problem 46. f , g : R → R find all f , g satisfying ∀ x, y ∈ R:

g( f ( x ) − y) = f ( g(y)) + x.

18
Solved Challenges

Problem 47. Find all pairs of positive integers x, y satisfying the equation

y x = x50

Problem 48. In the isosceles triangle ABC the angle BAC is a right angle. Point D
lies on the side BC and satisfies BD = 2 · CD. Point E is the foot of the perpendicular
of the point B on the line AD. Find the angle CED.

Problem 49. Let k,m and n be three different positive integers. Prove that
   
1 1 1
k− m− n− ≤ kmn − (k + m + n).
k m n

Problem 50. If R and S are two rectangles with integer sides such that the perimeter
of R equals the area of S and the perimeter of S equals the area of R, then we call R
and S a friendly pair of rectangles. Find all friendly pairs of rectangles.

19
B. Unsolved Challenges

Problem 51. The sum


44
∑ 2𝑠𝑖𝑛𝑥𝑠𝑖𝑛1[1 + 𝑠𝑒𝑐(𝑥 − 1)𝑠𝑒𝑐(𝑥 + 1)]
𝑥=2

4
( )
𝑛 Φ θ𝑛
can be written in the form ∑ (− 1)
𝑛=1
( ) , where Φ, Ψ are trigonometric functions and θ1,
Ψ θ𝑛

θ2, θ3, θ4 are degrees ∈ [0, 45]. Find θ1 + θ2 + θ3 + θ4.

Problem 52 A function 𝑓 from the positive integers to the positive integers is called INMO
if it satisfies
𝑔𝑐𝑑(𝑓(𝑓(𝑥)), 𝑓(𝑥 + 𝑦)) = 𝑔𝑐𝑑(𝑥, 𝑦)

for all pairs of positive integers 𝑥 and 𝑦.


Find all positive integers 𝑚 such that 𝑓(𝑚) = 𝑚 for all INMO functions 𝑓.

Problem 53 In △𝐴𝐵𝐶, let 𝐷 be the foot of the altitude from 𝐴 to 𝐵𝐶. Construct squares
𝐴𝐵𝑊𝑋 and 𝐶𝐴𝑌𝑍 outside △𝐴𝐵𝐶. Let 𝑀 be the midpoint of 𝑋𝑌 and 𝑃 be the intersection of
𝐵𝑍 and 𝐶𝑊. Prove that 𝑀, 𝐴, 𝑃, and 𝐷 are collinear.

Problem 54 If the maximum value of 𝑓(θ) = 𝑠𝑖𝑛θ + 𝑠𝑖𝑛3θ + 𝑠𝑖𝑛5θ Across all 𝑅 is of the
𝑎 𝑏
form 𝑐
where 𝑔𝑐𝑑(𝑏, 𝑐) = 1 and 𝑎, 𝑏, 𝑐 are positive integers, Where point of maxima is of
the form θ = 𝑎𝑟𝑐𝑠𝑖𝑛 ( ) and ℓ, 𝑚 are positive integers. Compute the distance of the point
𝑚

(ℓ, 𝑚) From the line 𝑎𝑥 + 𝑏𝑦 + 𝑐 = 0 (if ans is a irrational report its greatest integer
function)

20
B. Unsolved Challenges

Problem 55 Suppose 𝑋, 𝑌, 𝑍 are collinear points in that order such that 𝑋𝑌 = 1 and
𝑌𝑍 = 3. Let 𝑊 be a point such that 𝑌𝑊 = 5, and define 𝑂1 and 𝑂2 as the circumcenters of
triangles △𝑊𝑋𝑌 and △𝑊𝑌𝑍, respectively. What is the minimum possible length of segment
𝑂1𝑂2?

Problem 56 If

Then find
−2024
( − 𝑥 + 𝑦)

Problem 57 The line 𝑦 = 𝑘𝑥 (where 𝑘 is a positive real number) makes an acute angle of

70 with the 𝑥-axis. Point 𝑂 is at the origin and point 𝐴 is at (0, 4). Point 𝑃 is a point on
𝑎 𝑏
segment 𝑂𝐴 and 𝑀 and 𝑁 are points on the line 𝑦 = 𝑘𝑥. Let 𝑐
be the minimum possible
value of 𝐴𝑀 + 𝑀𝑃 + 𝑃𝑁 where 𝑎 and 𝑐 are relatively prime and 𝑏 is squarefree. Find
𝑎 + 𝑏 + 𝑐.

Problem 58 Let and for all 𝑥, 𝑦 ϵ [𝑎, 𝑏]. Define a

sequence . Show that 𝑥𝑛 converges to a fixed point of 𝑓.

21
B. Unsolved Challenges

Problem 59 Define 𝑓: 𝑅 → 𝑅 be a function such that


𝑁
𝑓(𝑥) = ∑ (𝑖 − 𝑥)(2𝑖 − 𝑥)
𝑖=1

if 𝑆(𝑁) be the set of minimum values of 𝑓(𝑥) for different 𝑁 find number of 𝑁 such that
+
subset of 𝑆(𝑁) are also the subset of 𝑍

Problem 60 Consider points 𝐷, 𝐸 and 𝐹 on sides 𝐵𝐶, 𝐴𝐶 and 𝐴𝐵, respectively, of a triangle
𝐴𝐵𝐶, such that 𝐴𝐷, 𝐵𝐸 and 𝐶𝐹 concurr at a point 𝐺. The parallel through 𝐺 to 𝐵𝐶 cuts 𝐷𝐹
and 𝐷𝐸 at 𝐻 and 𝐼, respectively. Show that triangles 𝐴𝐻𝐺 and 𝐴𝐼𝐺 have the same areas.

Problem 61 Let 𝐴𝐵𝐶 be a triangle with 𝐼 as incenter. The incircle touches 𝐵𝐶 at 𝐷. Let 𝐷′ be
the antipode of 𝐷 on the incircle. Make a tangent at 𝐷′ to incircle. Let it meet (𝐴𝐵𝐶) at 𝑋, 𝑌
respectively. Let the other tangent from 𝑋 meet the other tangent from 𝑌 at 𝑍. Prove that
(𝑍𝐵𝐷) meets 𝐼𝐵 at the midpoint of 𝐼𝐵

Problem 62 For a positive integer 𝑛, denote by 𝑔(𝑛) the number of strictly ascending
triples chosen from the set {1, 2, ..., 𝑛}. Find the least positive integer 𝑛 such that the
following holds: The number 𝑔(𝑛) can be written as the product of three different prime
numbers which are (not necessarily consecutive) members in an arithmetic progression
with common difference 336.

Problem 63 Let 𝑎𝑛{ }𝑛≥1 and {𝑏𝑛}𝑛≥1 be two infinite arithmetic progressions, each of which
the first term and the difference are mutually prime natural numbers. It is known that for

22
B. Unsolved Challenges

any natural 𝑛, at least one of the numbers 𝑎𝑛 + 𝑎𝑛+1 𝑏𝑛 + 𝑏𝑛+1 or( 2 2


)( 2 2
)
(𝑎 2
𝑛
2
)( 2 2
)
+ 𝑏𝑛 𝑎𝑛+1 + 𝑏𝑛+1 is a perfect square. Prove that 𝑎𝑛 = 𝑏𝑛, for any natural 𝑛.

Problem 64 Let

. Find

Problem 65 A tournament is played between 𝑛 people. Everyone plays with everyone else,
and no game ends in a draw. A number 𝑘 is said to be 𝑛-good if there exists such a
tournament in which there is, for every 𝑘 people, a player who has lost all of them. a) Prove
𝑘+1
that 𝑛 ≥ 2 − 1 b) Give all 𝑛 for which 2 is 𝑛-good.

Problem 66 For a positive integer 𝑛, two players 𝐴 and 𝐵 play the following game: Given a
pile of 𝑠 stones, the players take turn alternatively with 𝐴 going first. On each turn the
player is allowed to take either one stone, or a prime number of stones, or a positive
multiple of 𝑛 stones. The winner is the one who takes the last stone. Assuming both 𝐴 and 𝐵
play perfectly, for how many values of 𝑠 the player 𝐴 cannot win?

Problem 67 Let 𝑀(𝑛) = {𝑛, 𝑛 + 1, 𝑛 + 2, 𝑛 + 3, 𝑛 + 4, 𝑛 + 5} be a set of 6 consecutive


integers. Let’s take all values of the form
𝑎 𝑐 𝑒
𝑏
+ 𝑑
+ 𝑓

with the set {𝑎, 𝑏, 𝑐, 𝑑, 𝑒, 𝑓 = 𝑀(𝑛)}. Let


𝑥 𝑦 𝑧 𝑥𝑣𝑤+𝑦𝑢𝑤+𝑧𝑢𝑣
𝑢
+ 𝑣
+ 𝑤
= 𝑢𝑣𝑤

be the greatest of all these values.

23
B. Unsolved Challenges

a) show: for all odd 𝑛 hold: 𝑔𝑐𝑑(𝑥𝑣𝑤 + 𝑦𝑢𝑤 + 𝑧𝑢𝑣, 𝑢𝑣𝑤) = 1 iff
𝑔𝑐𝑑(𝑥, 𝑢) = 𝑔𝑐𝑑(𝑦, 𝑣) = 𝑔𝑐𝑑(𝑧, 𝑤) = 1. b) for which positive integers 𝑛 hold
𝑔𝑐𝑑(𝑥𝑣𝑤 + 𝑦𝑢𝑤 + 𝑧𝑢𝑣, 𝑢𝑣𝑤) = 1?

𝑛
Problem 68 A polynomial 𝑝(𝑥) of degree 1000 is such that 𝑝(𝑛) = (𝑛 + 1)2 for all
nonnegative integers 𝑛 such that 𝑛 ≤ 1000. Given that
𝑏
𝑝(1001) = 𝑎 · 2 − 𝑐,

where 𝑎 is an odd integer, and 0 < 𝑐 < 1001, find 𝑐 − (𝑎 + 𝑏))

Problem 69 Call a convex quadrilateral angle-Pythagorean if the degree measures of its


angles are integers 𝑤 ≤ 𝑥 ≤ 𝑦 ≤ 𝑧 satisfying
2 2 2 2
𝑤 +𝑥 +𝑦 =𝑧 .

Determine the maximum possible value of 𝑥 + 𝑦 for an angle-Pythagorean quadrilateral.

Problem 70 One can define the greatest common divisor of two positive rational numbers
as follows: for 𝑎, 𝑏, 𝑐, and 𝑑 positive integers with 𝑔𝑐𝑑(𝑎, 𝑏) = 𝑔𝑐𝑑(𝑐, 𝑑) = 1, write

𝑔𝑐𝑑 ( 𝑎
𝑏
,
𝑐
𝑑 )= 𝑔𝑐𝑑(𝑎𝑑,𝑏𝑐)
𝑏𝑑
.

For all positive integers 𝐾, let 𝑓(𝐾) denote the number of ordered pairs of positive rational
numbers (𝑚, 𝑛) with 𝑚 < 1 and 𝑛 < 1 such that
1
𝑔𝑐𝑑(𝑚, 𝑛) = 𝐾
.

What is ?

24
B. Unsolved Challenges

Problem 71 Euclid places a morsel of food at the point (0, 0) and an ant at the point (1, 2).
Every second, the ant walks one unit in one of the four coordinate directions. However,
whenever the ant moves to (𝑥, ± 3), Euclid’s notorious friend uncle chipotle picks it up and
puts it at (− 𝑥, ∓ 2), and whenever it moves to (± 2, 𝑦), his cousin uncle john puts it at
𝑝
(∓ 1, 𝑦), If 𝑝 and 𝑞 are relatively prime positive integers such that 𝑞 is the expected
number of steps the ant takes before reaching the food, find 𝑝 + 𝑞.

Problem 72 Determine all the triples {𝑎, 𝑏, 𝑐} of positive integers coprime (not necessarily
pairwise prime) such that 𝑎 + 𝑏 + 𝑐 simultaneously divides the three numbers
12 12 12 23 23 23 11004 11004 11004
𝑎 +𝑏 + 𝑐 ,𝑎 +𝑏 +𝑐 and 𝑎 +𝑏 +𝑐

Problem 73 In a group of 2021 people, 1400 of them are squid game runners. James Bond
wants to find one squid game runner. There are some missions that each need exactly 3
people to be done. A mission fails if at least one of the three participants in that mission is a
squid game runner. In each round James chooses 3 people, sends them to a mission and
sees whether it fails or not. What is the minimum number of rounds he needs to accomplish
his goal?

Problem 74 𝑛 > 1 is an odd number and 𝑎1, 𝑎2, ···, 𝑎𝑛 are positive integers such that
(
𝑔𝑐𝑑 𝑎1, 𝑎2, ···, 𝑎𝑛 = 1. If )
( 𝑛 𝑛 𝑛
)
𝑑 = 𝑔𝑐𝑑 𝑎1 + 𝑎1 · 𝑎2 ··· 𝑎𝑛, 𝑎2 + 𝑎1 · 𝑎2 ··· 𝑎𝑛, ···, 𝑎𝑛 + 𝑎1 · 𝑎2 ··· 𝑎𝑛 find all possible
values of 𝑑.

25
B. Unsolved Challenges

Problem 75 A complete number is a 9 digit number that contains each of the digits 1 to 9
exactly once. The difference number of a number 𝑁 is the number you get by taking the
differences of consecutive digits in 𝑁 and then stringing these digits together. For instance,
the difference number of 25143 is equal to 3431. The complete number 124356879 has the
additional property that its difference number, 12121212, consists of digits alternating
between 1 and 2. Determine all 𝑎 with 3 ≤ 𝑎 ≤ 9 for which there exists 𝑎 complete
number 𝑁 with the additional property that the digits of its difference number alternate
between 1 and 𝑎.


Problem 76 𝐴𝐵𝐶 is a right angled triangle with ∠𝐴 = 90 and 𝐷 be the midpoint of 𝐵𝐶. A
point 𝐹 is chosen on 𝐴𝐵. 𝐶𝐴 and 𝐷𝐹 meet at 𝐺 and 𝐺𝐵 ‖ 𝐴𝐷. 𝐶𝐹 and 𝐴𝐷 meet at 𝑂 and
2
𝐴𝐹 = 𝐹𝑂. 𝐺𝑂 meets BC at 𝑅. Find The sides of 𝐴𝐵𝐶 if the area of 𝐺𝐷𝑅 is
15

Problem 77 Shishimaru and hattori play a game. 2021 stones lie on a table. Starting with
shishimaru, they alternatively remove stones from the table, while obeying the following
rule. At the 𝑛-th turn, the active player (shishimaru if 𝑛 is odd, hattori if 𝑛 is even) can
remove from 1 to 𝑛 stones. Thus, shishimaru first removes 1 stone; then, hattori can remove
1 or 2 stones, as she wishes; then, shishimaru can remove from 1 to 3 stones, and so on. The
player who removes the last stone on the table loses, and the other one wins. Which player
has a strategy to win regardless of the other player’s moves?

Problem 78 You have a 3 × 2021 chessboard from which one corner square has been
removed. You also have a set of 3031 identical dominoes, each of which can cover two
adjacent chessboard squares. Let 𝑚 be the number of ways in which the chessboard can be
covered with the dominoes, without gaps or overlaps. What is the remainder when 𝑚 is
divided by 19?

Problem 79 Let 𝑀 be the midpoint of segment 𝐵𝐶 of △𝐴𝐵𝐶. Let 𝐷 be a point such that
𝐴𝐷 = 𝐴𝐵, 𝐴𝐷 ⊥ 𝐴𝐵 and points 𝐶 and 𝐷 are on different sides of 𝐴𝐵. Prove that:
2
𝐴𝐵 · 𝐴𝐶 + 𝐵𝐶 · 𝐴𝑀 ≥ 2
𝐶𝐷.

26
B. Unsolved Challenges

Problem 80 Let 𝑇1 = 8, 𝑇2 = 8, 𝑇𝑛 = (𝑇𝑛−1 + 𝑇𝑛−2)𝑚𝑜𝑑 10. Then find the value of 𝑇42.

Problem 81 Doraemon is a robot who can move freely on the unit circle and its interior,
but is attached to the origin by a retractable cord such that at any moment the cord lies in a
straight line on the ground connecting doraemon to the origin. Whenever his movement is
counterclockwise (relative to the origin), the cord leaves a coating of black paint on the
ground, and whenever his movement is clockwise, the cord leaves a coating of black paint
on the ground, and whenever his movement is clockwise, the cord leaves a coating of
orange paint on the ground. The paint is dispensed regardless of whether there is already
2
paint on the ground. The paints covers 1 liter/unit , and doraemon starts at (1, 0).Each
second, he moves in a straight line from the point (𝑐𝑜𝑠(θ), 𝑠𝑖𝑛(θ)) to the point

(𝑐𝑜𝑠(θ + 𝑎), 𝑠𝑖𝑛(θ + 𝑎)), where 𝑎 changes after each movement. 𝑎 starts out as 253 and

decreases by 2 each step. If he takes 89 steps, then the difference, in liters, between the
𝑎− 𝑏 ◦
amount of black paint used and orange paint used can be written as 𝑐
𝑐𝑜𝑡1 , where 𝑎, 𝑏
and 𝑐 are positive integers and no prime divisor of 𝑐 divides both 𝑎 and 𝑏 twice. Find
𝑎 + 𝑏 + 𝑐.
Problem 82 Cynthia loves Pokemon and she wants to catch them all. In victory Road, there
are a total of 80 Pokemon. Cynthia wants to catch as many of them as possible. However,
she cannot catch any two Pokemon that are enemies with each other. After exploring
around for a while, she makes the following two observations: 1. Every Pokemon in Victory
Road is enemies with exactly two other Pokemon. 2. Due to her inability to catch Pokemon
that are enemies with one other, the maximum number of the Pokemon she can catch is
equal to 𝑛. What is the sum of all possible values of 𝑛?

Problem 83 In △𝐴𝐵𝐶, 𝐴𝐵 = 4, 𝐵𝐶 = 5, and 𝐶𝐴 = 6. Circulars arcs 𝑝, 𝑞 and 𝑟 of measure



60 are drawn from 𝐴 to 𝐵, from 𝐴 to 𝐶, and from 𝐵 to 𝐶, respectively, so that 𝑝, 𝑞 lie
completely outside △𝐴𝐵𝐶 but 𝑟 does not. Let 𝑋, 𝑌, 𝑍 be the midpoint of 𝑝, 𝑞, 𝑟, respectively.
𝑎 𝑏+𝑐
If 𝑠𝑖𝑛∠𝑋𝑍𝑌 = 𝑑
, where 𝑎, 𝑏, 𝑐, 𝑑 are positive integers, 𝑔𝑐𝑑(𝑎, 𝑐, 𝑑) = 1, and 𝑏 is not
divisible by the square of a prime, compute 𝑎 + 𝑏 + 𝑐 + 𝑑.

27
B. Unsolved Challenges

Problem 84 On a table near the sea, there are 𝑁 glass boxes where 𝑁 < 2021, each
containing exactly 2021 balls. Sowdha and Rafi play a game by taking turns on the boxes
where Sowdha takes the first turn. In each turn, a player selects a non-empty box and
throws out some of the balls from it into the sea. If a player wants, he can throw out all of
the balls in the selected box. The player who throws out the last ball wins. Let 𝑆 be the sum
of all values of 𝑁 for which Sowdha has a winning strategy and let 𝑅 be the sum of all values
𝑅−𝑆
of 𝑁 for which Rafi has a winning strategy. What is the value of 10

Problem 85 A series of lockers, numbered 1 through 100, are all initially closed. Student 1
goes through and "flips" every 3rd locker ("flipping") a locker means changing its state: if
the locker is open he closes it, and if the locker is closed he opens it. Thus, Student 3 will
close the third locker, open the sixth, close the ninth Student 5 then goes through and "flips"
every 5th locker. This process continues with all students with odd numbers 𝑛 < 100 going
through and "flipping" every 𝑛-th locker.
How many lockers are open after this process?

Problem 86 Say there is a polynomial with integral coefficients such that there exists four
( ) ( ) ( ) ( )
distinct integers such that 𝑓 𝐼1 = 𝑓 𝐼2 = 𝑓 𝐼3 = 𝑓 𝐼4 = 2021, find sum of all such
integers such 𝐼′ such that 𝑓(𝐼′) = 2023. [note 𝐼𝑖 is integer for 𝑖 = 1, 2, 3, ··· 𝑛]

Problem 87 There is a table with 𝑛 rows and 18 columns. Each of its cells contains a 0 or a
1. The table satisfies the following properties: 1) Every two rows are different. 2) Each row
contains exactly 6 cells that contain 1. 3) For every three rows, there exists a column so that
the intersection of the column with the three rows (the three cells) all contains 0. What is
the greatest possible value of 𝑛?

28
B. Unsolved Challenges

Problem 88 On a party with 99 guests, hosts shin chan and masao play a game (the hosts
are not regarded as guests). There are 99 chairs arranged in a circle; initially, all guests
hang around those chairs. The hosts take turns alternately. By a turn, a host orders any
standing guest to sit on an unoccupied chair 𝑐. If some chair adjacent to 𝑐 is already
occupied , the same host orders one guest on such chair to stand up (if both chairs adjacent
to 𝑐 are occupied, the host chooses exactly one of them). All orders are carried out
immediately. shinchan makes the first move; her goal is to fulfill, after some move of hers,
that at least 𝑘 chairs are occupied. Determine the largest 𝑘 for which shinchan can reach the
goal, regardless of masao play.

Problem 89 Find all positive integer 𝑚 such that there exist an infinite AP (𝑎𝑛) and an
infinite GP of positive integer such that 𝑚|𝑎𝑛 − 𝑔𝑛 for all 𝑛 ≥ 1 and 𝑚|𝑎2 − 𝑎1.

Problem 90 Vertices of a triangle are taken from the set A,B,C in the same order and its
sides are extended to vertices P,Q,R in same order. if BP is thrice of AB, RC is twice of AC and
𝑎
BQ is just half of BC. if ratio of △𝐴𝐵𝐶 to △𝑃𝑄𝑅 is in form 𝑏 where a and b are positive
2 2
integers and 𝑔𝑐𝑑(𝑎, 𝑏) = 1, find 5𝑎 + 𝑏

Problem 91 201 positive integers are written on a line, such that both the first one and the
last one are equal to 19999. Each one of the remaining numbers is less than the average of
its neighboring numbers, and the differences between each one of the remaining numbers
and the average of its neighboring numbers are all equal to a unique integer. Find the
second-to-last term on the line

Problem 92 Points 𝑋 and 𝑌 are the midpoints of arcs 𝐴𝐵 and 𝐵𝐶 of the circumscribed
circle of triangle 𝐴𝐵𝐶. Point 𝑇 lies on side 𝐴𝐶. It turned out that the bisectors of the angles
𝐴𝑇𝐵 and 𝐵𝑇𝐶 pass through points 𝑋 and 𝑌 respectively. What angle 𝐵 can be in triangle 𝐴𝐵𝐶
?

29
B. Unsolved Challenges

Problem 93 In △𝐴𝐵𝐶 the median 𝐴𝑀 is drawn. the foot of perpendicular from 𝐵 to the
angle bisector of ∠𝐵𝑀𝐴 is 𝐵1 and the foot of perpendicular from 𝐶 to the angle bisector of
𝐵1𝐴1
∠𝐴𝑀𝐶 is 𝐶1. Let 𝑀𝐴 and 𝐵1𝐶1 intersect at 𝐴1. Find 𝐴1𝐶1
.

Problem 94 Kalia has 3 red color ice cream and 3 black color ice cream. Find the number
of distinct ways that kalia can place these checkers in stacks. Two ways of stacking ice
creams are the same if each stack of the rest way matches a corresponding stack in the
second way in both size and color arrangement. So, for example, the 3 stack arrangement
𝑅𝐵𝑅, 𝐵𝑅, 𝐵 is distinct from 𝑅𝐵𝑅, 𝑅𝐵, 𝐵, but the 4 stack arrangement 𝑅𝐵, 𝐵𝑅, 𝐵, 𝑅 is the
same as 𝐵, 𝐵𝑅, 𝑅, 𝑅𝐵.

Problem 95 A water bottle (cylindrical in shape) stands upon a horizontal table. from a
point on this plane, a man stares the cap of the water bottle, from which four of its corner
◦ ◦ ◦
points are visible, their angular elevations from the eye of the observer are 30 , α , 30 and

60 . assuming the cap of the bottle to be perfectly circular and ratio of circumference of
bottle cap to circular base of cylinder to be 1:1. find distance of point from the observer

which subtends the α , from the observer’s eye, also find alpha and sum of distance of the
◦ ◦
distance between the eye and points that subtended 30 , 60 given that the point which
subtend angle α is in the extended line of sight of first point from the observer, horizontal
distance between contact point and point at which the angle α is subtended is 6 units, and
distance between first point and the point at which the first point subtends angle is 4 units

Problem 96 Find the number of pairs (𝑛, 𝑞), where 𝑛 is a positive integer and 𝑞 a
non-integer rational number with 0 < 𝑞 < 2000, that satisfy 𝑞 = { 2} { 2000
𝑛!
}

30
B. Unsolved Challenges

Problem 97 Suppose 𝐴𝐵𝐶𝐷 is a trapezoid with 𝐴𝐵 ‖ 𝐶𝐷 and 𝐴𝐵 ⊥ 𝐵𝐶. Let 𝑋 be a point on


segment 𝐴𝐷 such that 𝐴𝐷 bisects ∠𝐵𝑋𝐶 externally, and denote 𝑌 as the intersection of 𝐴𝐶
and 𝐵𝐷. If 𝐴𝐵 = 10 and 𝐶𝐷 = 15, compute the maximum possible value of 𝑋𝑌.

Problem 98 21 bandits live in the city of Wasseypur, each of them having some enemies
among the others. Initially each bandit has 240 bullets, and duels with all of his enemies.
Every bandit distributes his bullets evenly between his enemies, this means that he takes
the same number of bullets to each of his duels, and uses each of his bullets in only one
duel. In case the number of his bullets is not divisible by the number of his enemies, he
takes as many bullets to each duel as possible, but takes the same number of bullets to
every duel, so it is possible that in the end the bandit will have some remaining bullets.
Shooting is banned in the city, therefore a duel consists only of comparing the number of
bullets in the guns of the opponents, and the winner is whoever has more bullets. After the
duel the police take the bullets of the winner and as an act of protest the loser shoots all of
his bullets into the air. What is the largest possible number of bullets the police can have
after all of the duels have ended?
Being someone's enemy is mutual. If two opponents have the same number of bullets in
their guns during a duel, then the police take the bullets of the bandit who has the wider
hat among them.
Example: If a bandit has 13 enemies then he takes 18 bullets with himself to each duel, and
they will have 6 leftover bullets after finishing all their duels.

Problem 99 Define
12 11 10 9 8 7 6 5 4 3 2
𝑃(𝑥) = 𝑥 + 12𝑥 + 66𝑥 + 220𝑥 + 495𝑥 + 792𝑥 + 924𝑥 + 792𝑥 − 159505𝑥 + 220𝑥 + 66𝑥 + 12𝑥 + 1
𝑃(19)
Find sum of digits of integers 4 .
20

Problem 100 Triangle 𝐴𝐵𝐶 is inscribed in circle ω with 𝐴𝐵 = 5, 𝐵𝐶 = 7, and 𝐴𝐶 = 3. The


bisector of angle 𝐴 meets sid 𝐵𝐶 at 𝐷 and circle ω at a second point 𝐸. Let γ be the circle

31
B. Unsolved Challenges

2 𝑚
with diameter 𝐷𝐸. Circles ω and γ meet at 𝐸 and a second point 𝐹. Then 𝐴𝐹 = 𝑛
, where 𝑚
and 𝑛 are relatively prime positive integers. Find 𝑚 + 𝑛.

Problem 101 Let 𝑚 ≥ 𝑛 be positive integers. MOTU is given 𝑚𝑛 posters of patlu with
different integer dimensions of 𝑘 × 𝑙 with 1 ≥ 𝑘 ≥ 𝑚 and 1 ≥ 𝑙 ≥ 𝑛. He must put them
all up one by one on his bedroom wall without rotating them. Every time he puts up a
poster, he can either put it on an empty spot on the wall or on a spot where it entirely
covers a single visible poster and does not overlap any other visible poster. Determine the
minimal area of the wall that will be covered by posters.

Problem 102 Bheem and raju are playing a game. raju has 𝑘 + ( ) cards with their front
𝑘
2
sides face down on the table. The cards are constructed as follows: For each 1 ≤ 𝑛 ≤ 𝑘,
𝑎𝑛
there is a blue card with 𝑛 written on the back, and a fraction 𝑏𝑛
written on the front,

( )
where 𝑔𝑐𝑑 𝑎𝑛, 𝑏𝑛 = 1 and 𝑎𝑛, 𝑏𝑛 > 0. For each 1 ≤ 𝑖 < 𝑗 ≤ 𝑘, there is a red card with
𝑎𝑖+𝑎𝑗
(𝑖, 𝑗) written on the back, and a fraction 𝑏𝑖+𝑏𝑗
written on the front. It is given that no two
cards have equal fractions. In a turn bheem can pick any two cards and raju tells bheem
which card has the larger fraction on the front. Show that, in fewer than 10000 turns,
bheem can determine which red card has the largest fraction out of all of the red cards.

Problem 103 There are three flies of negligible size that start at the same position on a
circular track with circumference 1000 meters. They fly clockwise at speeds of 2, 6, and 𝑘
meters per second, respectively, where 𝑘 is some positive integer with 7 ≤ 𝑘 ≤ 1000.
Suppose that at some point in time, all three flies meet at a location different from their
starting point. How many possible values of 𝑘 are there?

32
B. Unsolved Challenges

Problem 104 The number of sequences 𝑎𝑛 of 2𝑛 terms can be formed using exactly 𝑛(1’s)
𝑘
(
ans exactly 𝑛(-1’s) whose partial sums are always non negative: ∑ 𝑎𝑖 ≥ 0 , 1 ≤ 𝑘 ≤ 2𝑛
𝑖=1
)

Problem 105 Is there a number 𝑛 such that one can write 𝑛 as the sum of 2022 perfect
squares and (with at least) 2022 distinct ways?

Problem 106 Let 𝑀 be a set of six distinct positive integers whose sum is 60. These
numbers are written on the faces of a cube, one number to each face. A move consists of
choosing three faces of the cube that share a common vertex and adding 1 to the numbers
on those faces. Determine the number of sets 𝑀 for which it’s possible, after a finite number
of moves, to produce a cube all of whose sides have the same number.

Problem 107 While running from an unrealistically rendered zombie, uncle chipotle runs
into a vacant lot in the shape of a square, 100 meters on a side. Call the four corners of the
lot corners 1, 2, 3, and 4, in clockwise order. For 𝑘 = 1, 2, 3, 4, let 𝑑𝑘 be the distance
between chipotle and corner 𝑘. Let
(a) 𝑑1 < 𝑑2 < 𝑑4 < 𝑑3,
(b) 𝑑2 is the arithmetic mean of 𝑑1 and 𝑑3, and
(c) 𝑑4 is the geometric mean of 𝑑2 and 𝑑3.
2 𝑎−𝑏 𝑐
If 𝑑1 can be written in the form 𝑑
, where 𝑎, 𝑏, 𝑐, and 𝑑 are positive integers, 𝑐 is
square-free, and the greatest common divisor of 𝑎, 𝑏, and 𝑑 is 1, find the sum of all possible
remainder when 𝑎 + 𝑏 + 𝑐 + 𝑑 is divided by 1000.

33
B. Unsolved Challenges

Problem 108 To any triangle with side lengths 𝑎, 𝑏, 𝑐 and the corresponding angles α, β, γ
(measured in radians), the 6-tuple (𝑎, 𝑏, 𝑐, α, β, γ) is assigned. Find the minimum possible
number 𝑛 of distinct terms in the 6-tuple assigned to a scalene triangle.

Problem 109 Rama and bholi play a game writing numbers on a board as follows: The two
players play in turns and if in the board is written the positive integer 𝑛, the player whose
turn is chooses a prime divisor 𝑝 of 𝑛 and writes the numbers 𝑛 + 𝑝. In the board, it is
written at the start number 2 and Rama plays first. The game is won by whoever shall be
first able to write a number bigger or equal to 31. Find who player has a winning strategy,
that is who may write the appropriate numbers may win the game no matter how the other
player plays.

Problem 110 Find all positive integers 𝑛 such that the number
6 3
𝑛 + 5𝑛 + 4𝑛 + 116

is the product of two or more consecutive numbers.

Problem 111 Laxman has a standard four-sided die. Each roll, he gains points equal to the
value of the roll multiplied by the number of times he has now rolled that number; for
example, if his first rolls were 3, 3, 2, 3, he would have 3 + 6 + 2 + 9 = 20 points. Find
the expected number of points laxman will have after the die 25 times.

Problem 112 Raiyan stands on the bottom-left square of a 2022 by 2022 grid of squares,
where each square is colored either black, gray, or white according to the pattern as
depicted to the right. Each second he moves either one square up, one square to the right,
or both one up and to the right, selecting between these three options uniformly and
independently. Noting that he begins on a black square, find the probability that Raiyan is
still on a black square after 2022 seconds.

34
B. Unsolved Challenges

(𝑛(𝑛+1)(2𝑛+1))!
Problem 113 Find number of integer from 0 to 2022 such that is an
((𝑛−1)𝑛(2𝑛+5))!
integer.

Problem 114 Simplify


𝑛
(2𝑛)!
∑ 2 2 .
𝑘=0 (𝑘!) ((𝑛−𝑘)!)

2 2
Problem 115 Find all pairs of positive integers (𝑚, 𝑛) such that 𝑚 − 𝑚𝑛 + 𝑛 + 1
3 3
𝑚+𝑛 𝑚 +𝑛
divides both numbers 3 + (𝑚 + 𝑛)! and 3 + 𝑚 + 𝑛.

Problem 116 𝑂 is the circumcenter of △𝐴𝐵𝐶 and 𝐶𝐷 is the median to 𝐴𝐵. 𝐺 is the centroid
of △𝐴𝐶𝐷. Prove that 𝑂𝐺 is perpendicular to 𝐶𝐷 only and only if △𝐴𝐵𝐶 is isosceles with
𝐴𝐵 = 𝐴𝐶.

Problem 117 let be a natural number 𝑛, and 𝑛 real numbers 𝑎1, 𝑎2, ···, 𝑎𝑛. Prove that there
exists a real number 𝑎 such that 𝑎 + 𝑎1, 𝑎 + 𝑎2, ···, 𝑎 + 𝑎𝑛 are all irrational.

Problem 118 In a ∆𝑃𝑄𝑅 𝑋ϵ𝑃𝑄 𝑎𝑛𝑑 𝑌ϵ𝑃𝑅, if ∠QPR = 30° such that PQ = 7 and PR = 8,
computer minimum value of the distance QY + XR + XY.

35
B. Unsolved Challenges

Problem 119 given a 4-digit number (𝑎𝑏𝑐𝑑)10 such that both (𝑎𝑏𝑐𝑑)10 and (𝑑𝑐𝑏𝑎)10 are
multiples of 7, having the same remainder modulo 37. Find 𝑎, 𝑏, 𝑐, 𝑑

3
Problem 120 Let 𝑁 be the number of ordered triples (𝑎, 𝑏, 𝑐) ∈ {1, ···, 2022} such that
2 2 2
𝑎 + 𝑏 + 𝑐 = 0 ( 𝑚𝑜𝑑 2023). What are the last three digits of 𝑁?

𝑘
Problem 121 Find 𝑘 where 2 is the largest power of 2 that divides tha product
2022 · 2023 · 2024 ··· 4048.

4
Problem 122 Vijay picks two random distinct primes 1 ≤ 𝑝, 𝑞 ≤ 10 . Let 𝑟 be the
2205403200
probability that 3 ≡ 1 𝑚𝑜𝑑 𝑝𝑞. Estimate 𝑟 in the form 0. 𝑎𝑏𝑐𝑑𝑒𝑓, where 𝑎, 𝑏, 𝑐, 𝑑, 𝑒
, 𝑓 are decimal digits.

𝑝−1
2

{
Problem 123 Determine the number of primes 𝑝 < 100 such that ∑ 𝑘 𝑝 is an integer,
2
}
𝑘=1
where {𝑥} = 𝑥 − [𝑥].

36
B. Unsolved Challenges

Problem 124 Let 𝑓(𝑥) = Φ(𝑥)Ψ(𝑥), where Φ(𝑥) and Ψ(𝑥) are monic polynomials of
positive degree with integer coefficients. Then prove at least one of the polynomials Φ(𝑥)
and Ψ(𝑥) is recursive.

Problem 125 Find all positive integer solutions (𝑎, 𝑏, 𝑐) to the function
2 2 2
𝑎 + 𝑏 + 𝑐 = 2005, where 𝑎 ≤ 𝑏 ≤ 𝑐.

Problem 126 Let 𝑆 be a set of 𝑛 distinct real numbers. Let 𝐴𝑆 be the set of numbers that
occur as averages of two distinct elements of 𝑆. For a given 𝑛 ≥ 2, what is the smallest
possible number of elements in 𝐴𝑆?

Problem 127 The incircle of a triangle ABC touches the sides BC and AC at point D and E,
respectively. Suppose P is the point on the shorter arc DE of the incircle such that Angle
APE = Angle DPB. The segments AP and BP meet the segment DE at points K and L,
respectively. If KL = 4 find DE.

Problem 128 A sequence of positive integers 𝑎1, 𝑎2, 𝑎3, 𝑎4 ··· 𝑎𝑛 (necessarily not in same
order or order of ascending or descending), such that 𝑛 ≤ 99, if
(
4 𝑎𝑛+1)2 − 4(𝑎𝑛)(𝑎𝑛+1) + (𝑎𝑛)2 = 0, find number of possible values of 𝑎1.

Problem 129 There are three distinct positive integers, 𝑎, 𝑏, 𝑐 where 1 ≤ 𝑎, 𝑏, 𝑐 ≤ 100.
𝑐 𝑐
How many ways are there for 𝑎 + 𝑏 to be divisible by 130.

37
B. Unsolved Challenges

Problem 130 Let 𝑋 = {1, 2, 3,..., 𝑛} where 𝑛ϵ𝑁 define .


Compute number of injective functions possible from 𝑋 → 𝑆.

Problem 131 In triangle △𝐴𝐵𝐶, the points 𝐴′, 𝐵′, 𝐶′ are on sides 𝐵𝐶, 𝐴𝐶, 𝐴𝐵 respectively.
Also, 𝐴𝐴′, 𝐵𝐵′, 𝐶𝐶′ intersect at the point 𝑂 (they are concurrent at 𝑂). Also,
𝐴𝑂 𝐵𝑂 𝐶𝑂 𝐴𝑂 𝐵𝑂 𝐶𝑂
𝑂𝐴′
+ 𝑂𝐵′ + 𝑂𝐶′ = 92. Find the value of 𝑂𝐴′ × 𝑂𝐵′ × 𝑂𝐶′ .

Problem 132 Let 𝐼 be the incenter of a triangle 𝐴𝐵𝐶. 𝐷, 𝐸, 𝐹 are the symmetric points of 𝐼
with respect to 𝐵𝐶, 𝐴𝐶, 𝐴𝐵 respectively. Knowing that 𝐷, 𝐸, 𝐹, 𝐵 are concyclic, find all
possible values of ∠𝐵.

Problem 133 Given is a triangle 𝐴𝐵𝐶 and points 𝐷 and 𝐸, respectively on 𝐵𝐶 and 𝐴𝐵. 𝐹 it is
intersection of lines 𝐴𝐷 and 𝐶𝐸. We denote as |𝐶𝐷| = 𝑎, |𝐵𝐷| = 𝑏, |𝐷𝐹| = 𝑐 and |𝐴𝐹| = 𝑑.
|𝐵𝐸|
Determine the ratio |𝐴𝐸| in terms of 𝑎, 𝑏, 𝑐 and 𝑑

Problem 134 Jamuna lowers five matching wooden discs over bars placed on the vertices
of a regular pentagon. Then she leaves five smaller congruent checkers these rods drop.
Then she stretches a ribbon around the large discs and a second ribbon around the small
discs. The first ribbon has a length of 56 centimeters and the second one of 50 centimeters.
jamuna looks at her construction from above and sees an area demarcated by the two
ribbons, what is the area of that area

38
B. Unsolved Challenges

Problem 135 Find the number of pairs of integers (𝑥, 𝑦) such that:
2
6(𝑥! + 3) = 𝑦 + 5

Problem 136 How many ways can the vertices of a cube be colored red or blue so that the
color of each vertex is the color of the majority of the three vertices adjacent to it?

Problem 137 Consider the following three lines in the Cartesian plane:
{ℓ1: 2𝑥 − 𝑦 = 7 ℓ2: 5𝑥 + 𝑦 = 42 ℓ3: 𝑥 + 𝑦 = 14

and let 𝑓𝑖(𝑃) correspond to the reflection of the point 𝑃 across ℓ𝑖. Suppose 𝑋 and 𝑌 are
( (
points on the 𝑥 and 𝑦 axes, respectively. such that 𝑓1 𝑓2 𝑓3(𝑋) )) = 𝑌. Let 𝑡 be the length of
2
segment 𝑋𝑌; what is the sum of all possible values of 𝑡 ?

Problem 138 Let 𝐴𝐵𝐶 be a triangle. Let Ω denote the incircle of △𝐴𝐵𝐶 having radius 𝑟𝑜.
Draw tangents to Ω which are parallel to the sides of 𝐴𝐵𝐶. Let Ω1, Ω2, Ω3 be the inradii of the
1
three corner triangles so formed each having equal radii of 9
. Also, tangents to these three
circles are drawn which are also parallel to the sides of the inner corner triangles. Find the
minimum value of perimeter of triangle 𝐴𝐵𝐶.

Problem 139 Suppose that in a certain society, each pair of persons can be classified as
either amicable or hostile. We shall say that each member of an amicable pair is a friend of
the other, and each member of a hostile pair is a foe of the other. Suppose that the society
has 𝑛 persons and 𝑞 amicable pairs, and that for every set of three persons, at least one pair

39
B. Unsolved Challenges

is hostile. Prove that there is at least one member of the society whose does include
(
𝑞 1 − 4𝑞/𝑛
2
) or fewer amicable pairs.

Problem 140 Consider a function 𝑓: 𝑁→𝑁


Suppose that for all 𝑚, 𝑛∈𝑁, exactly one of
𝑓(𝑚 + 1), 𝑓(𝑚 + 2), ⋯, 𝑓(𝑚 + 𝑓(𝑛))

is divisible by 𝑛. Prove that 𝑓 has an infinite number of fixed points (inputs that get mapped
to themselves).

Problem 141 Two trains start from point A and point B simultaneously towards each
other. Their initial speed is 0 after which they move with some uniform speed. Then, they
accelerate for a while (acceleration of both trains is different) before attaining some
uniform speed again. The ratio of speeds during uniform motion is 4:3. At the time of their
meeting, the speeds of the train were equal and they arrived at point A and B
𝑎
simultaneously. If the ratio of the accelerations of the train is given by 2 , where a,b are
𝑏+𝑘
2 3
twin prime and k is an integer, calculate 𝑎 + 𝑏 + 𝑘 .

Problem 142 The circumference of a circle is divided into 𝑝 equal parts by the points
𝐴1, 𝐴2, ⋯𝐴𝑝, where 𝑝 is an odd prime number.
How many different self-intersecting 𝑝-gons are there with these points as vertices if two 𝑝
-gons are considered different only when neither of them can be obtained from the other by
rotating the circle? (A self-intersecting polygon is a polygon such that some of its sides
intersect at other points besides the vertices).

Problem 143 If A and B are two rectangles with integer sides such that perimeter of A =
area of B and perimeter of B = area of A, then we call A and B friendly pair of rectangles.

40
B. Unsolved Challenges

Set S contains all {𝑎, 𝑏, 𝑐, 𝑑} such that (𝑎 * 𝑏, 𝑐 * 𝑑) are dimensions of friendly pairs of
rectangles. Call (𝑎𝑖, 𝑏𝑖, 𝑐𝑖, 𝑑𝑖)∀ 1 ≤ 𝑖 ≤ 𝑛(𝑆) where 𝑛(𝑆) is the cardinality of set S cyclic if
(𝑎𝑖, 𝑏𝑖, 𝑐𝑖, 𝑑𝑖) given in order are sides of a cyclic φquadrilateral.

Denote 𝐴𝑟(𝑖) as the area of cyclic quadrilateral having sides cyclic. Denote Ω(𝑖) as
, where ɸ and φ are adjacent angles of cyclic quadrilateral having sides
cyclic. Denote ω(𝑖) as .
(Note: i is assorted in increasing order of perimeter of cyclic quadrilateral, i.e. indice 1 is
assigned to cyclic quadrilateral having minimum perimeter and indice 𝑛(𝑆) is assigned to
cyclic quadrilateral having maximum perimeter and is greatest integer less than or
equal to x)

Let XYZ be an equilateral triangle, extend XY beyond Y to a point 𝑌' so that 𝑌𝑌' = .
XY, similarly extend YZ beyond Z to a point 𝑍' so that 𝑍𝑍' = Ω(𝑖). YZ and extend XZ beyond
3
X to a point 𝑋' so that 𝑋𝑋' = ω(𝑖) · 𝑋𝑍. If area ∆𝑋𝑌𝑍 = 4
remains constant but length
of 𝑋𝑋', 𝑌𝑌', 𝑍𝑍' changes for each value of i, then sum of all possible values of area of ∆𝑋'𝑌'𝑍'
is K.

Compute .

Problem 144 Two touching circles with fixed center 𝐴 and 𝐵 respectively having same
radii 𝑟. 𝐴 third circle touching both of circles is drawn with center 𝑂 and radius 𝑟1. another
circle which is moving in the plane with center 𝐶 and radius 𝑅 is drawn externally tangent
to circle with center 𝑂. Again, two circles with center 𝐷 and 𝐸 are drawn such that they are
externally tangent to circles with center 𝐴, 𝑂 and 𝐵, 𝑂 respectively. if the minimum
perimeter of the pentagon 𝐴𝐵𝐸𝐶𝐷 is obtained for 𝑅 = 𝑘 · 𝑟1. find value of k

2 2
Problem 145 x and y are real numbers such that 6 − 𝑥, 3 + 𝑦 , 11 + 𝑥, 14 − 𝑦 are
greater than zero. Find the maximum of the function

41
B. Unsolved Challenges

Problem 146 Fix an integer 𝑛 ≥ 4. Let 𝐶𝑛 be the collection of all 𝑛-point configurations in
the plane, every three points of which span a triangle of area strictly greater than 1. For
each configuration 𝐶 ∈ 𝐶𝑛 let 𝑓(𝑛, 𝐶) be the maximal size of a sub configuration of 𝐶 subject
to the condition that every pair of distinct points has distance strictly greater than 2.
Determine the minimum value 𝑓(𝑛) which 𝑓(𝑛, 𝐶) achieves as 𝐶 runs through 𝐶𝑛.

Problem 147 Consider the sequence 𝑥𝑛 > 0 defined with the following recurrence
relation 𝑥1 = 0 and for 𝑛 > 1

2 2
( 𝑛
)
(𝑛 + 1) 𝑥𝑛+1 + 2 + 4 (𝑛 + 1)𝑥𝑛+1 + 2
𝑛+1
+2
2𝑛−2 2 2
= 9𝑛 𝑥𝑛 + 36𝑛𝑥𝑛 + 32.

Show that if 𝑛 is a prime number larger or equal to 5, then 𝑥𝑛 is an integer

Problem 148 A jalebi is a loop of 2𝑎 + 2𝑏 + 4 unit squares which can be obtained by


cutting a concentric 𝑎 × 𝑏 hole out of an (𝑎 + 2) × (𝑏 + 2) rectangle, for some positive
integers 𝑎 and 𝑏. (The side of length 𝑎 of the hole is parallel to the side 𝑎 + 2 of the
rectangle).
Consider an infinite grid of unit square cells. For each even integer 𝑛 ≥ 8, a bakery of order
𝑛 is a finite set of cells 𝑆 such that, for every 𝑛-cell jalebi 𝐵 in the grid, there exists a
congruent copy of 𝐵 all of whose cells are in 𝑆. (The copy can be translated and rotated). We
denote by 𝑓(𝑛) the smallest possible number of cells in a bakery of order 𝑛. Find a real
number α such that, for cell sufficiently large even integers 𝑛 ≥ 8, we have
1 𝑓(𝑛)
100
< α < 100
𝑛

Problem 149 8𝑎𝑛+3 − 6𝑎𝑛+1 − 𝑎𝑛 = 0 for every 𝑛 ≥ 1∈𝑍 𝑎1 = 𝑎2 = 𝑎3 = 1

42
B. Unsolved Challenges

𝑎𝑘
Let 𝑓(𝑘) = 𝑎𝑘−1
for every 𝑘 ≥ 2∈𝑍 and let 𝑔(𝑘) = 𝑓(𝑘 + 1) − 𝑓(𝑘)

( ) ( ) ( ) ( ◦)
Show that if 𝑔 𝑘1 < 0 and 𝑔 𝑘2 > 0 then 𝑓 𝑘2 < 𝑐𝑜𝑠 20 < 𝑓 𝑘1 ( )

Problem 150 Define 𝑓: 𝑁→𝑁 ∀𝑛 ∊ 𝑁

Computer remainder when A is divided by 2023.

43
Indian National Mathematics Olympiad (INMO)
Problems

(1986 - 2022)

44
Problem 1 A person who left home between 4 p.m. and 5 p.m. returned between 5 p.m.
and 6 p.m. and found that the hands of his watch had exactly exchanged places, when
did he go out ? INMO 1986

Problem 2 Solve

INMO 1986

Problem 3 Two circles with radii a and b respectively touch each other externally. Let c
be the radius of a circle that touches these two circles as well as a common tangent to
the two circles. Prove that

INMO 1986

Problem 4 Find the least natural number whose last digit is 7 such that it becomes 5
times larger when this last digit is carried to the beginning of the number.
INMO 1986

Problem 5 If is a polynomial with integer coefficients and , , , three distinct


integers, then show that it is impossible to have , ,
INMO 1986

45
Problem 6 Construct a quadrilateral which is not a parallelogram, in which a pair of
opposite angles and a pair of opposite sides are equal.
INMO 1986

Problem 7 If , , , are integers greater than 1 such that and have no common
factor except 1 and show that , for some integer greater
than 1.
INMO 1986

Problem 8 Suppose are six sets each with four elements and
are sets each with two elements, Let
. Given that each elements of belongs to exactly four of the 's and to exactly three
of the 's, find .
INMO 1986

Problem 9 Show that among all quadrilaterals of a given perimeter the square has the
largest area.
INMO 1986

Problem 10 Given and as relatively prime positive integers greater than one, show

that is not a rational number.


INMO 1987

46
Problem 11 Determine the largest number in the infinite sequence

INMO 1987

Problem 12 Let be the set of all triplets of integers such that


For each triplet in , take number . Add all
these numbers corresponding to all the triplets in . Prove that the answer is divisible
by 7.
INMO 1987

Problem 13 If , , , and are natural numbers, and then prove that the
relation does not hold.
INMO 1987

Problem 14 Find a finite sequence of 16 numbers such that:


(a) it reads same from left to right as from right to left.
(b) the sum of any 7 consecutive terms is ,
(c) the sum of any 11 consecutive terms is .
INMO 1987

Problem 15 Prove that if coefficients of the quadratic equation are


odd integers, then the roots of the equation cannot be rational numbers.
INMO 1987

47
Problem 16 Construct the , given , (the altitudes from and ) and
, the median from the vertex .
INMO 1987

Problem 17 Three congruent circles have a common point and lie inside a given
triangle. Each circle touches a pair of sides of the triangle. Prove that the incentre and
the circumcentre of the triangle and the common point are collinear.
INMO 1987

Problem 18 Prove that any triangle having two equal internal angle bisectors (each
measured from a vertex to the opposite side) is isosceles.
INMO 1987

Problem 19 Let be a rearrangement of the numbers


. Suppose that is odd. Prove that the product is
an even integer.
INMO 1988

Problem 20 Prove that the product of 4 consecutive natural numbers cannot be a perfect
cube.
INMO 1988

48
Problem 21 Five men, , , , , are wearing caps of black or white color without
each knowing the color of his cap. It is known that a man wearing black cap always
speaks the truth while the ones wearing white always tell lies. If they make the
following statements, find the color worn by each of them:
: I see three black caps and one white cap.
: I see four white caps
: I see one black cap and three white caps
: I see your four black caps.
INMO 1988

Problem 22 If and are positive and , prove that

INMO 1988

Problem 23 Show that there do not exist any distinct natural numbers , , , such
that and .
INMO 1988

Problem 24 If are the coefficients of the polynomial

show that is even.


INMO 1988

49
Problem 25 Given an angle and a point outside the angle . Draw a
straight line through meeting in and in such that the triangle
has a given perimeter.
INMO 1988

Problem 26 A river flows between two houses and , the houses standing some
distances away from the banks. Where should a bridge be built on the river so that a
person going from to , using the bridge to cross the river may do so by the
shortest path? Assume that the banks of the river are straight and parallel, and the
bridge must be perpendicular to the banks.
INMO 1988

Problem 27 Show that for a triangle with radii of circumcircle and incircle equal to ,
respectively, the inequality holds.
INMO 1988

Problem 28 Prove that the Polynomial can't


be expressed as a product , where and are both
polynomial with integral coefficients and with degree at least .
INMO 1989

Problem 29 Let and be any four real numbers, not all equal to zero. Prove that
the roots of the polynomial can't all be real.
INMO 1989

50
Problem 30 Let denote a subset of the set having the
property that no two elements of add up to . Prove that can't have more than
elements.
INMO 1989

Problem 31 Determine all for which is not the square of any integer,
divides .|
INMO 1989

Problem 32 For positive integers , define to be . Determine the sets of


positive integers for which

(a) is an even number,

(b) is a multiple of .
INMO 1989

Problem 33 Triangle has incentre and the incircle touches at


respectively. Let meet at . Show that is perpendicular to .
INMO 1989

Problem 34 Let be one of the two points of intersection of two circles with centers
respectively.The tangents at to the two circles meet the circles again at
. Let a point be located so that is a parallelogram. Show that is also the
circumcenter of triangle .
INMO 1989

51
Problem 35 Given the equation

has four real, positive roots, prove that


(a)
(b)
with equality in each case holding if and only if the four roots are equal.
INMO 1990

Problem 36 Determine all non-negative integral pairs for which

INMO 1990

Problem 37 Let be a function defined on the set of non-negative integers and taking
values in the same
set. Given that

(a) for all non-negative integers ;

(b) ,

find the possible values that can take.


(Notation : here refers to largest integer that is , e.g. ).
INMO 1990

52
Problem 38 Consider the collection of all three-element subsets drawn from the set
.
Determine the number of those subsets for which the sum of the elements is a multiple
of 3.
INMO 1990

Problem 39 Let , , denote the sides of a triangle. Show that the quantity

must lie between the limits and 2. Can equality hold at either limit?
INMO 1990

Problem 40 Triangle is scalene with angle having a measure greater than 90


degrees. Determine
the set of points that lie on the extended line , for which

where refers to the (positive) distance between and .


INMO 1990

Problem 41 Let be an arbitrary acute angled triangle. For any point lying
within the triangle, let
, , denote the feet of the perpendiculars from onto the sides , ,
respectively.
Determine the set of all possible positions of the point for which the triangle
is isosceles.
For which position of will the triangle become equilateral?
INMO 1990

53
Problem 42 Find the number of positive integers for which

(i) ;

(ii) 6 is a factor of .
INMO 1991

Problem 43 Given an acute-angled triangle , let points be located as


follows: is the point where altitude from on meets the outwards-facing
semicircle on as diameter. Points are located similarly.
Prove that where is
the area of triangle .
INMO 1991

Problem 44 Given a triangle let

Prove that .
INMO 1991

54
Problem 45 Let be real numbers with , , , and
.

Prove that .
INMO 1991

Problem 46 Triangle has an incenter . Let points , be located on the line


segments , respectively, so that and
. Given that the points lie on a straight line, find the possible values of the
measure of angle .
INMO 1991

Problem 47 (i) Determine the set of all positive integers for which divides
;

(ii) Prove that does not divide for any positive integer .
INMO 1991

Problem 48 Solve the following system for real

INMO 1991

55
Problem 49 There are objects of total weight , each of the weights being a positive
integers. Given that none of the weights exceeds , prove that the ten objects can be
divided into two groups that balance each other when placed on 2 pans of a balance.
INMO 1991

Problem 50 Triangle has an incenter l its incircle touches the side at


. The line through parallel to meets the incircle again at and the tangent to
the incircle at meets at points respectively. Prove that triangle
is similar to triangle .
INMO 1991

Problem 51 For any positive integer , let denote the number of ordered pairs

of positive integers for which . Determine the set of positive


integers for which
INMO 1991

Problem 52 In a triangle , . Prove that .


INMO 1992

Problem 53 If such that and , then show

that each of lies in the closed interval . Can attain the extreme value
or ?
INMO 1992

56
Problem 54 Find the remainder when is divided by 92.
INMO 1992

Problem 55 Find the number of permutations of


such that for any , does not form a permutation of
.
INMO 1992

Problem 56 Two circles and intersect at two distinct points in a plane. Let a
line passing through meet the circles and in and respectively. Let
be the midpoint of and let meet the circles and in and
respectively. Show that is also the midpoint of .
INMO 1992

Problem 57 Let be a polynomial in with integer coefficients and suppose that for
five distinct integers one has . Show
that there does not exist an integer such that .
INMO 1992

Problem 58 Let be an integer. Find the number of ways in which one can place
the numbers in the squares of a chess board, one on each,
such that the numbers in each row and in each column are in arithmetic progression.
INMO 1992

57
Problem 59 Determine all pairs of positive integers for which is a
perfect square.
INMO 1992

Problem 60 Let be an -sided regular polygon. If

, find .
INMO 1992

Problem 61 Determine all functions such that

INMO 1992

Problem 62 The diagonals and of a cyclic quadrilateral intersect at


. Let be the circumcenter of triangle and be the orthocenter of triangle
. Show that the points are collinear.
INMO 1993

Problem 63 Let be a quadratic polynomial with . Given


any integer , show that there is an integer such that
INMO 1993

58
Problem 64 If and , show that
INMO 1993

Problem 65 Let be a triangle in a plane . Find the set of all points (distinct
from ) in the plane such that the circumcircles of triangles , ,
have the same radii.
INMO 1993

Problem 66 Show that there is a natural number such that when written in decimal
notation ends exactly in 1993 zeros.
INMO 1993

Problem 67 Let be a triangle right-angled at and be its circumcircle. Let


be the circle touching the lines and , and the circle internally. Further, let
be the circle touching the lines and and the circle externally. If
be the radii of prove that .
INMO 1993

Problem 68 Let and be a subset of having elements.


Show that has 2 distinct elements and whose sum is divisible by .
INMO 1993

59
Problem 69 Let be a bijective function from to itself. Show that
there is a positive integer such that for each in , where
denotes the composition times.
INMO 1993

Problem 70 Show that there exists a convex hexagon in the plane such that

(i) all its interior angles are equal;

(ii) its sides are in some order.


INMO 1993

Problem 71 Let be the centroid of the triangle in which the angle at is


obtuse and and be the medians from and respectively onto the sides

and . If the points and are concyclic, show that


. If further is a point on the line extended such that is a parallelogram,
show that triangle and are similar.
INMO 1994

Problem 72 If prove that .


INMO 1994

Problem 73 In any set of square integers, prove that one can always find a subset of
numbers, sum of whose elements is divisible by
INMO 1994

60
Problem 74 Find the number of nondegenerate triangles whose vertices lie in the set of
points in the plane such that , , and are integers.
INMO 1994

Problem 75 A circle passes through the vertex of a rectangle and touches its
sides and at and respectively. If the distance from to the line
segment is equal to units, find the area of rectangle .
INMO 1994

Problem 76 Find all real-valued functions on the reals such that ,

for all , and for


INMO 1994

Problem 77 In an acute angled triangle , , is the orthocenter, and


is the midpoint of . On the line , take a point such that
. Show that .
INMO 1995

Problem 78 Show that there are infinitely many pairs of relatively prime integers

(not necessarily positive) such that both the equations have


integer roots.
INMO 1995

61
Problem 79 Show that the number of element subsets of
with is less than the number of those with
INMO 1995

Problem 80 Let be a triangle and a circle be drawn lying outside the triangle,
touching its incircle externally, and also the two sides and . Show that the

ratio of the radii of the circles and is equal to


INMO 1995

Problem 81 Let . Let be real numbers all less than and such
that for . Show that

INMO 1995

Problem 82 Find all primes for which the quotient is a square.


INMO 1995

Problem 83 a) Given any positive integer , show that there exist distinct positive
integers and such that divides for ;

b) If for some positive integers and , divides for all positive integers
, prove that
INMO 1996

62
Problem 84 Let and be two concentric circles in the plane with radii and
respectively. Show that the orthocenter of any triangle inscribed in circle lies in the
interior of circle . Conversely, show that every point in the interior of is the
orthocenter of some triangle inscribed in .
INMO 1996

Problem 85 Solve the following system for real :

INMO 1996

Problem 86 Let be a set containing elements. Find the number of ordered triples
of subsets of such that is a subset of and is a proper subset of
.
INMO 1996

Problem 87 Define a sequence by and and


for . prove that for any , is also a term in
this sequence.
INMO 1996

63
Problem 88 There is a array (matrix) consisting of and and there are
exactly zeroes. Show that it is possible to remove all the zeros by deleting some
rows and some columns.
INMO 1996

Problem 89 Let be a parallelogram. Suppose a line passing through and


lying outside the parallelogram meets and produced at and
respectively. Show that
INMO 1997

Problem 90 Show that there do not exist positive integers and such that

INMO 1997

Problem 91 If are three real numbers and for some


real number , prove that
INMO 1997

Problem 92 In a unit square one hundred segments are drawn from the center to the
sides dividing the square into one hundred parts (triangles and possibly

quadrilaterals). If all parts have equal perimeter , show that .


INMO 1997

64
Problem 93 Find the number of array whose entries are from the set
and which are such that the sum of the numbers in each of the four rows and in each of
the four columns is divisible by .
INMO 1997

Problem 94 Suppose and are two positive real numbers such that the roots of the
cubic equation are all real. If is a root of this cubic with minimal

absolute value, prove that


INMO 1997

Problem 95 In a circle with center , let be a chord that is not a diameter. Let
be the midpoint of this chord . Take a point on the circle with as
diameter. Let the tangent to at meet at . Show that
.
INMO 1998

Problem 96 Let and be two positive rational numbers such that is also a
rational number. Prove that and themselves are rational numbers.
INMO 1998

Problem 97 Let be four integers such that is not divisible by . If there is an


integer such that is divisible by 5, prove that there is an
integer such that is also divisible by 5.
INMO 1998

65
Problem 98 Suppose is a cyclic quadrilateral inscribed in a circle of radius one
unit. If , prove that is a square.
INMO 1998

Problem 99 Suppose are three real numbers such that the quadratic equation
has roots of the form where
and are real numbers. Show that
(i) The numbers are all positive.
(ii) The numbers form the sides of a triangle.
INMO 1998

Problem 100 It is desired to choose integers from the collection of integers,


namely, such that the average of these chosen
integers is itself an integer and as minimum as possible. Show that this can be done for
each positive integer and find this minimum value for each .
INMO 1998

Problem 101 Let be an acute-angled triangle in which are points on


respectively such that ; ; and bisects
internally, Suppose meets and in and respectively. If ,
, , find the perimeter of .
INMO 1999

66
Problem 102 In a village persons volunteered to clean up, for a fair, a rectangular
field with integer sides and perimeter equal to feet. For this purpose, the field
was divided into equal parts. If each part had an integer area, find the length and
breadth of the field.
INMO 1999

Problem 103 Show that there do not exist polynomials and each having
integer coefficients and of degree greater than or equal to 1 such that

INMO 1999

Problem 104 Let and be two concentric circles. Let and be any two
equilateral triangles inscribed in and respectively. If and are any two
points on and respectively, show that

INMO 1999

Problem 105 Given any four distinct positive real numbers, show that one can choose
three numbers from among them, such that all three quadratic equations

have only real roots, or all three equations have only imaginary
roots.
INMO 1999

67
Problem 106 For which positive integer values of can the set be
split into disjoint -element subsets such that in each of these sets

.
INMO 1999

Problem 107 The incircle of touches , , at , , respectively.


The line through parallel to meets at , and the line through parallel
to meets at . Show that the line bisects and bisects .
INMO 2000

Problem 108 Solve for integers :


INMO 2000

Problem 109 If are real numbers such that and

then prove that .


INMO 2000

Problem 110 In a convex quadrilateral , , and


. Prove that
INMO 2000

68
Problem 111 Let be three real numbers such that . prove
that if is a root of the cubic equation (real or complex),
then
INMO 2000

Problem 112 For any natural numbers , ( ), let denote the number of
congruent integer-sided triangles with perimeter . Show that

(i) ;

(ii) .
INMO 2000

Problem 113 Let be a triangle in which no angle is . For any point in the
plane of the triangle, let denote the reflections of in the sides
respectively. Prove that

(i) If is the incenter or an excentre of , then is the circumcenter of


;

(ii) If is the circumcentre of , then is the orthocentre of ;

(iii) If is the orthocentre of , then is either the incentre or an excentre of


.
INMO 2001

Problem 114 Show that the equation has


infinitely many solutions in integers .
INMO 2001

69
Problem 115 If are positive real numbers such that , Prove that

INMO 2001

Problem 116 Show that given any nine integers, we can find four, such that
is divisible by . Show that this is not always true for eight integers.
INMO 2001

Problem 117 is a triangle. is the midpoint of . , and


. Show that is obtuse. If is the circumcenter of
, show that is equilateral.
INMO 2001

Problem 118 Find all functions such that for


all
INMO 2001

70
Problem 119 For a convex hexagon in which each pair of opposite sides is
unequal, consider the following statements.

( ) is parallel to .( ) .

( ) is parallel to .( ) .

( ) is parallel to .( ) .

Show that if all six of these statements are true then the hexagon is cyclic.

Prove that, in fact, five of the six statements suffice.


INMO 2002

Problem 120 Find the smallest positive value taken by for positive
integers , , .
Find all , , which give the smallest value
INMO 2002

Problem 121 If , are positive reals such that show that


.
INMO 2002

Problem 122 Is it true that there exist 100 lines in the plane, no three concurrent, such
that they intersect in exactly 2002 points?
INMO 2002

71
Problem 123 Do there exist distinct positive integers , , such that , , ,
, , , form an arithmetic progression (in
some order).
INMO 2002

Problem 124 The numbers , , are arranged in an array, so that the


numbers in each row increase from left to right, and the numbers in each column
increase from top to bottom. Let be the number in position . Let be the

number of possible values for . Show that


INMO 2002

Problem 125 Let be an interior point of an acute-angled triangle . The line


meets the line at , and the line meets the line at . The lines
and intersect each other at . Let be the foot of the perpendicular from the
point to the line . Show that the line bisects the angle .
INMO 2003

Problem 126 Find all primes and even such that


.
INMO 2003

Problem 127 Show that has at least one real root


for all real . Find the sum of the non-real roots.
INMO 2003

72
Problem 128 Find all -digit numbers which use only the digits and and are
divisible by .
INMO 2003

Problem 129 Let a, b, c be the side lengths and S the area of a triangle ABC. Denote

, and . Prove that there exists a triangle with side

lengths x, y, z, and the area of this triangle is .


INMO 2003

Problem 130 Each lottery ticket has a 9-digit numbers, which uses only the digits , ,
. Each ticket is colored red, blue or green. If two tickets have numbers which differ in all
nine places, then the tickets have different colors. Ticket is red, and ticket
is green. What color is ticket ?
INMO 2003

Problem 131 is a convex quadrilateral. , , , are the midpoints of the


sides , , , . bisects at . , and

. Prove that is a square


INMO 2004

Problem 132 is a prime. Find all integers , , such that


.
INMO 2004

73
Problem 133 If is a real root of , show that
INMO 2004

Problem 134 is a triangle, with sides , , , circumradius , and exradii , ,


If , show that , , , and .
INMO 2004

Problem 135 S is the set of all ( , , , , , ) where , , , , , are integers such


that . Find the largest which divides abcdef for all
members of .”
INMO 2004

Problem 136 Show that the number of 5-tuples ( , , , , ) such that


is odd
INMO 2004

Problem 137 Let be the midpoint of side of a triangle . Let the median
intersect the incircle of at and being nearer to
than . If , prove that the sides of triangle are in the ratio
in some order.
INMO 2005

74
Problem 138 Let and be positive integers such that . Find the
minimum possible value of .
INMO 2005

Problem 139 Let be positive real numbers, not all equal, such that some two of

the equations have a common root, say . Prove that

is real and negative;

the remaining third quadratic equation has non-real roots.


INMO 2005

Problem 140 All possible -digit numbers, in each of which the digits occur in
nonincreasing order (from left to right, e.g. ) are written as a sequence in
increasing order. Find the -the number in this sequence.
INMO 2005

Problem 141 Let be a given positive integer. A sequence of positive integers


is such that , for , is obtained from by adding some nonzero digit of
. Prove that

a) the sequence contains an even term;

b) the sequence contains infinitely many even terms.


INMO 2005

75
Problem 142 Find all functions such that
for all .
INMO 2005

Problem 143 In a non equilateral triangle the sides form an arithmetic


progression. Let be the incentre and the circumcentre of the triangle Prove
that

(1) is perpendicular to ;

(2) If meets in , and , are the midpoints of , respectively


then is the circumcentre of triangle .
INMO 2006

Problem 144 Prove that for every positive integer there exists a unique ordered pair
of positive integers such that

INMO 2006

Problem 145 Let denote the set of all triples of integers. Define
by
Find all triples such that
INMO 2006

76
Problem 146 Some 46 squares are randomly chosen from a chess board and
colored in red. Show that there exists a block of 4 squares of which at least three
are colored in red.
INMO 2006

Problem 147 In a cyclic quadrilateral , , , ,


and . Prove that

(1) ;

(2) .
INMO 2006

Problem 148(a) Prove that if is a integer such that then there exists an

integer such that

(b) Find the smallest positive integer for which whenever an integer is such that

then there exists an integer such that


INMO 2006

Problem 149 In a triangle right-angled at , the median through bisects the


angle between and the bisector of . Prove that

INMO 2007

77
Problem 150 Let be a natural number such that for some natural
numbers . Prove that

where 's , 's , 's are all nonzero integers. Further, if does not divide at least
one of prove that can be expressed in the form , where
are natural numbers none of which is divisible by .
INMO 2007

Problem 151 Let and be positive integers such that has real
roots and .

Prove that and are integers if and only if is the square of an


integer.

(Here denotes the largest integer not exceeding )


INMO 2007

Problem 152 Let be permutation of . A pair


is said to correspond to an inversion of if but . How many
permutations of , , have exactly two inversions?

For example, In the permutation , there are 6 inversions corresponding to


the pairs .
INMO 2007

78
Problem 153 Let be a triangle in which . Let be the midpoint of
and be a point on . Suppose is the foot of perpendicular from on
. Define

Prove that

Hence show that and if and only if is equilateral.


INMO 2007

Problem 154 If , , are positive real numbers, prove that

INMO 2007

Problem 155 Let be triangle, its in-center; be the reflections of


in respectively. Suppose the circum-circle of triangle passes
through . Prove that are concyclic, where is the in-center of triangle
.
INMO 2008

Problem 156 Find all triples such that , where is a prime and
and are natural numbers.
INMO 2008

79
Problem 157 Let be a set of real numbers such that has at least four elements.
Suppose has the property that is a rational number for all distinct
numbers in . Prove that there exists a positive integer such that is
a rational number for every in .
INMO 2008

Problem 158All the points with integer coordinates in the -Plane are coloured using
three colors, red, blue and green, each color being used at least once. It is known that
the point is red and the point is blue. Prove that there exist three points
with integer coordinates of distinct colors which form the vertices of a right-angled
triangle.
INMO 2008

Problem 159 Let be a triangle; be three equal, disjoint circles inside


such that touches and ; touches and ; and
touches and . Let be a circle touching circles externally. Prove
that the line joining the circum-center and the in-center of triangle passes
through the center of .
INMO 2008

Problem 160 Let be a polynomial with integer coefficients. Prove that there exist
two polynomials and , again with integer coefficients, such that
(i) is a polynomial in , and
(ii) is a polynomial in .
INMO 2008

80
Problem 161 Let be a triangle and let be an interior point such that
.Let be the mid points of
respectively.Suppose .Prove that are collinear.
INMO 2009

Problem 162 Define a a sequence as follows

, if number of positive divisors of is odd


, if number of positive divisors of is even

(The positive divisors of include as well as .)Let be the real


number whose decimal expansion contains in the -th place,
.Determine,with proof,whether is rational or irrational.
INMO 2009

Problem 163 Find all real numbers such that:

(Here denotes the largest integer not exceeding .)


INMO 2009

Problem 164 All the points in the plane are colored using three colors.Prove that there
exists a triangle with vertices having the same color such that either it is isosceles or its
angles are in geometric progression.
INMO 2009

81
Problem 165 Let be an acute angled triangle and let be its ortho center. Let
denote the largest altitude of the triangle . Prove that:

INMO 2009

Problem 166 Let be positive real numbers such that .Prove that:

.
INMO 2009

Problem 167 Let be a triangle with circum-circle . Let be a point in the


interior of triangle which is also on the bisector of . Let
meet in respectively. Suppose is the point of intersection of
with ; and is the point of intersection of with . Prove that is
parallel to .
INMO 2010

Problem 168 Find all natural numbers such that does divide .
INMO 2010

Problem 169 Find all non-zero real numbers which satisfy the system of
equations:

INMO 2010

82
Problem 170 How many 6-tuples are there such that each of
is from the set and the six expressions

for (where is to be taken as ) are all equal to one another?


INMO 2010

Problem 171Let be an acute-angled triangle with altitude . Let be its


ortho-center and be its circum-center. Suppose is an acute-angled triangle
and its circum-center. Let be the reflection of in the line . Show that
lies on the line joining the mid-points of and .
INMO 2010

Problem 172 Define a sequence by , and

for

For every and prove that divides


.

Suppose divides for some natural numbers and . Prove that divides

INMO 2010

Problem 173 Let be points on the sides respectively of a


triangle such that and
Show that is equilateral.
INMO 2011

83
Problem 174 Call a natural number faithful if there exist natural numbers
such that and and
Show that all but a finite number of natural numbers are faithful.
Find the sum of all natural numbers which are not faithful.
INMO 2011

Problem 175 Let and


be two polynomials with integral coefficients
such that is a prime and and Suppose that
there exists a rational number such that Prove that
INMO 2011

Problem 176 Suppose five of the nine vertices of a regular nine-sided polygon are
arbitrarily chosen. Show that one can select four among these five such that they are
the vertices of a trapezium.
INMO 2011

Problem 177 Let be a cyclic quadrilateral inscribed in a circle Let


be the midpoints of arcs of respectively. Suppose
that Show that are all concurrent.
INMO 2011

Problem 178 Find all functions satisfying


For all .
INMO 2011

84
Problem 179 Let be a quadrilateral inscribed in a circle. Suppose

and subtends degrees at the center of the circle . Find the


maximum possible area of .
INMO 2012

Problem 180 Let and be two sets of prime


numbers, such that and . Suppose and
. Prove that divides .
INMO 2012

Problem 181 Define a sequence of functions by


for . Prove that each
is a polynomial with integer coefficients.
INMO 2012

Problem 182 Let be a triangle. An interior point of is said to be good if


we can find exactly rays emanating from intersecting the sides of the triangle
such that the triangle is divided by these rays into smaller triangles of equal
area. Determine the number of good points for a given triangle .
INMO 2012

Problem 183 Let be an acute angled triangle. Let be points on


such that is the median, is the internal bisector and is
the altitude. Suppose that and
Show that is equilateral.
INMO 2012

85
Problem 184 Let be a function satisfying , and

for all , simultaneously.

Find the set of all possible values of the function .

If and , find the set of all integers such that .


INMO 2012

Problem 185 Let and be two circles touching each other externally at Let
and be the centers of and respectively. Let be a line which is tangent to
at and passing through and let be the line tangent to at and
passing through Let If then prove that the triangle
is equilateral.
INMO 2013

Problem 186 Find all and primes satisfying

INMO 2013

Problem 187 Let such that . Show that the equation


has no integer solution.
INMO 2013

86
Problem 188 Let be an integer greater than and let be the number of non empty
subsets of with the property that the average of the elements of is
an integer.Prove that is always even.
INMO 2013

Problem 189 In an acute triangle let be its circumcentre, centroid and


orthocenter. Let and Let be the
midpoint of If the triangles have the same area, find all the
possible values of
INMO 2013

Problem 190 Let be six positive real numbers satisfying


and Further, suppose that
and Prove that and
INMO 2013

Problem 191 In a triangle , let be the point on the segment such that
. Suppose that the points , and the centroids of
triangles and lie on a circle. Prove that .
INMO 2014

Problem 192 Let be a natural number. Prove that,

is even.
INMO 2014

87
Problem 193 Let be natural numbers with . Suppose that the sum of their
greatest common divisor and least common multiple is divisible by . Prove that

the quotient is at most . When is this quotient exactly equal to


INMO 2014

Problem 194 Written on a blackboard is the polynomial . Calvin and


Hobbes take turns alternately (starting with Calvin) in the following game. At his turn,
Calvin should either increase or decrease the coefficient of by . And at this turn,
Hobbes should either increase or decrease the constant coefficient by . Calvin wins if
at any point of time the polynomial on the blackboard at that instant has integer roots.
Prove that Calvin has a winning strategy.
INMO 2014

Problem 195 In a acute-angled triangle , a point lies on the segment . Let


denote the circumcenter of triangles and respectively. Prove
that the line joining the circumcentre of triangle and the orthocentre of triangle
is parallel to .
INMO 2014

Problem 196 Let be a natural number. Let , and define


to be the set of all those elements of which belong to exactly one of and . Show
that , where is a collection of subsets of such that for any two
distinct elements of of we have . Also find all such collections
for which the maximum is attained.
INMO 2014

88
Problem 197 Let be a right-angled triangle with . Let is the
altitude from on . Let and be the incenters of triangles
and respectively.Show that circumcenter of triangle lie on the
hypotenuse .
INMO 2015

Problem 198 For any natural number write the finite decimal expansion of (for

example we write as its infinite decimal expansion not . Determine the

length of non-periodic part of the (infinite) decimal expansion of .


INMO 2015

Problem 199 Find all real functions such that


.
INMO 2015

Problem 200 There are four basketball players . Initially the ball is with
. The ball is always passed from one person to a different person.
In how many ways can the ball come back to after moves? (for example
, or
.
INMO 2015

89
Problem 201 Let be a convex quadrilateral.Let diagonals and
intersect at . Let and are altitudes from on the side
and respectively. Show that has a incircle if and only if

INMO 2015

Problem 202 Show that from a set of square integers one can select six numbers
such that .
INMO 2015

Problem 203 Let be a triangle in which . Suppose the orthocentre of

the triangle lies on the incircle. Find the ratio .


INMO 2016

Problem 204 For positive real numbers which of the following statements
necessarily implies : (I) , (II)
? Justify your answer.
INMO 2016

Problem 205 Let denote the set of natural numbers. Define a function by
and . We write and in general
for any .

(i) Show that for each , there exists such that .

90
(ii) For , let denote the number of elements in the set
. Prove that , for .
INMO 2016

Problem 206 Suppose points of the circumference of a circle are colored red and
the remaining points are colored blue . Given any natural number , prove that
there is a regular -sided polygon all of whose vertices are blue
INMO 2016

Problem 207 Let be a right-angle triangle with . Let be a point on


such that the inradii of the triangles and are equal. If this common
value is and if is the inradius of triangle , prove that

INMO 2016

Problem 208 Consider a non constant arithmetic progression


. Suppose there exist relatively prime positive integers and such that
and are also the terms of the same arithmetic progression. Prove that
the terms of the arithmetic progression are all integers.
INMO 2016

91
Problem 209 In the given figure, is a square sheet of paper. It is folded along
such that goes to a point different from and , on the side and
goes to . The line cuts in . Show that the inradius of the triangle
is the sum of the inradii of the triangles and .

INMO 2017

Problem 210 Suppose is an integer and all the roots of


are integers. Find all possible values of .
INMO 2017

Problem 211 Find the number of triples where is a real number and ,
belong to the set such that where
denotes the fractional part of the real number . (For example
.)
INMO 2017

92
Problem 212 Let be a convex pentagon in which
and the side lengths are five consecutive integers in
some order. Find all possible values of .
INMO 2017

Problem 213 Let be a convex pentagon in which


and the side lengths are five consecutive integers in
some order. Find all possible values of .
INMO 2017

Problem 214 Let be an integer and consider the sum

Show that
form the sides of a triangle whose area and inradius are also
integers.
INMO 2017

Problem 215 Let be a non-equilateral triangle with integer sides. Let and
be respectively the mid-points of and ; let be the centroid of
. Suppose, , , , are concyclic. Find the least possible perimeter of .
INMO 2018

93
Problem 216 For any natural number , consider a rectangular board made up of
unit squares. This is covered by types of tiles : red tile, green tile and
domino. (For example, we can have types of tiling when : red-red ;
red-green ; green-red ; green-green ; and blue.) Let denote the number of ways of
covering rectangular board by these types of tiles. Prove that, divides
.
INMO 2018

Problem 217 Let and be two circles with respective centers and
intersecting in two distinct points and such that is an obtuse angle.
Let the circumcircle of intersect and respectively in points
and . Let the line intersect in ; let the line intersect in
. Prove that, the points are concyclic.
INMO 2018

Problem 218 Find all polynomials with real coefficients such that
divides .
INMO 2018

Problem 219 There are girls in a class sitting around a circular table, each having
some apples with her. Every time the teacher notices a girl having more apples than
both of her neighbors combined, the teacher takes away one apple from that girl and
gives one apple each to her neighbors. Prove that this process stops after a finite
number of steps.
(Assume that the teacher has an abundant supply of apples.)
INMO 2018

94
Problem 220 Let denote set of all natural numbers and let be a function
such that

for all ;

divides for all .

Prove that, there exists an odd natural number such that for all in
INMO 2018

Problem 221 Let be a triangle with . Let be a point on the


segment and be a point on line such that is tangent to the
circumcircle of triangle at and is perpendicular to . Given that
and . Determine in degrees.
INMO 2019

Problem 222 Let be a regular pentagon.For , let


be the pentagon whose vertices are the midpoint of the sides
. All the vertices of each of the pentagons are
arbitrarily coloured red or blue. Prove that four points among these points have the
same color and form the vertices of a cyclic quadrilateral.
INMO 2019

Problem 223 Let be distinct positive integers. Prove that


Further,
determine when equality holds.
INMO 2019

95
Problem 224 Let and be positive integers such that . Prove that there
are distinct primes such that divides for all
.
INMO 2019

Problem 225 Let be the diameter of a circle and let be a point on different
from and . Let be the foot of perpendicular from onto .Let be a
point on the segment such that is equal to the semi perimeter of
.Show that the excircle of opposite is tangent to .
INMO 2019

Problem 226 Let be a function defined from real, to the set of


all positive real numbers such that
for all
for all
for all
Prove that
for all
for all
INMO 2019

Problem 227 Let and be two circles of unequal radii, with centers and
respectively, intersecting in two distinct points and . Assume that the center of
each circle is outside the other circle. The tangent to at intersects again in
, different from ; the tangent to at intersects again at , different from
. The bisectors of and meet and again in and
, respectively. Let and be the circumcenter of triangles and
, respectively. Prove that is the perpendicular bisector of the line segment .
INMO 2020

96
Problem 228 Suppose is a polynomial with real coefficients, satisfying the
condition , for every real . Prove that
can be expressed in the form
for some real
numbers and non-negative integer .
INMO 2020

Problem 229 Let be a subset of . Suppose there is a positive integer


such that for any integer , one can find positive integers so that
and all the digits in the decimal representations of (expressed without
leading zeros) are in . Find the smallest possible value of .
INMO 2020

Problem 230 Let be an integer and let be real


numbers such that . Prove that

INMO 2020

Problem 231 Infinitely many equidistant parallel lines are drawn in the plane. A positive
integer is called frameable if it is possible to draw a regular polygon with
sides all whose vertices lie on these lines, and no line contains more than one vertex of
the polygon.

(a) Show that are frameable.


(b) Show that any integer is not frameable.
(c) Determine whether is frameable.
INMO 2020

97
Problem 232 A stromino is a rectangle. Show that a board divided into
twenty-five squares cannot be covered by strominos such that each stromino
covers exactly three squares of the board, and every square is covered by one or two
strominos. (A stromino can be placed either horizontally or vertically on the board.)
INMO 2020

Problem 233 Suppose is an integer, and let be


integers such that for any two integers and satisfying
. Determine the maximum possible value of .
INMO 2021

Problem 234 Find all pairs of integers so that each of the two cubic polynomials
has all the roots to be integers.
INMO 2021

Problem 235 Betal marks points on the plane such that no three are collinear, and
draws all possible segments joining these. He then chooses any of these
segments, and marks their midpoints. Finally, he chooses a segment whose midpoint is
not marked yet, and challenges Vikram to construct its midpoint using only a
straightedge. Can Vikram always complete this challenge?

Note. A straightedge is an infinitely long ruler without markings, which can only be
used to draw the line joining any two given distinct points.
INMO 2021

98
Problem 236 A Magician and a Detective play a game. The Magician lays down cards
numbered from to face-down on a table. On each move, the Detective can point to
two cards and inquire if the numbers on them are consecutive. The Magician replies
truthfully. After a finite number of moves, the Detective points to two cards. She wins if
the numbers on these two cards are consecutive, and loses otherwise.

Prove that the Detective can guarantee a win if and only if she is allowed to ask at least
questions.
INMO 2021

Problem 237 In a convex quadrilateral , , ,


and . Extend to meet the circumcircle of triangle
at . Prove that .
INMO 2021

Problem 238 Let be the set of all polynomials with real coefficients. Find all
functions satisfying the following conditions:

a. maps the zero polynomial to itself,


b. for any non-zero polynomial , , and
c. for any two polynomials , the polynomials and
have the same set of real roots.
INMO 2021

99
Problem 239 Let be an interior point on the side of an acute-angled triangle
. Let the circumcircle of triangle intersect again at and the
circumcircle of triangle intersect again at . Let , , and
intersect the circumcircle of triangle again at , and
, respectively. Let and be the incentres of triangles and
, respectively. Prove that are concyclic.
INMO 2022

Problem 240 Find all natural numbers for which there is a permutation of
that satisfies:

INMO 2022

Problem 241 For a positive integer , let denote the number of arrangements of
the integers into a sequence such that for all ,
and for all , . For example,
is , since the possible arrangements are and

(a) Find
(b) If is the largest non-negative integer so that divides , show that
.
(c) Find the largest non-negative integer so that divides
INMO 2022

100
Indian Team Selection Test (TST) Problems

(2001 - 2019)

101
Problem 1 Let , , . Prove that if , then
.
TST 2001

Problem 2 Two symbols and obey the rule . Given a word


consisting of letters and letters , show that there is
a unique cyclic permutation of this word which reduces to .
TST 2001

Problem 3 In a triangle with incircle and incenter , the segments ,


, cut at , , , respectively. Rays , , meet the sides
, , at , , respectively. Prove that:

When does equality occur?


TST 2001

Problem 4 For any positive integer , show that there exists a polynomial of
degree with integer coefficients such that are all distinct
powers of .
TST 2001

102
Problem 5 Let be a cubic polynomial with integer coefficients. Suppose that a
prime divides for , , , , where are distinct
integers from the set . Prove that divides all the coefficients of
.
TST 2001

Problem 6 Find the number of all unordered pairs of subsets of an


-element set, such that and .
TST 2001

Problem 7 If on , triangles and are constructed externally


such that , .
, .
COnstructed externally on is triangle with ,
.
Prove that 1. is perpendicular to .

2. If is the projection of on , then prove that .


TST 2001

Problem 8 Find all functions satisfying : for all


.
TST 2001

103
Problem 9 Points are chosen on side of a
triangle in that order. Let be the inradius of triangle for
, and be the inradius of . Show that there is a constant
independent of such that :

TST 2001

Problem 10 Complex numbers , , have the property that is an


integer for every natural number . Prove that the polynomial
has integer coefficients.
TST 2001

Problem 11 Let be a prime. For each , define to be


the unique integer in such that and set
. Prove that :

TST 2001

Problem 12 Each vertex of an grid is colored blue, green or red in such a way
that all the boundary vertices are red. We say that a unit square of the grid is
properly colored if:
all the three colors occur at the vertices of the square, and
one side of the square has the endpoints of the same color.
Show that the number of properly colored squares is even.
TST 2001

104
Problem 13 Let be a rectangle, and let be a circular arc passing through
the points and .
Let be the circle tangent to the lines and and to the circle , and lying
completely inside the rectangle .
Similarly let be the circle tangent to the lines and and to the circle
, and lying completely inside the rectangle .
Denote by and the radii of the circles and , respectively, and by the
inradius of triangle .
(a) Prove that .
(b) Prove that one of the two common internal tangents of the two circles and
is parallel to the line and has the length .
TST 2001

Problem 14 A strictly increasing sequence has the property that


for all . Suppose is the least positive integer
for which there exist positive integers such that . Prove that
and .
TST 2001

Problem 15 Let be a polynomial of degree with real coefficients and let


. Prove that

TST 2001

105
Problem 16 Let and be three points on a line with between and
. Let be semicircles, all on the same side of and with
as diameters, respectively. Let be the line perpendicular to through . Let
be the circle which is tangent to the line , tangent to internally, and tangent to
externally. Let be the point of contact of and . The diameter of
through meets in . Show that .
TST 2002

Problem 17 Show that there is a set of consecutive positive integers containing


exactly primes. (You may use the fact that there are primes less than 1000)
TST 2002

Problem 18 Let . How many quadratics are there of the


form , with equal roots, and such that are distinct elements
of ?
TST 2002

Problem 19 Let be the circumcenter and the orthocenter of an acute triangle


. Show that there exist points , , and on sides , , and
respectively such that and the lines
, , and are concurrent.
TST 2002

Problem 20 Let be positive reals such that . Prove that

TST 2002

106
Problem 21 Determine the number of -tuples of integers such
that for each and for .
TST 2002

Problem 22 Given two distinct circles touching each other internally, show how to
construct a triangle with the inner circle as its incircle and the outer circle as its nine
point circle.
TST 2002

Problem 23 Let

be the sum of positive divisors of an integer .


Show that for positive integers and with

TST 2002

Problem 24 Find all positive integers such that is a power of .


TST 2002

107
Problem 25 On each day of their tour of the West Indies, Sourav and Srinath have
either an apple or an orange for breakfast. Sourav has oranges for the first days,
apples for the next days, followed by oranges for the next days, and so on.
Srinath has oranges for the first days, apples for the next days, followed by
oranges for the next days, and so on.
If , and if the tour lasted for days, on how many days did they
eat the same kind of fruit?
TST 2002

Problem 26 Let denote the set of all ordered triples of nonnegative


integers. Find all functions satisfying

for all nonnegative integers , , .


TST 2002

Problem 27 Let be a triangle and an exterior point in the plane of the


triangle. Suppose the lines , , meet the sides , , (or
extensions thereof) in , , , respectively. Suppose further that the areas of
triangles , , are all equal. Prove that each of these areas is equal
to the area of triangle itself.
TST 2002

108
Problem 28 Let be integers with . A set of non-negative
integers is olympic if and if . Show that the set
of all non-negative integers is the union of pairwise disjoint olympic sets.
TST 2002

Problem 29 Let and be two triangles such that is the midpoint of


and is the midpoint of . bisects and bisects
Prove that .
TST 2002

Problem 30 Let be an odd prime and let be an integer not divisible by . Show
that there are triples of integers with and such
that
TST 2002

Problem 31 Let be arbitrary real numbers. Prove the inequality

TST 2002

Problem 32 Is it possible to find positive integers not exceeding , such


that all pairwise sums of them are different?
TST 2002

109
Problem 33 Let be a positive integer and let where
is the square root of , and and are polynomials with real coefficients. Show
that for any real number the equation has only real roots.
TST 2002

Problem 34 Consider the square grid with and at its diagonal


ends. Paths from to are composed of moves one unit to the right or one unit
up. Let (n-th catalan number) be the number of paths from to which stay
on or below the diagonal . Show that the number of paths from to which
cross from below at most twice is equal to
TST 2002

Problem 35 Let be an acute triangle. Let , and be isosceles


triangles exterior to , with , and , such
that
Let be
the intersection of lines and , let be the intersection of and
, and let be the intersection of and . Find, with proof, the value of the
sum

TST 2002

Problem 36 Let be positive real numbers. Prove that

TST 2002

110
Problem 37 Given a prime , show that there exists a positive integer such that the
decimal representation of has a block of consecutive zeros.
TST 2002

Problem 38 Let be the midpoints of the sides , respectively,


of an acute non-isosceles triangle , and let be the feet of the
altitudes through the vertices on these sides respectively. Consider the arc
of the nine point circle of triangle lying outside the triangle. Let the
point of trisection of this arc closer to be . Define analogously the points
(on arc ) and (on arc ). Show that triangle is equilateral.
TST 2003

Problem 39 Find all triples of positive integers such that


(i) ;
(ii) ; and
(iii) is divisible by each of the numbers .
TST 2003

Problem 40 Find all functions such that for all reals and ,

TST 2003

111
Problem 41 There are four lines in the plane, no three concurrent, no two parallel, and
no three forming an equilateral triangle. If one of them is parallel to the Euler line of
the triangle formed by the other three lines, prove that a similar statement holds for
each of the other lines.
TST 2003

Problem 42 On the real number line, paint red all points that correspond to integers of
the form , where and are positive integers. Paint the remaining
integer point blue. Find a point on the line such that, for every integer point
, the reflection of with respect to is an integer point of a different color than
.
TST 2003

Problem 43 A zig-zag in the plane consists of two parallel half-lines connected by a


line segment. Find , the maximum number of regions into which zig-zags can
divide the plane. For example, (see the diagram). Of these
regions how many are bounded? [The zig-zags can be as narrow as you please.]
Express your answers as polynomials in of degree not exceeding .

TST 2003

112
Problem 44 is a polynomial with integer coefficients and for every natural we
have . is a sequence that: for every one of
is divisible by Prove that
TST 2003

Problem 45 Let be a triangle, and let denoted its inradius and the
exradii opposite the vertices , respectively. Suppose
. Prove that
(a) triangle is acute,
(b) .
TST 2003

Problem 46 Let be a positive integer and a partition of


such that . Prove that there exist , ,
such that one of is the sum of the other two.
TST 2003

Problem 47 Let be a positive integer greater than , and let be a prime such that
divides and divides . Prove that is a square.
TST 2003

Problem 48 Let be a cyclic quadrilateral. Let , , be the feet of the


perpendiculars from to the lines , , , respectively. Show that
if and only if the bisectors of and are concurrent
with .
TST 2004

113
Problem 49 Prove that for every positive integer there exists an -digit number
divisible by all of whose digits are odd.
TST 2004

Problem 50 For positive reals find the minimum value of

TST 2004

Problem 51 Given a permutation of , an ordered


pair is called an inversion of if and . Let
denote the no. of inversions of the permutation . Find the average of as
varies over all permutations.
TST 2004

Problem 52 Prove that in any triangle ,

TST 2004

Problem 53 Find all triples of positive integers such that

TST 2004

114
Problem 54 Suppose the polynomial has only real zeros
and let . Assume that has no real roots.
Prove that
TST 2004

Problem 55 Let be a bijection of the set of all natural numbers onto itself. Prove that
there exists positive integers such that

TST 2004

Problem 56 A set of 4 points in the plane is said to be Athenian set if


there is a point of the plane satisfying

(*) does not lie on any of the lines for ;


(**) the line joining to the midpoint of the line is perpendicular to the line
joining to the midpoint of , being distinct.

(a) Find all Athenian sets in the plane.


(b) For a given Athenian set, find the set of all points in the plane satisfying (*)
and (**)
TST 2004

Problem 57 Determine all integers such that is divisible by for


some
TST 2004

115
Problem 58 The game of is played on an infinite board of lattice points
. Initially there is a at . A move consists of removing a from
point and placing a at each of the points and
provided both are vacant. Show that at any stage of the game there is a at
some lattice point with
TST 2004

Problem 59 Let be a triangle and let be a point in its interior. Denote by ,


, the feet of the perpendiculars from to the lines , ,
, respectively. Suppose that Denote
by , , the excenters of the triangle . Prove that is the circumcenter
of the triangle .
TST 2004

Problem 60 Show that the only solutions of the equation , in positive


integers and prime are
(i)
(ii) and is a prime of the form ,
TST 2004

Problem 61 Determine all functions such that

for all reals where is a given


constant.
TST 2004

116
Problem 62 Let be a triangle and its incenter. Let and be the inradii of
triangles and respectively.

(a) Show that there exists a function such that where


and

(b) Prove that


TST 2004

Problem 63 Define a function by the following rule:


(a) is nondecreasing
(b) for each , i sthe number of times appears in the range of ,

Prove that and for all


TST 2004

Problem 64 Two runners start running along a circular track of unit length from the
same starting point and in the same sense, with constant speeds and
respectively, where and are two distinct relatively prime natural numbers.
They continue running till they simultaneously reach the starting point. Prove that

(a) at any given time , at least one of the runners is at a distance not more than

units from the starting point.

(b) there is a time such that both the runners are at least units away from
the starting point. (All distances are measured along the track). is the greatest
integer function.
TST 2004

117
Problem 65 Let be real numbers such that . Prove

that
TST 2004

Problem 66 Find all primes with the following property: for any prime

, the number is squarefree (i.e. is not divisible by the square of a prime).


TST 2004

Problem 67 Every point with integer coordinates in the plane is the center of a disk
with radius .

(1) Prove that there exists an equilateral triangle whose vertices lie in different
discs.

(2) Prove that every equilateral triangle with vertices in different discs has
side-length greater than .
TST 2004

118
Problem 68 Let be an acute-angled triangle and be a circle with as
diameter intersecting and at and respectively.
Tangents are drawn at and to intersect at . Show that the ratio of the
circumcentre of triangle to that if is a rational number.
TST 2004

Problem 69 Let and be


two real polynomials. Suppose that there exists an interval of length greater
than SUCH THAT BOTH AND ARE nEGATIVE FOR and
both are positive for and . Show that there is a real such that

TST 2004

Problem 70 An integer is said to be good if is not the square of an integer.


Determine all integers with the following property: can be represented, in
infinitely many ways, as a sum of three distinct good integers whose product is the
square of an odd integer.
TST 2004

Problem 71 Let be a triangle with all angles . Let be the Fermat


point of triangle , that is, the interior point of such that
. For each one of the three triangles ,
and , draw its Euler line - that is, the line connecting its circumcenter
and its centroid.
Prove that these three Euler lines pass through one common point.
Remark. The Fermat point is also known as the first Fermat point or the first
Torricelli point of triangle .
TST 2005

119
Problem 72 Prove that one can find a such that , there exist three
positive integers , , such that

(i) ;

(ii) is the cube of an integer.


TST 2005

Problem 73 If , , are three positive real numbers such that

, prove that
TST 2005

Problem 74 Consider a -sided polygon inscribed in a circle ( ). Partition the


polygon into triangles using non-intersecting diagonals. Prove that,
irrespective of the triangulation, the sum of the in-radii of the triangles is a constant.
TST 2005

Problem 75 Let denote the number of positive divisors of the positive integer
. Prove that there exist infinitely many positive integers such that the equation
does not have a positive integer solution .
TST 2005

120
Problem 76 There are students at a university. Some students join together to
form several clubs (a student may belong to different clubs). Some clubs join
together to form several societies (a club may belong to different societies). There
are a total of societies. Suppose that the following conditions hold:

i.) Each pair of students are in exactly one club.

ii.) For each student and each society, the student is in exactly one club of the society.

iii.) Each club has an odd number of students. In addition, a club with
students ( is a positive integer) is
in exactly societies.

Find all possible values of .


TST 2005

Problem 77 Let be two rational numbers. Let be a set of positive real


numbers with the properties:

(i) and ;

(ii) if and , then .

Let denote the set of all irrational numbers in . prove that every such
that , contains an element with property
TST 2005

121
Problem 78 Find all functions satisfying

for any two positive integers and .

Remark. The abbreviation stands for the set of all positive integers:
.
By , we mean (and not ).
TST 2005

Problem 79 A merida path of order is a lattice path in the first quadrant of


- plane joining to using three kinds of steps ,
and , i.e. joins to etc... An ascent
in a merida path is a maximal string of consecutive steps of the form . If
denotes the number of merida paths of order with exactly ascents, compute
and .
TST 2005

Problem 80 Let be a convex quadrilateral. The lines parallel to and


through the orthocentre of intersect and Respectively at
and . prove that the perpendicular through to the line passes through the
orthocentre of triangle
TST 2005

122
Problem 81 Given real numbers s.t. and , prove
that there exist integers and s.t.

TST 2005

Problem 82 Consider a matrix of size whose entries are real numbers of


absolute value not exceeding . The sum of all entries of the matrix is . Let be an
even positive integer. Determine the least number such that every such matrix
necessarily has a row or a column with the sum of its entries not exceeding in
absolute value.
TST 2005

Problem 83 For a given triangle ABC, let X be a variable point on the line BC such that
the point C lies between the points B and X. Prove that the radical axis of the
incircles of the triangles ABX and ACX passes through a point independent of X.
TST 2005

Problem 84 Determine all positive integers , such that


TST 2005

123
Problem 85 For real numbers not all equal to , define a real function
. Suppose for some real
. prove that there exist a real number s.t.
TST 2005

Problem 86 Let be a positive integer divisible by . Find the number of


permutations of which satisfy the condition
for all .
TST 2006

Problem 87 Let be a parallelogram. A variable line through the vertex


intersects the rays and at the points and , respectively. Let and
be the -excenters of the triangles and . Show that the angle
is independent of the line .
TST 2006

Problem 88 There are markers, each with one side white and the other side black.
In the beginning, these markers are aligned in a row so that their white sides are
all up. In each step, if possible, we choose a marker whose white side is up (but not
one of the outermost markers), remove it, and reverse the closest marker to the left
of it and also reverse the closest marker to the right of it. Prove that, by a finite
sequence of such steps, one can achieve a state with only two markers remaining if
and only if is not divisible by .
TST 2006

124
Problem 89 Let be a triangle and let be a point in the plane of that is
inside the region of the angle but outside triangle .

(a) Prove that any two of the following statements imply the third.

(i) the circumcentre of triangle lies on the ray .

(ii) the circumcentre of triangle lies on the ray .

(iii) the circumcentre of triangle lies on the ray .

(b) Prove that if the conditions in (a) hold, then the circumcenter of triangles
and lie on the circumcircle of triangle
TST 2006

Problem 90 Let be a prime number and let be a finite set containing at least
elements. A collection of pairwise mutually disjoint -element subsets of is
called a -family. (In particular, the empty collection is a -family.) Let
(respectively, ) denote the number of -families having an even (respectively,
odd) number of -element subsets of Prove that and differ by a multiple of
.
TST 2006

Problem 91 Let be an equilateral triangle, and let and be points on


and respectively. Let and
. Prove that if , then the union of the triangular regions
covers the triangle .
TST 2006

125
Problem 92 Let be a triangle with inradius , circumradius , and with sides
. Prove that

TST 2006

Problem 93 the positive divisors of a positive integer are ordered

Suppose . Find all possible values of .


TST 2006

Problem 94 Let be arithmetic progressions of integers, each of


terms, such that any two of these arithmetic progressions have at least two common
elements. Suppose of these arithmetic progressions have common difference
and the remaining arithmetic progressions have common difference where
. Prove that

TST 2006

Problem 95 Find all triples such that are integers in the set
satisfying and .
TST 2006

126
Problem 96 Let be a real number for each and each and let
be an integer such that

Let and be positive integers such that . Prove that there exist
integers not all zero, such that

TST 2006

Problem 97 Let be subsets of a finite set such that for


each . For a subset of let . Suppose
for each subset of at least one of the following conditions holds

, , .
Prove that .
TST 2006

Problem 98 Show that in a non-equilateral triangle, the following statements are


equivalent:
The angles of the triangle are in arithmetic progression.
The common tangent to the Nine-point circle and the Incircle is parallel to the
Euler Line.
TST 2007

127
Problem 99 Find all integer solutions of the equation
TST 2007

Problem 100 Let be the set of all bijective functions from the set
to itself. For each define

Determine
(Here for all )
TST 2007

Problem 101 Let be a trapezoid with parallel sides . Points


and lie on the line segments and , respectively, so that
. Suppose that there are points and on the line
segment satisfying Prove
that the points , , and are concyclic.|
TST 2007

Problem 102 Let be non-negative real numbers such that


and Show that

TST 2007

128
Problem 103 Given a finite string of symbols and , we denote as the
number of s in minus the number of s (For example,
). We call a string balanced if every substring of
(consecutive symbols) has the property (Thus
is not balanced, since it contains the substring whose
value is Find, with proof, the number of balanced strings of length
TST 2007

Problem 104 A sequence of real numbers is defined by the formula


here is an arbitrary real number,
denotes the greatest integer not exceeding , and . Prove that
for sufficiently large.
TST 2007

Problem 105 Let be a finite set of points in the plane such that no three of them are
on a line. For each convex polygon whose vertices are in , let be the
number of vertices of , and let be the number of points of which are
outside . A line segment, a point, and the empty set are considered as convex
polygons of , , and vertices respectively. Prove that for every real number

where the sum is taken over all convex polygons with


vertices in .
TST 2007

129
Problem 106 Circles and with centers and are externally tangent at
point and internally tangent to a circle at points and respectively. Line
is the common tangent of and at . Let be the diameter of
perpendicular to , so that are on the same side of . Prove that lines
, , and are concurrent.
TST 2007

Problem 107 Find all integer solutions of the equation where


is a prime such that
TST 2007

Problem 108 Find all function(s) satisfying the equation

For all
TST 2007

Problem 109 Let be a triangle with .Prove that if is point of


contact of Incircle And Nine-Point Circle, Then ,
being inradius.
TST 2009

130
Problem 110 Let us consider a simple graph with vertex set . All ordered pair
of integers with , are elements of V.
is connected to by an edge and to by another edge
for all integers k.
Prove that for all , there exists a path fromm to .
TST 2009

Problem 111 Let be two distinct odd natural numbers.Define a Sequence


like following:

.
Prove that there exists a natural number such that .
TST 2009

Problem 112 Let be circumcircle of .Let be radius of circle touching


& internally.Define similarly.

Prove That .
TST 2009

131
Problem 113 Let and be two monic polynomials of degree= having
complex coefficients.
We know that there exist complex numbers , such that

.
Prove that there exists such that
.
TST 2009

Problem 114 Prove The Following identity:

.
The Second term on the left hand side is to be regarded as zero for j=0.
TST 2009

Problem 115 Let be any point in the interior of a .Prove That

.
TST 2009

Problem 116 Let be a natural number which divides .Prove That


.
TST 2009

132
Problem 117 Let

and be two polynomials with real


coefficients.
Let g(x) have as two of its roots. Prove That has a positive root less
than .
TST 2009

Problem 118 For a certain triangle all of its altitudes are integers whose sum is less
than 20. If its Inradius is also an integer Find all possible values of area of the
triangle.
TST 2009

Problem 119 Find all integers with the following property:

There exists three distinct primes such that


whenever are distinct positive integers with the property that
at least one of divides ,
one of divides all of these differences.
TST 2009

133
Problem 120 Let be a simple graph with vertex set .
and are connected by an edge for . Let be a subset of and
be the induced subgraph associated with . Let be number of
components of having an odd number of vertices.
Let
for .

Prove That .
TST 2009

Problem 121 Let be a triangle in which . Let be the midpoint


of , be the altitude from on , and be the altitude from onto
. Suppose that produced meets (extended) at . If is the
orthocenter of , prove that is perpendicular to .
TST 2010

Problem 122 Two polynomials and


have real coefficients, and is an interval on the real line of
length greater than . Suppose and take negative values on , and they
take non-negative values outside . Prove that there exists a real number such
that .
TST 2010

134
Problem 123 For any integer , let be the maximum number of triples
consisting of non-negative integers
(not necessarily distinct) such that the following two conditions are satisfied:

(a) for all ;


(b) , then , and .

Determine for all .


TST 2010

Problem 124 Let be positive real numbers such that


. Prove that

TST 2010

Problem 125 Given an integer , show that there exist an integer an and
distinct positive integers , all greater than , such that the sums

and are both -the powers of some integers.


(Here denotes the number of positive integers less than and relatively
prime to .)
TST 2010

135
Problem 126 Let be a given integer. Show that the number of strings of length
consisting of s and s such that there are equal number of and blocks in
each string is equal to

TST 2010

Problem 127 Let be a cyclic quadrilateral and let be the point of


intersection of its diagonals and . Suppose and meet in . Let
the midpoints of and be and respectively. If is the circumcircle of
triangle , prove that is tangent to .
TST 2010

Problem 128 Call a positive integer good if either or can be written as


product of Even number of prime numbers, not necessarily distinct.
Let where are positive integers.

(a) Show that there exist distinct positive integers such that
are all good numbers.
(b) Suppose are such that is a good number for all positive integers
. Prove that .
TST 2010

136
Problem 129 Let be a array of positive real numbers such that
the sum of numbers in row as well as in each column is .
Show that there exists and such that

TST 2010

Problem 130 Let be a triangle. Let be the brocard point. Prove that

TST 2010

Problem 131 Find all functions such that


for all reals
TST 2010

Problem 132 Prove that there are infinitely many positive integers for which there
exists consecutive odd positive integers such that
and are both perfect squares. If are two positive
integers satisfying this condition, then we have
TST 2010

137
Problem 133 Let be a triangle each of whose angles is greater than
. Suppose a circle centered with cuts segments in in and
in such that they are on a circle in counterclockwise direction in that
order.Suppose further are equilateral. Prove that:

The radius of the circle is where is an area.

TST 2011

Problem 134 Let the real numbers satisfy the relations


and Prove that

TST 2011

Problem 135 A set of distinct integer weights is said to


be balanced if after removing any one of weights, the remaining weights
can be split into two subcollections (not necessarily with equal size)with equal sum.

Prove that if there exist balanced sets of sizes then also a balanced set of
size .
Prove that for all odd there exist a balanced set of size .
TST 2011

138
Problem 136 Find all positive integer satisfying the conditions

is a perfect square.
TST 2011

Problem 137 Suppose are non-integral real numbers for such that
is an integer for all integers . Prove that none of
is rational.
TST 2011

Problem 138 Let be a non-empty finite subset of positive integers . A subset


of is called good if for every integer there exists an in such that
. Let

Prove that :
If is not good then the number of pairs in is even.
the number of good subsets of is odd.
TST 2011

139
Problem 139 Let be a convex pentagon such that
and Let be the midpoint of and
let be the circumcenter of triangle Given that prove
that
TST 2011

Problem 140 Prove that for no integer is a perfect square.


TST 2011

Problem 141 Consider a square grid which is divided into unit


squares(think of a chess-board). The set of all unit squares intersecting the main
diagonal of the square or lying under it is called an -staircase. Find the number of
ways in which an -stair case can be partitioned into several rectangles, with sides
along the grid lines, having mutually distinct areas.
TST 2011

Problem 142 Let be an acute-angled triangle. Let be internal


bisectors with on respectively. Prove that

TST 2011

Problem 143 Find all pairs of nonnegative integers for which

TST 2011

140
Problem 144 Let and be two infinite sequences of integers
such that

for all integers . Prove that there exists a positive integer such that

TST 2011

Problem 145 Let be an isosceles triangle with . Let be a point


on the segment such that . Let be a point on the segment
such that . Prove that .
TST 2012

Problem 146 Let and be real numbers. Prove that the equation
has real roots.
TST 2012

Problem 147 How many -tuples of natural numbers are there for
which and are
simultaneously true?
TST 2012

141
Problem 148 Let be a trapezium with . Let be a point on
such that is between and ; and let be the midpoints of
respectively. Let intersect in and intersect in . Prove
that .
TST 2012

Problem 149 Let be integers where is a prime. Prove that the


following statements are equivalent:

TST 2012

Problem 150 Let be a function such that


for all . Prove that satisfies
for all .
TST 2012

Problem 151 The circumcentre of the cyclic quadrilateral is . The second


intersection point of the circles and , other than , is , which lies in
the interior of the triangle . Choose a point on the extension of
beyond , and a point on the extension of beyond . Prove that
if and only if .
TST 2012

142
Problem 152 Let be a polynomial with
complex coefficients such that and . Prove that

TST 2012

Problem 153 Determine the greatest positive integer that satisfies the following
property: The set of positive integers can be partitioned into subsets
such that for all integers and all there
exist two distinct elements of whose sum is
TST 2012

Problem 154 Determine all sequences of positive integers, such


that for every positive integer there exists an integer with

TST 2012

Problem 155 Show that there exist infinitely many pairs of positive integers
with the property that divides , divides , and

TST 2012

143
Problem 156 Suppose that students are standing in a circle. Prove that there
exists an integer with such that in this circle there exists a
contiguous group of students, for which the first half contains the same number
of girls as the second half.
TST 2012

Problem 157 Let be a triangle with and let be the midpoint of


. The angle bisector of intersects the circle through and at the
point inside the triangle . The line intersects the circle through
and in two points and . The lines and meet at a point , and the
lines and meet at a point . Show that is the incentre of triangle
.
TST 2012

Problem 158 Let be a nonempty set of primes satisfying the property that for each

proper subset of , all the prime factors of the number are also
in . Determine all possible such sets .
TST 2012

Problem 159 In a array we have positive reals s.t. the sum of the numbers in
each of the columns is . Show that we can select a number in each column s.t. the

sum of the selected numbers in each row is at most .


TST 2012

144
Problem 160 A quadrilateral without parallel sides is circumscribed around
a circle with center . Prove that is a point of intersection of middle lines of
quadrilateral (i.e. barycentre of points ) iff
.
TST 2012

Problem 161 Find the least positive integer that cannot be represented as
for some positive integers .
TST 2012

Problem 162 Let denote the set of all positive real numbers. Find all functions
satisfying

for all .
TST 2012

Problem 163 For a prime , a natural number and an integer , we let


denote the exponent of in the prime factorisation of . For example,
and . Find all pairs such that
.
TST 2013

145
Problem 164 Let by a cyclic quadrilateral with circumcenter . Let be
the point of intersection of the diagonals and , and the
circumcenters of triangles , , respectively. Prove that

TST 2013

Problem 165 We define an operation on the set by

For two natural numbers and , which are written in base as


and (possibly with leading 0's), we define
where written in base is with , for
. For example, we have since and .

For a natural number , let , where denotes the largest


integer less than or equal to . Prove that is a bijection on the set of natural
numbers.
TST 2013

Problem 166 Let be positive real numbers such that . If is a


positive integer then prove that

TST 2013

146
Problem 167 In a triangle with , is a point on the segment
such that the inradii of triangles and are equal. If then
prove that .
TST 2013

Problem 168 A marker is placed at the origin of an integer lattice. Calvin and Hobbes
play the folLowing game. Calvin starts the game and each of them takes turns
alternatively. At each turn, one can choose two (not necessarily distinct) integers
, neither of which was chosen earlier by any player and move the marker by
units in the horizontal direction and units in the vertical direction. Hobbes wins if
the marker is back at the origin any time after the first turn. Prove or disprove that
Calvin can prevent Hobbes from winning.
Note: A move in the horizontal direction by a positive quantity will be towards the
right, and by a negative quantity will be towards the left (and similar directions in
the vertical case as well).
TST 2013

Problem 169 Let be an integer. There are beads numbered . Two


necklaces made out of some of these beads are considered the same if we can get
one by rotating the other (with no flipping allowed). For example, with , the
necklace with four beads in the clockwise order is same as the one with
in the clockwise order, but is different from the one with in the
clockwise order.
We denote by (respectively ) the number of ways in which we can use
all the beads to make an even number (resp. an odd number) of necklaces each of
length at least . Prove that divides .
TST 2013

147
Problem 170 In a triangle , with , let and denote its
circumcenter and orthocenter, respectively. Let be the reflection of with
respect to . Prove that and are collinear if and only if .
TST 2013

Problem 171 For a positive integer , a cubic polynomial is said to be -good if


there exist distinct integers such that all the roots of the
polynomial are integers for . Given a positive integer
prove that there exists an -good cubic polynomial.
TST 2013

Problem 172 Find all functions from the set of real numbers to itself satisfying
for all real numbers .
TST 2013

Problem 173 An integer is called friendly if the equation


has a solution over the positive integers.
a) Prove that there are at least friendly integers in the set .
b) Decide whether is friendly.
TST 2013

148
Problem 174 Players and play a game with coins and boxes
arranged around a circle. Initially distributes the coins among the boxes so that
there is at least coin in each box. Then the two of them make moves in the order
by the following rules:
(a) On every move of his passes coin from every box to an adjacent box.
(b) On every move of hers chooses several coins that were not involved in 's
previous move and are in different boxes. She passes every coin to an adjacent box.
Player 's goal is to ensure at least coin in each box after every move of hers,
regardless of how plays and how many moves are made. Find the least that
enables her to succeed.
TST 2013

Problem 175 For a positive integer , a sum-friendly odd partition of is a


sequence of odd positive integers with and
such that for all positive integers , can
be uniquely written as a subsum . (Two subsums
and with and
are considered the same if and for
.) For example, is a sum-friendly odd partition of . Find the number of
sum-friendly odd partitions of .
TST 2013

Problem 176 In a triangle , let denote its incenter. Points are


chosen on the segments , respectively, such that
and . The circumcircles of triangles
intersect lines , respectively, at points (different from
), respectively. Prove that are concyclic.
TST 2013

149
Problem 177 Let be an integer and the set of all positive integers that are
greater than or equal to . Let be a nonempty subset of such that the
following two conditions hold:
I. if with , then
II. if with , then .
Prove that .
TST 2013

Problem 178 A positive integer is called a double number if it has an even number
of digits (in base 10) and its base 10 representation has the form
with for , and . For
example, is a double number. Determine whether or not there are infinitely
many double numbers such that is a square and is not a power of
.
TST 2013

Problem 179 Let be an integer and a sequence of


polynomials with integer coefficients. One is allowed to make moves as
follows: in the -th move one chooses an element of the sequence with
degree of at least and replaces it with . The process
stops when all the elements of the sequence are of degree . If
, determine whether or not it is possible
to make appropriate moves such that the process stops with a sequence of
identical polynomials of degree 1.
TST 2013

150
Problem 180 In a triangle , with , is a point on the line
such that is perpendicular to . A circle passing through and touching
the line at a point intersects the line for the second time at
. Let be a point on the line different from such that . Let
be the point of intersection of the lines and . Prove that the points
are concyclic if and only if is perpendicular to .
TST 2013

Problem 181 Let be an odd prime and an odd natural number.Show that
does not divide
TST 2014

Problem 182 Let be positive real numbers.Prove that


.
TST 2014

Problem 183 In a triangle , points and are on and respectively


such that , is not perpendicular to and is not
perpendicular to .Let be the circle with as center and as its
radius.Find the angles of triangle given that the orthocenter of triangles
and lie on .
TST 2014

151
Problem 184 Let and be rational numbers, such that . Prove
that is the square of a rational number.
TST 2014

Problem 185 Let be a natural number.A triangulation of a convex n-gon is a division


of the polygon into triangles by drawing diagonals no two of which
intersect at an interior point of the polygon.Let denote the number of
triangulations of a regular n-gon such that each of the triangles formed is
isosceles.Determine in terms of .
TST 2014

Problem 186 For integers we define if and


if .
Given a natural number show that there exist natural numbers with
such that ,where
, being composed with itself times.
TST 2014

Problem 187 Find all polynomials with integer coefficients such that and
are co-prime for all natural numbers .
TST 2014

152
Problem 188 Let be a positive integer. Find the smallest integer with the
following property; Given any real numbers such that
and for , it is possible to
partition these numbers into groups (some of which may be empty) such that the
sum of the numbers in each group is at most .
TST 2014

Problem 189 Starting with the triple , define a


sequence of triples by

for .Show that each of the sequences converges


to a limit and finds these limits.
TST 2014

Problem 190 In a triangle , let be its incenter; the point at which the
incircle touches the line ; the midpoint of and the orthocenter of
triangle . Prove that the line is perpendicular to the line .
TST 2014

153
Problem 191 For let be non-zero real numbers, and let
.Suppose that the following statements hold:

satisfy triangle inequality

also satisfy triangle inequality.

Prove that exactly one of is negative.


TST 2014

Problem 192 Let be a positive integer, and let be an infinite sequence of


real numbers. Assume that for all nonnegative integers and there exists a
positive integer such that

Prove that the sequence is periodic, i.e. there exists some such that
for all .
TST 2014

Problem 193 In a triangle , with and , is a point


on line different from . Suppose that the circumcenter and orthocenter of
triangles and lie on a circle. Prove that .
TST 2014

154
Problem 194 Determine whether there exists an infinite sequence of nonzero digits
and a positive integer such that for every integer , the
number is a perfect square.
TST 2014

Problem 195 In how many ways rooks can be placed on a by chess board such
that every row and every column has at least one rook?
(Any number of rooks are available,each square can have at most one rook and there
is no relation of attacking between them)
TST 2014

Problem 196 Prove that in any set of distinct real numbers there exist two

pairs and with or , such that


TST 2014

Problem 197 Find all positive integers and such that .


TST 2014

Problem 198 Let be a triangle with . Let and be two different


points on line such that and is located
between and . Suppose that there exists an interior point of segment
for which . Let the ray intersect the circle at . Prove
that .
TST 2014

155
Problem 199 Find all positive integers such that and are also
integers.
TST 2015

Problem 200 A -digit number is called a number if its digits belong to the set
and the difference of every pair of consecutive digits is .
a) Find the total number of cute numbers.
b) Prove that the sum of all cute numbers is divisible by .
TST 2015

Problem 201 Prove that for any triangle , the inequality

holds.
TST 2015

Problem 202 Let be a triangle in which . Let be its


orthocentre and its circumcentre. Let and be respectively the midpoints of
the arc not containing and arc not containing . Let and be
respectively the reflections of in and in . Prove that lie
on a circle if and only if are collinear.
TST 2015

156
Problem 203 For a composite number , let denote its largest proper divisor.
Show that there are infinitely many for which is a perfect square.
TST 2015

Problem 204 Every cell of a board is coloured either by red or blue. Find the
number of all colorings in which there are no squares in which all cells are
red.
TST 2015

Problem 205 Let be a convex quadrilateral and let the diagonals and
intersect at . Let be respectively the incentre of triangles
. Let be respectively the excentres of
triangles opposite . Show that lie on a
circle if and only if lie on a circle.
TST 2015

Problem 206 Let and be two polynomials with integer coefficients such that the
leading coefficients of both the polynomials are positive. Suppose is odd and
the sets and are the same. Prove that there exists
an integer such that .
TST 2015

157
Problem 207 Let points be given inside a rectangle such that no two of them lie
on a line parallel to one of the sides of . The rectangle is to be dissected into
smaller rectangles with sides parallel to the sides of in such a way that none of
these rectangles contains any of the given points in its interior. Prove that we have to
dissect into at least smaller rectangles.
TST 2015

Problem 208 Let be an integer, and let be the set


Determine the largest positive integer that
cannot be written as the sum of one or more (not necessarily distinct) elements of
.
TST 2015

Problem 209 Find all functions from such that


, for all .
TST 2015

Problem 210 Let be a simple graph on the infinite vertex set


. Suppose every subgraph of on a finite vertex subset is
-colorable, Prove that itself is -colorable.
TST 2015

158
Problem 211 In a triangle , a point is on the segment , Let and be
the incentres of triangles and respectively. The lines and
intersect the circumcircle of triangle at and , respectively.
Let be the point of intersection of lines and . Suppose is also the
reflection of in where is the incentre of triangle . Prove that
.
TST 2015

Problem 212 Find all triples consisting of a prime number and two
positive integers and such that and are both powers of .
TST 2015

Problem 213 There are lamps, each with two states: or . For each
non-empty subset of the set of these lamps, there is a which operates
on the lamps in ; that is, upon this button each of the lamps in
changes its state(on to off and off to on). The buttons are identical and it is not
known which button corresponds to which subset of lamps. Suppose all the lamps
are off initially. Show that one can always switch all the lamps on by performing at
most operations.
TST 2015

Problem 214 Consider a fixed circle with three fixed points and on it.
Also, let us fix a real number . For a variable point on
, let be the point on the segment such that . Let be the
second point of intersection of the circumcircles of the triangles and
. Prove that as varies, the point lies on a fixed circle.
TST 2015

159
Problem 215 Let be a finite set of pairs of real numbers such that for any pairs
in we have . Let be a pair of real numbers(not
necessarily from ). We define for all as follows: for
all , if we let ; otherwise we choose a pair
in for which and set . Show that
there exists an integer such that .
TST 2015

Problem 216 Let be a given integer. Prove that infinitely many terms of the

sequence , defined by are odd. (For a real number ,


denotes the largest integer not exceeding .)
TST 2015

Problem 217 An acute-angled is inscribed into a circle . Let


be the centroid of , and let be the altitude of this triangle. A ray
meets at . Prove that the circumcircle of the triangle is tangent
to
TST 2016

Problem 218 Given that is a natural number such that the leftmost digits in the
decimal representations of and are the same, find all possible values of the
leftmost digit.
TST 2016

160
Problem 219 Let a,b,c,d be real numbers satisfying and
. Prove that

TST 2016

Problem 220 We say a natural number is perfect if the sum of all the positive
divisors of is equal to . For example, is perfect since its positive divisors
add up to . Show that an odd perfect number has at least
distinct prime divisors.

Note: It is still not known whether odd perfect numbers exist. So assume such a
number is there and prove the result.
TST 2016

Problem 221 Find all functions such that


for all reals .
TST 2016

161
Problem 222 An equilateral triangle with side length is divided into congruent
triangular cells as shown in the figure below. Initially all the cells contain
. A move consists of selecting two adjacent cells (i.e., cells sharing a common
boundary) and either increasing or decreasing the numbers in both the cells by
simultaneously. Determine all positive integers such that after performing several
such moves one can obtain consecutive numbers in
some order.

TST 2016

Problem 223 Let be an acute triangle with orthocenter . Let be the point
such that the quadrilateral is a parallelogram. Let be the point on the
line such that bisects . Suppose that the line intersects the
circumcircle of the triangle at and . Prove that .
TST 2016

Problem 224 Suppose that a sequence of positive real numbers satisfies

for every positive integer . Prove that


for every .
TST 2016

162
Problem 225 Let be a natural number. A sequence of numbers
is called if each is an element of the set and the
ordered pairs are all different for (here we consider
the subscripts modulo ). Two good sequences and
are called if there exists an integer such that
for all (again taking subscripts modulo ). Suppose that there
exists a non-trivial permutation (i.e., a permutation which is different from the
identity permutation) of and an good sequence
which is similar to . Show that
.
TST 2016

Problem 226 Suppose are two positive rational numbers. Assume for some
positive integers , it is known that is a rational number. Prove that
each of and is a rational number.
TST 2016

Problem 227 Let and be positive integers such that . Define

for . Prove that if all the numbers


are integers, then is divisible by an odd prime.
TST 2016

163
Problem 228 For a finite set of positive integers, a partition of into two disjoint
nonempty subsets and is if the least common multiple of the elements
in is equal to the greatest common divisor of the elements in . Determine the
minimum value of such that there exists a set of positive integers with exactly
good partitions.
TST 2016

Problem 229 Let be a natural number. We define sequences and of


integers as follows. We let and . For , we let

Given that
for some natural number , prove that is a power of two.
TST 2016

Problem 230 Let be an acute triangle and let be the midpoint of .A


circle passing through and meets the sides and at points and
respectively. Let be the point such that is a parallelogram. Suppose

that lies on the circumcircle of . Determine all possible values of .


TST 2016

Problem 231 Let be an odd natural number. We consider an grid which is


made up of unit squares and edges. We color each of these edges
either or . If there are at most edges, then show that there exists a
unit square at least three of whose edges are .
TST 2016

164
Problem 232 Let be an acute triangle with circumcircle . Let and
be respectively the midpoints of the arcs and of . Show that
the inradius of triangle is not less than the inradius of triangle .
TST 2016

Problem 233 Find all functions such that


for all (Here denotes the set of all real numbers.)
TST 2016

Problem 234 Let denote the set of all natural numbers. Show that there exists two
nonempty subsets and of such that every number in can
be expressed as the product of a number in and a number in ; each prime
number is a divisor of some number in and also some number in ; one of the
sets and has the following property: if the numbers in this set are written as
, then for any given positive integer there exists
such that . Each set has infinitely many composite numbers.
TST 2016

165
Problem 235 Let and
with real numbers,
an integer and . Define and
.

(a) Find the number of unordered pairs of polynomials with exactly


two common roots.

(b) For any , find the sum of the elements of .


TST 2017

Problem 236 Find all positive integers such that


TST 2017

Problem 237 Let be a cyclic quadrilateral inscribed in circle with


. Let and be the projections of on the
lines respectively. Let be the mid-points of sides
respectively.

(a) Prove that are concyclic.

(b) If is the radius of and is the distance between its center and , then find
the radius of the circle in (a) in terms of and .
TST 2017

166
Problem 238 In an acute triangle , points and lie on side with
. Let be the circumcenters of triangles
, respectively. Prove that
are con-cyclic if and only if are collinear.
TST 2017

Problem 239 Let be pairwise distinct positive integers such that

is an integer. Prove that is not a


prime number.”
TST 2017

Problem 240 There are lamps arranged in a circle in that order. At


any given time, each lamp is either on or off. Every second, each lamp undergoes a
change according to the following rule:

(a) For each lamp , if have the same state in the previous second,
then is off right now. (Indices taken mod .)

(b) Otherwise, is on right now.

Initially, all the lamps are off, except for which is on. Prove that for infinitely
many integers all the lamps will be off eventually, after a finite amount of time.
TST 2017

167
Problem 241 Let be distinct positive real numbers with . Prove that

TST 2017

Problem 242 Define a sequence of integers and


for all . Suppose is a prime with
. Prove that it is possible to choose such that for any
.
TST 2017

Problem 243 Let be a positive integer. An matrix is called good if each


entry is a non-negative integer, the sum of entries in each row and each column is
equal. A permutation matrix is an matrix consisting of ones and
zeroes such that each row and each column has exactly one non-zero entry.
Prove that any good matrix is a sum of finitely many permutation matrices.
TST 2017

Problem 244 Suppose are non constant polynomials. Suppose neither of


is the square of a real polynomial but is. Prove that is not the
square of a real polynomial.
TST 2017

168
Problem 245 Let be a positive integer relatively prime to . We paint the vertices of
a regular -gon with three colors so that there is an odd number of vertices of each
color. Show that there exists an isosceles triangle whose three vertices are of
different colors.
TST 2017

Problem 246 Let and be fixed points on the coordinate


plane. A nonempty, bounded subset of the plane is said to be nice if

there is a point in such that for every point in , the segment lies
entirely in ; and

for any triangle , there exists a unique point in and a permutation


of the indices for which triangles and are
similar.

Prove that there exist two distinct nice subsets and of the set
such that if and are the unique choices of
points in , then the product is a constant independent of the triangle
.
TST 2017

Problem 247 Find all positive integers for which all positive divisors of can be
put into the cells of a rectangular table under the following constraints:
each cell contains a distinct divisor; the sums of all rows are equal; and the sums of
all columns are equal.
TST 2017

169
Problem 248 Let be a triangle with and let be its
incenter. The line meets at , and the line through perpendicular to
meets at . Prove that the reflection of in lies on the circumcircle
of triangle .
TST 2017

Problem 249 Prove that for any positive integers and we have

TST 2017

Problem 250 Let be an acute angled triangle with incenter . Line


perpendicular to at meets and at points and respectively. Let
be the incenters of and respectively. Suppose
lie on a circle. Prove that .
TST 2017

Problem 251 For each define the polynomial


Prove that

(a) For each , has a unique positive real root ;

(b) is a strictly increasing sequence;

(c)
TST 2017

170
Problem 252 Let be a positive integer which is not a perfect square, and consider

the equation Let be the set of positive integers for which the
equation admits a solution in with , and let be the set of positive
integers for which the equation admits a solution in with . Show
that .
TST 2017

Problem 253 Let be an acute triangle. are the touch points of


incircle with respectively. intersect incircle at

respectively. If, Then

prove that . Also prove that there exists integers such that,
, .
TST 2018

Problem 254 A digit number is called interesting if its digits are distinct and is
divisible by . Then find the number of interesting numbers.
TST 2018

Problem 255 Let be sequences of positive reals such that,

for all .
Prove that, .
TST 2018

171
Problem 256 Let be a convex quadrilateral inscribed in a circle with center
which does not lie on either diagonal. If the circumcentre of triangle lies
on the line , prove that the circumcentre of triangle lies on the line .
TST 2018

Problem 257 For an integer find all so that


(a)

(b)

(c)
TST 2018

Problem 258 A convex polygon has the property that its vertices are coloured by three
colors, each color occurring at least once and any two adjacent vertices having
different colors. Prove that the polygon can be divided into triangles by diagonals, no
two of which intersect in the interior of the polygon, in such a way that all the
resulting triangles have vertices of all three colors.
TST 2018

Problem 259 A rectangle with odd integer side lengths is divided into small
rectangles with integer side lengths. Prove that there is at least one among the small
rectangles whose distances from the four sides of are either all odd or all even.
TST 2018

172
Problem 260 Let be three points in that order on a line in the plane, and
suppose . Draw semicircles and respectively with and
as diameters, both on the same side of . Let the common tangent to and
touch them respectively at and , . Let and be points on the
segment such that the semicircle with as diameter touches in
and in .
Prove that are concyclic.
Prove that are concyclic.
TST 2018

Problem 261 Find all functions such that


for all .
TST 2018

Problem 262 For a natural number , define to be the set of all triplets
of natural numbers, with odd and , such that
and divides . Find all values of for which is finite.
TST 2018

Problem 263 In triangle , let be the excircle opposite to . Let and


be the points where is tangent to , and , respectively. The circle
intersects line at and . Let be the midpoint of . Prove that
the circle is tangent to .
TST 2018

173
Problem 264 Sir Alex plays the following game on a row of 9 cells. Initially, all cells are
empty. In each move, Sir Alex is allowed to perform exactly one of the following two
operations:
Choose any number of the form , where is a non-negative integer, and put it into
an empty cell.
Choose two (not necessarily adjacent) cells with the same number in them; denote
that number by . Replace the number in one of the cells with and erase the
number in the other cell.
At the end of the game, one cell contains , where is a given positive integer,
while the other cells are empty. Determine the maximum number of moves that Sir
Alex could have made, in terms of .
TST 2018

Problem 265 Let be a positive integer. Define a chameleon to be any sequence of


letters, with exactly occurrences of each of the letters and . Define a
swap to be the transposition of two adjacent letters in a chameleon. Prove that for
any chameleon , there exists a chameleon such that cannot be changed to
using fewer than swaps.
TST 2018

Problem 266 Let be a finite set, and let be the set of all functions from to
. Let be an element of , and let be the image of under . Suppose
that for every in with . Show that .
TST 2018

174
Problem 267 Find the smallest positive integer or show no such exists, with the
following property: there are infinitely many distinct -tuples of positive rational
numbers such that both

are integers.
TST 2018

Problem 268 Let be a triangle and be cevians concurrent at a


point . Suppose each of the quadrilaterals and has
both circumcircle and incircle. Prove that is equilateral and coincides with
the center of the triangle.
TST 2018

Problem 269 Let be a natural number. Let be real


numbers such that and
If , the
smallest integer larger than , then show that
TST 2018

Problem 270 Determine all integers having the following property: for any
integers whose sum is not divisible by , there exists an index
such that none of the numbers
is divisible by . Here, we let
when .
TST 2018

175
Problem 271 In an acute angled triangle with , let denote the
incenter and the midpoint of side . The line through perpendicular to
intersects the tangent from to the incircle (different from line ) at a
point > Show that is tangent to the circumcircle of triangle .
TST 2019

Problem 272 Show that there do not exist natural numbers such
that the numbers are all
powers of
TST 2019

Problem 273 Let be a given positive integer. Sisyphus performs a sequence of turns
on a board consisting of squares in a row, numbered to from left to right.
Initially, stones are put into square , and the other squares are empty. At every
turn, Sisyphus chooses any nonempty square, say with stones, takes one of these
stones and moves it to the right by at most squares (the stone should say within
the board). Sisyphus' aim is to move all stones to square .
Prove that Sisyphus cannot reach the aim in less than

turns. (As usual, stands for the least integer


not smaller than .)
TST 2019

Problem 274 Let denote the set of all positive rational numbers. Determine all
functions satisfying for all
TST 2019

176
Problem 275 Let be a natural number. A tiling of a board is a placing of
dominos (of size or ) such that each of them covers exactly two
squares of the board and they cover all the board.Consider now two separate
tilings of a board: one with red dominos and the other with blue dominos.
We say two squares are red neighbors if they are covered by the same red domino in
the red tiling; similarly define blue neighbors.

Suppose we can assign a non-zero integer to each of the squares such that the
number on any square equals the difference between the numbers on its red and
blue neighbors i.e the number on its red neighbor minus the number on its blue
neighbor. Show that is divisible by
TST 2019

Problem 276 Let be a function such that


for all pairs of positive integers. Prove that there
exists a positive integer which divides all values of .
TST 2019

Problem 277 Given any set of positive integers, show that at least one of the
following two assertions holds:

(1) There exist distinct finite subsets and of such that


;

(2) There exists a positive rational number such that for all
finite subsets of .
TST 2019

177
Problem 278 Let be an acute-angled scalene triangle with circumcircle and
circumcenter . Suppose . Let be the orthocenter and be the
incenter of triangle . Let be the midpoint of the arc of the circumcircle
of triangle , containing .

Let be a point on the arc of not containing , such that


. Let be the circumcenter of triangle . Prove that the
lines and meet on .
TST 2019

Problem 279 Let be a positive integer. The organizing committee of a tennis


tournament is to schedule the matches for players so that every two players play
once, each day exactly one match is played, and each player arrives at the
tournament site the day of his first match, and departs the day of his last match. For
every day a player is present on the tournament, the committee has to pay coin to
the hotel. The organizers want to design the schedule so as to minimize the total cost
of all players' stays. Determine this minimum cost.
TST 2019

Problem 280 Determine all non-constant monic polynomials with integer


coefficients for which there exists a natural number such that for all ,
divides
TST 2019

178
Problem 281 Determine all functions satisfying

for all .
TST 2019

Problem 282 Let be the circumcentre, and be the circumcircle of an


acute-angled triangle . Let be an arbitrary point on , distinct from ,
, , and their antipodes in . Denote the circumcenter of the triangles ,
, and by , , and , respectively. The lines , ,
perpendicular to , , and pass through , , and , respectively.
Prove that the circumcircle of triangle formed by , , and is tangent to the
line .
TST 2019

Problem 283 Let be a set of distinct positive even numbers and


be a set of distinct positive odd numbers such that
Prove that

TST 2019

Problem 284 Let be a triangle with Points are


chosen on the sides respectively so that
Let and be the perimeters of the triangles and , respectively.
Prove that
TST 2019

179
Problem 285 Let be an integer. Solve in reals:

TST 2019

Problem 286 Let the points and be the circumcenter and orthocenter of an
acute angled triangle Let be the midpoint of Let be the point on
the angle bisector of such that Let be the point such that
is a rectangle. Prove that are collinear.
TST 2019

Problem 287 Determine all positive integers satisfying the condition that there
exists a unique positive integer such that there exists a rectangle which can be
decomposed into congruent squares and can also be decomposed into
congruent squares.
TST 2019

Problem 288 There are coins on a table. Some are placed head up and others
tail up. A group of persons perform the following operations: the first person
chooses any one coin and then turns it over, the second person chooses any two
coins and turns them over and so on and the -th person turns over all the
coins. Prove that no matter which sides the coins are up initially, the persons
can come up with a procedure for turning the coins such that all the coins have the
same side up at the end of the operations.
TST 2019

180
Solutions for Solved Problems

(Page 10 - 19)
Solutions

Solution. 1.
Note that g(1) = f (1) = 0, so 1 is a root of both f ( x ) and g( x ). Let p and q be the
other two roots of f(x), so p2 and q2 are the other two roots of g( x ). We then get pq = −c
andp2 .q2 = − a, so a = −c2 . Also, (− a)2 = ( p + q + 1)2 = p2 + q2 + 1 + 2( pq + p + q) =
−b + 2b = b.Therefore b = c4 . Since f (1) = 0we therefore get 1 + c − c2 + c4 = 0 .
Factorising, we get (c + 1)(c3 − c2 + 1) = 0. Note that c3 − c2 + 1 = 0 has no integer
root and hence c = −1, b = 1, a = −1. Therefore a2023 + b2023 + c2023 = −1

Solution. 2. denote Sn = αn + βn + γn also ,Sn ∈ Z +


claim:- S10,000+n+k = Sk now from netwon sums clearly Sn can be recursively written
in terms of Sn−1 + Sn−2 + · · · + S1
α35005 + β35005 + γ35005 ≡ S35005 ≡ S5005 ( mod 10000)
S13 ≡ Sn ( mod 16)
S5 ≡ Sn ( mod 625) by CRT Sn ≡ −7( mod 625)
Sn ≡ 9( mod 16)
Sn ≡ −7( mod 10, 000)
Sn ≡ 9993( mod 10, 000), where n = 35005 so S35005 ≡ 9993( mod 10, 000)

Solution. 3. Let D = (0, 0), A = (0, a), B = (−b, 0), C = (c, 0). If you construct squares
circumscribing ABWX and CAYZ whose sides are parallel to the axes, it becomes
clear that:

W = (−( a + b), b)
X = (− a, a + b)
Y = ( a, a + c)
Z = ( a + c, c)

So we need to show that M and P lie on the y-axis. Since X and Y are equidistant from
the y-axis, M lies on the y-axis. The equations for lines CW and BZ are:

b
y=− ( x − c)
a+b+c
c
y= ( x + b)
a+b+c
 
We now see that the intersection, P, is 0, a+bcb+c , so we’re done.
182
Solutions

Solution. 4. 2x + 1 = 4x + 2x+1 + 1 which can be written in another form as
p √
4 x + 4x +1 + 2x +2 + 1, now this can be further expanded so on to get g ( x ) =
q p √
4x + 4x+1 + 4x+2 + · · · so we have g( x ) = 2x + 1. so g(2021) = 22021 + 1 which
is 8609( mod 10000) so we get 23 as our ans

Solution. 5. as we are given xyzw = 1 , so from this we get second equation as


y
z + yx + wx + wz = 2. so say a = yz , b = yx , wx = c, wz = d. so we get a + b + c + d = 2.
from fourth equation we get 1a + 1b + 1c + 1d = −1. so we get abc + abd + acd + bcd = −1.
also from third equation we get ab + bc + cd + ad + w2 y2 + x2 z2 = −3. notice we want
ac and bd. so ac = x21z2 . so this gives ab + bc + cd + ad + ac + bd = −3. and abcd = 1.
so we get a equation α4 − 2α3 − 3α2 + α + 1 = 0 whose roots are a, b, c, d. so we
get (α + 1)(α3 − 3α2 + 1) = 0. this gives α = −1. and three distinct complex ( not
necessarily non real) solutions. so as α = −1. we get any one pair say yx = −1. so
x = −y = k for some k ∈ C. so as z, w, will be distinct we will get 4 quadruples from
−k, k, w, z solution so we can have such 4 · 4 = 16 quadruples.


Solution. 6. so we construct a triangle ABC such that ∠ ACB = π2 and AB = 3, BC =

2, AC = 1. now choose a point P inside the triangle such√that ∠ APC = ∠ APB√=
3 2
∠ BPC = 2π 3 . so we get [ APC ] + [ APB ] + [ BPC ] = [ ABC ]. so 4 · ( xy + yz + zx ) = 2 .

2√ 2
so we get xy + yz + zx = now adding the given three equations we get x2 +
.
√ 3 p √
y2 + z2 = 9−3 6 . and hence we get x + y + z = 3 + 6. now we are provided that
√ p √
x2 + xz√+ z2 = 3. so we get ( x + z)2 − xz = 3 or y2 + 3 + 6 − 2y 3 + 6 − zx = 3
and 9−3 6 − y2 + zx = 3. adding these two equations we get y2 = 2√
. solving on
√ 3(3+ 6)
6−2 6
we get y2 = 9 . so we get m + n + p + q = 23 .

183
Solutions
p
Solution. 7. Squaring the given equation, we arrive at the conclusion that 2p q2 + r
p p √
is an integer, or therefore q2 + r is an integer. Let q2 + r = u and s2 + t = v.
Therefore, we arrive at u2 − q2 = r and v2 − s2 = t. This gives us u − q = 1 from
difference of squares and r being prime, and similarly v − s = 1. Therefore, we get
u + q = r and v + s = t. This gives us the following four vital equations which we will
constantly use throughout the solution:

2q + 1 = r (1)

2s + 1 = t(2)
u = q + 1(3)
v = s + 1(4)
p √
Go back to the original equation now. Since q2 + r = u = q + 1 and s2 + t =
v = s + 1, plugging this back into the original equation, we arrive at p + (q + 1) =
(s + 1) =⇒
p+q = s
. Taking the equation modulo 2, we arrive at p = 2, q = 2, or , s = 2.
Assume p p = 2, which p gives us s = q + 2. Going back to the original equation,
+ 2+r = (q + 2)2 + t. Therefore, 2
we have 2 q psquaring this, we get 4 + (q +
r ) + 4 q2 + r = q2 + 4q + 4 + t. Therefore, r + 4 q2 + r = 4q + t =⇒ (3) =⇒
p

r + 4(q + 1) = 4q + t =⇒ r + 4 = t. Therefore, t = 2q + 5 using (1). Since t is


prime, 2q + 5 is prime, since s is prime q + 2 is prime, and q is prime, and since r is
prime 2q + 1 is prime. Therefore, we have an obvious contradiction mod 3 unless
one of the values is 3. The only value that gives q being prime is 3, which gives
( p, q, r, s, t) = (2, 3, 7, 5,
p11). √
If q = 2, then from q2 + r = q + 1 we have 4 + r = 3 =⇒ r = 5. Plugging this
back into the original equation, we obtain p + 3 = s + 1 =⇒ p + 2 = s. We also have
t = 2s + 1 from (2), so therefore from p being prime, we must have s − 2 being prime,
from t being prime we must have 2s + 1 being prime, and s is prime. This is again a
contradiction mod 3 unless one of the values is 3. If s = 3, then we get p = 1 absurd. If
s − 2 = 3, then s = 5 and p = 3 and t = 11. There are no other solutions for this case.
Therefore, we arrive at the solution ( p, q, r, s, t)√= (3, 2, 5, 5, 11).
2
p we have to consider s = 2. Since s + t = s + 1 = 3, we must have
Lastly,
p + q2 + r = 3, which is clearly absurd.
The solutions are whence ( p, q, r, s, t) = (2, 3, 7, 5, 11), (3, 2, 5, 5, 11) .

Solution. 8. Let P( x, y) be the assertion that f ( x f (y) + f ( x )) = 2 f ( x ) + xy. P(1, y)


shows that f is surjective. Hence there exists z such that f (z) = 0. P(z, z) implies
f (0) = z2 . Assume z = 0. Then P( x, 0) implies f ( f ( x )) = 2 f ( x ) and because of
surjectivity we get f ( x ) = 2x for all x but this does not satisfy the original equation.
Hence z ̸= 0. Now, P(z, y) implies f (z f (y)) = zy and therefore f must also be
184
Solutions

injective hence bijective. P(z, 0) implies f (z3 ) = 0 = f (z) and because of injectivity
we get z = z3 and thus z = ±1. In any case f (0) = z2 = 1. Now, P(−1, −1) implies
f (−1) = 0 and thus z = −1. Now, P( x, −1) yields f ( f ( x )) = 2 f ( x ) − x meaning we
can prove easily by induction f (n) = n + 1 for every positive integer n. Also, P(−1, y)
yields f (− f ( x )) = − x so that we can extend this proof to the negative integers setting
x = n. Assume f ( a) = 2a for some a. Then P( a, −4) implies f (− a) = 0 and thus
a = 1 meaning this a is unique. Now, P( x, −2) implies f ( f ( x ) − x ) = 2( f ( x ) − x ) and
therefore f ( x ) − x = a = 1 and hence f ( x ) = x + 1 for every real x which is indeed a
solution.

Solution. 9. The question is asking, how many permutations of this set have no orbit
with length greater than 2?
There can be 1, 3, 5, or 7 orbits of length 1.
One orbit of length 1: pick a number, then count how many pairings are possible.
7 · 5 · 3 · 1 = 105
Three orbits of length 1: choose three, then count the pairings. (73) · 3 · 1 = 105
Five orbits of length 1: choose five, then just one pairing is possible. (75) · 1 = 21
Seven orbits of length 1: that is, f ( a) = a 1
105 + 105 + 21 + 1 = 232

q  q  q  q 
Solution. 10. Lemma 1: f n ( x ) ̸= f n+1 ( x ) =⇒ f n +1 ( x ) = f n +2 ( x )

jp k
Let f n ( x ) = a.
a2 ≤ f n ( x ) ≤ a2 + 2a and a2 + 2a + 1 ≤ f n+1 ( x ).
a2 + j2a + 1 ≤ kf n+1 ( x ) ≤ a2 + 3a + 1 = a2 + 3a + 1 < ( a + 2)2
p
=⇒ f n ( x ) = a + 1.
Calculating f n+2 ( x ) for the largest value of f n+1 ( x ):
f n+j2 ( x ) ≤ a2 +k 3a + jp1+1+a+ 2
k 1 = a + 4a + 3 < ( a + 2)
2
p
So f n +1 ( x ) = f n+2 ( x ) = a + 1. ■
q  q  q  q 
Lemma 2: f n (x) = f n+1 ( x ) = a =⇒ f n +2 ( x ) = f n +3 ( x ) = a + 1

a2 ≤ f n ( x ) < f n+1 ( x ) ≤ a2 + 2a.


Calculating f n+2 ( x ) for the smallest value of f n ( x ):
a2 + a + 1 ≤ f n+1 ( x ) and f n+2 ( x ) ≥ a2 + a + 1 + 1 + a = a2 + 2a + 2 > ( a + 1)2 .
185
Solutions

Calculating f n+2 ( x ) for the largest value of f n+1 ( x ):


f n+2 ( x ) ≤ a2 + 2a + 1 + a = a2j+ 3a + 1 <k( a + 2)2 .
( a + 1)2 < f n +2 < ( a + 2)2 ⇒
p
f n +2 ( x ) = a + 1
jp k jp k jp k jp k
From Lemma 1, f n +1 ( x ) ̸ = f n+2 ( x ) =⇒ f n +2 ( x ) = f n +3 ( x ) . ■
Since f 21 ( x ) = 2013 and 442 < 2013 < 452 , f 22 ( x ) = 2013 + 1 + 44 = 2058 ≥ 452 . If
we use the jabove lemmas repeatedly, we get
√  p k
x = f (x) = a
jp k jp k
f 2 (x) = f 3 (x) = a + 1
...
jp k jp k
20
f (x) = 21
f ( x ) = a + 10 = 44
So a = 34 and 342 ≤ x < 352 .
Repeatedly applting f gives f 21 ( x ) = x + 35 + 36 + 36 + 37 + 37 + · · · + 43 + 44 + 45.
Also f 21 ( x ) = 2013 = x + 35 + 80 · 10 ⇒ x = 1178

Solution. 11. Note that cubes are −1, 0, 1 (mod 7). Then 18 + c3 − b3 is 2, 3, 4, 5, 6
(mod 7). But a! is 0 (mod 7) if a ≥ 7. So a < 7. Now also only a = 2, 3, 4, 6 work by
mod 7. Now we bash, giving a = 2, 3, 4 not working. But a = 6 works, with b = 3 and
c = 9. Thus we get 1.

2
Solution. 12. Let x − 3 = a, y − 32 = b and z − 32 = c. We have a + b + c = 0. Also,

        
2 2 2 2 2 2 4
1= a+ b+ + b+ c+ + c+ a+ = ab + bc + ca + =⇒
3 3 3 3 3 3 3

Hence, -1/3=
1
ab + c( a + b) = ab − ( a + b)2 = − a2 − b2 − ab ⇒ a2 + b2 + ab =
3
. Then,
4 2
= 4a2 + 4b2 + 4ab ≥ a2 + b2 − 2ab = ( a − b)2 ⇒ √ ≥ a − b = x − y
3 3
 
Equality occurs when ( x, y, z) = 32 + √1 , 32 − √1 , 23 .
3 3
186
Solutions

Solution. 13 We claim that (n, m) = (2, 2), (4, 2) are the only solutions. It is easy
to see they work, so it suffices to show that these are the only solutions. Let p be
the smallest prime divisor of n. If p ≤ n − ϕ(n) − 1, then taking modulo p yields
2n ≡ 1 (mod p). Let d be the order of 2 modulo p. Then d | gcd(n, p − 1) = 1 =⇒
d = 1 =⇒ p | 1. Contradiction. So all prime divisors of n are ≥ n − ϕ(n).
α
Let n = p1α1 p2α2 ...pk k with pi being primes and p1 < p2 < ... < pk . We have n −
α −1
ϕ(n) = p1α1 −1 p2α2 −1 ...pk k ( p1 p2 ...pk − ( p1 − 1)( p2 − 1)...( pk − 1)) ≤ p1 . If k ≥ 2, then
α −1
p1 p2 − ( p1 − 1)( p2 − 1) = p1 + p2 − 1 ≤ p1α1 −1 p2α2 −1 ...pk k ( p1 p2 ...pk − ( p1 − 1)( p2 −
1)...( pk − 1)) ≤ p1 =⇒ p2 ≤ 1. Contradiction. So k = 1 and we must have α1 ≤ 2.
So n must be of the form p or p2 where p is a prime. If n = p where p is a prime,
then 2 p = pm =⇒ p = 2 =⇒ (n, m) = (2, 2) is a solution. If n = p2 where p is a
2
prime, then 2 p + ( p − 1)! = p2m + 1. If p ≥ 3, taking modulo 4 yields ( p − 1)! ≡ 2
(mod 4) =⇒ p ≤ 3 =⇒ p = 3 but when p = 3 there is no solution. Finally, if p = 2,
then we get (n, m) = (4, 2) as a solution.

Solution. 14. property P:- The numbers z1 + z2 , z1 + z3 , z2 + z3 are pairwise distinct.


Assume the contrary: z1 + z2 = z1 + z3 =⇒ z2 = z3 , contradiction with the initial
condition.
Denote (z1 + z2 )3 = (z2 + z3 )3 = (z3 + z1 )3 = z. Then z ̸= 0. If we assume z = 0,
then z1 + z2 = z1 + z3 = z2 + z3 = 0, contradiction with the property P. Results:
∃ρ > 0, α ∈ R such that z = ρ(cos α + i sin α). 
√ α α
The equation w3 = z has the roots: w0 = 3 ρ cos + i sin ;
 3 3
α + 2π α + 2π α + 4π α + 4π
  
√ √
w1 = ρ cos
3 + i sin ; w2 = ρ cos
3 + i sin . Hence:
3 3 3 3
{z1 + z2 , z1 + z3 , z2 + z3 } ⊂ {w0 , w1 , w2 }. Using the property P results: {z1 + z2 , z1 +
z3 , z2 + z3 } = {w0 , w1 , w2 }. WLOG, we can consider: z1 +z2 = w0; z1 + z3= w1 ; z 2+
√ √ α α
z3 = w2 . Results: z1 − z2 = w1 − w2 = 3 ρ · 3 sin + π − i sin +π ;
3 3
√ √ α+π α+π

z2 − z3 = w0 − w1 = 3 ρ · 3 sin − i cos ; z 3 − z 1 = w2 − w0 =
3 3
√ α + 2π α + 2π √ √
 
√3 ρ· 3 sin − i cos . | z1 − z2 | = | z2 − z3 | = | z3 − z1 | = 3 ρ · 3
3 3

R π/2 R1
Solution. 15. First note that sin(sin x )dx = √sin x dx. Now we also know that
0 0 1− x 2

187
Solutions
x3
x− 3! ≤ sin x ≤ x for all x ∈ R. So
3
x − x3!
Z 1 Z 1 Z 1
sin x x
√ dx ≤ √ dx ≤ √ dx
0 1− x2 0 1 − x2 0 1 − x2

x3
Z 1 Z 1 Z 1 Z 1
x 1 sin x x
=⇒ √ dx − √ dx ≤ √ dx ≤ √ dx
0 1 − x2 3! 0 1 − x2 0 1 − x2 0 1 − x2
Z 1
1 sin x
=⇒ 1 − ≤ √ dx ≤ 1
9 0 1 − x2
Z π/2
8
=⇒ ≤ sin(sin x )dx ≤ 1
9 0
Hence, proved.

Solution. 16. Let L := limx→∞ x f ( x ). So there exists N > 1 such that | x f ( x ) − L| ≤ 1


for x ≥ N. Now write
Z ∞ Z N Z ∞
f (x) f (x) f (x)
dx = dx + dx. (1)
1 x 1 x N x
The first integral on the right-hand side of (1) is a proper integral and so convergent.
For the second integral, write

| f ( x )| | x f ( x )| | x f ( x ) − L| | L| 1 + | L|
= 2
≤ 2
+ 2 ≤ . (2)
x x x x x2
R∞ dx
R ∞ f (x)
So, since N x 2 is convergent, N x dx is (absolutely) convergent too, by the com-
parison test.
Next, put x t = y. Then
Z a Z at Z at Z at
f (y) 1/t f (y) 1/t f (y)
t f ( x t ) dx = y dy = (y − 1) dy + dy
1 1 y 1 y 1 y

and so Z a Z at Z ∞
t f (y) 1/t f (y)
lim t f ( x ) dx = lim (y − 1) dy + dy.
t→∞ 1 t→∞ 1 y 1 y
So we are done if we show that
Z at
f (y)
lim (y1/t − 1) dy = 0. (3)
t→∞ 1 y

To prove (3), we use (2) to write


Z at Z N Z at 1/t
| f (y)| | f (y)| y −1
0≤ (y1/t − 1) dy ≤ (y1/t − 1) dy + (1 + | L|) dy.
1 y 1 y N y2
188
Solutions

So if M := maxy∈[1,N ] | f (y)|, then


Z at Z N 1/t Z at 1/t
| f (y)| y −1 y −1
0≤ (y1/t − 1) dy ≤ M dy + (1 + | L|) dy.
1 y 1 y N y2

An easy calculation shows that


Z N 1/t Z at 1/t
y −1 y −1
lim dy = lim dy = 0
t→∞ 1 y t→∞ N y2

and that completes the proof of (3) and the first part of your problem.
1
For the second part of your problem, apply the first part to the function f ( x ) = 1+ x ,
which satisfies all the conditions required, to get
Z a Z ∞
dx dx
lim t = = ln 2.
t→∞ 1 1 + xt 1 x (1 + x )

Solution. 17. Note that since RHS ∈ Z, then all solutions must be integers.
Let n = 2 and the equation is x = n2 and so : x = 2k implies 2k = k and so k = 0
 

and so x = 0 which indeed is a solution. x = 2k + 1 implies 2k + 1 = k and so k = −1


and so x = −1 which indeed is a solution.
  SoA2 = {−1, 0}
Let n = 3 and the equation is x = n2 + n3 and so : x = 6k implies 6k = 5k and so
k = 0 and so x = 0 which indeed is a solution. x = 6k + 1 implies 6k + 1 = 5k and so
k = −1 and so x = −5 which indeed is a solution. x = 6k + 2 implies 6k + 2 = 5k + 1
and so k = −1 and so x = −4 which indeed is a solution. x = 6k + 3 implies
6k + 3 = 5k + 2 and so k = −1 and so x = −3 which indeed is a solution. x = 6k + 4
implies 6k + 4 = 5k + 3 and so k = −1 and so x = −2 which indeed is a solution.
x = 6k + 5 implies 6k + 5 = 5k + 3 and so k = −2 and so x = −7 which indeed is a
solution. So A2 = {−7, −5, −4, −3, −2, 0}
And so A2 ∪ A3 = {−7, −5, −4, −3, −2, −1, 0}

Solution. 18. Substitute x := y + 1 to the preposition to get | f (y + 1) − f (y)| ≤


|y + 1 − y| = 1. Because f is injective, | f (y + 1) − f (y)| = 1.
Now set f (0) = a. Because | f (1) − f (0)| = 1, either f (1) = a + 1 or f (1) = a − 1.
Suppose f (1) = a + 1. We induct that f (n) = a + n.
For n = 0, 1, this is correct. Suppose for n = k, k + 1, this claim is correct, then:
| f (k + 2) − f (k + 1)| = 1 f (k + 2) = f (k + 1) + 1 ∨ f (k + 1) − 1 f (k + 2) = a + k +
189
Solutions

2 ∨ a + k But f (k ) = a + k, so since f is injective, f (k + 2) = a + k + 2, and the claim is


proven for n ≥ 0.
For n = 1, 0, this is correct. Suppose for n = k + 1, k, this claim is correct, then:
| f (k − 1) − f (k)| = 1 f (k − 1) = f (k) + 1 ∨ f (k) − 1 f (k − 1) = a + k + 1 ∨ a + k − 1
But f (k + 1) = a + k + 1, so f (k − 1) = a + k − 1, and the claim is proven for n ≤ 0.
Hence f (n) = a + n for all integer n.
The same argument can be applied to where f (0) = a, f (1) = a − 1 to get f (n) =
a − n. Hence we have our solutions:
f ( x ) = c + x for all x, or f ( x ) = c − x for all x, for any c.

 Rb Rb
Solution. 19. Let P( a, b) be the assertion a2 + ab + b2 f ( x ) dx = 3 x2 f ( x ) dx, and
a a

t ∈ R, we have
Zt Zt
2
P(0, t) =⇒ t f ( x ) dx = 3 x2 f ( x ) dx
0 0

since f is continuous here, we know that f ′ exists, and differentiating with respect to t
on both sides gives

Zt Zt
2 2
t f (t) + 2t f ( x ) dx = 3t f (t) =⇒ f ( x ) dx = t f (t)
0 0

and differentiating with respect to t again gives

f (t) = f (t) + t f ′ ( x ) =⇒ f ′ (t) = 0

and so f is a constant function. It is easy to check that f ( x ) = C where C ∈ R works.


Rt Rt Rt
Solution. 20. t = 0 dx ≤ 0 e x dx ≤ 0 edx = et ⇒ 1 + t ≤ et ≤ 1 + et, (∀) t ∈ [0, 1] .
1
R1 n
R 1 xn R1 n e
⇒ 1 + n+ 1 = 0 (1 + x ) dx ≤ 0 e dx ≤ 0 (1 + ex ) dx = 1 + n+1 , (∀) n ∈ N
R1 n
⇒ lim 0 e x dx = 1.
n→∞

190
Solutions

Solution. 21. Lemma: If P, Q, R are three 2 × 2 matrices, then


det( P + Q + R) = det( P + Q) + det( Q + R) + det( P + R) − det( P) − det( Q) −
det( R).
Apply the above lemma to get that
det( A + 2B) − det( B + 2A) = det(2B) − det(2A) − (det( B) − det( A)).
Now use that det( A) = 0 and det( B) = 2 to get the answer as 6.

Solution. 22. Since 346346 ≡ 4 (mod 9), we need at least four perfect cubes. On the
other hand, (7 · 346115 )3 + (346115 )3 + (346115 )3 + (346115 )3 = (73 + 13 + 13 + 13 ) ·
346345 = 346346 . So the minimum number is 4 .

Solution. 23. Suppose , that x = c is a integer solution to this equation . Then,


2abc4 − a2 c2 − b2 − 1 = 0 =⇒ ac2 (2bc2 − a) = b2 + 1 . Note that b2 + 1 prime factors
are either 2 or of form 4k + 1. Also we see that if , a is even then , we get 4|b2 + 1 a
contradiction . similarly we see that c is also odd. And indeed if b is odd , then we
again get a similar contradiction. Now , if a ≡ 3 (mod 4) then , we it means there is
prime factor of a which is of form 4k + 3 a condtradiction. Also c must also be of form
4k + 1 . And b is even . But then we have 2bc2 − a2 ≡ 3 (mod 4) a contradiction .

Solution. 24. ∠ AUC ≡ ∠ AXB ≡ ∠ DXC ≡ ∠ AVD ⇒ ∠ AUT + ∠CUT = ∠ AVT +


∠TVP ⇒ ∠CUT ≡ ∠TVA.
Denote { R} = (VPU ) ∩ AV. Using the fact that ∠CUT ≡ ∠TVR, we get that R, C
and U are colinear.
RP AP RP CP
Now AR · AV = AP · AU ⇒ △ ARP ∼ △ AUV ⇒ UV = AV , but UV = CU ,
AP CP
thus AV = CU ⇒ △CPU ∼ △ ACV. Similarly we prove that △CRV ∼ △ ACU ⇒
AC AV
△ ACV ∼ △ ACU ⇒ ∠VAC ≡ ∠CAU ⇒ AC = AU ⇒ AV = AU ⇒ A, C and M are
colinear.
∠ ATP ≡ ∠ ACP ≡ ∠VCO ≡ ∠OCU, where {O} = AC ∩ UV
∠ MVT + ∠ MAT = ∠ MVT + π − ∠TAC = ∠VUT + π − ∠TPV = π, hence
MVTA is cyclic⇒ ∠ MVA ≡ ∠ MTA
191
Solutions
π π
∠ MVA = ∠VUR = 2 − ∠OCU ⇒ ∠ MTA + ∠ ATP = 2 ⇒ ∠ PTM = π2 .

Solution. 25. We prove both parts. Define MB , MC as the arc midpoints of AC and AB,
O as the circumcenter, and TB , TC as the B-extouch point with AC and the C-extouch
point with AB.
Part 1: If AB + AC = 3BC, then P, I, Q are collinear.
Proof: Observe that s = 2BC, so BTC = s − BC = BC, and similarly CTB = BC.
Therefore, we have
1
∠QCA = ∠TC CB = 90 − B
2
This implies Q is the arc midpoint of ABC. Similarly, P is the arc midpoint of APB, so
P, O, MC and Q, O, MB are collinear. Observe that MB MC is the perpendicular bisector
of AI. Then,
1 BC 1 BC
QMC = 2R sin A = sin A =
2 sin A 2 2 cos 21 A
If QMC = 12 AI, we’re done. Observe that the length of the tangent from A to the
incircle is s − BC = BC, so

1 r BC 1 r sin 12 A 1
tan A = ⇒ QMC = = AI
2 BC 2 cos 21 A 2 = 2

and we are done.


Part 2: If P, I, Q are collinear, then AB + AC = 3BC.
If we let SB , SC be the touch-points of ωC , ω B with AC, AB respectively, then by
homothety, P, SC , MC and Q, SB , MB are collinear. Furthermore, it is well known that
( QISB A) and ( PISC A) are cyclic. If O′ = QMB ∩ PMC , then

∠ ASC O′ + ∠ ASB O′ = ∠ AIQ + ∠ AIP = 180

which means ( ASC O′ SB ) is cyclic. Therefore, ∠ MC O′ MB = 180 − ∠ A. However, since


MC MB = 90 − 21 A, this means PQ = 90 − 21 A, so PQ = MC MB .
Define S = CQ ∩ PB. By pascals on PBMB QCMC , we have I, S, O′ are collinear.
Since S is the exsimilicenter of the incircle and circumcircle, we have I, O, S are collinear.
Now, I claim the only way for O′ , O, I to be collinear is when O = O′ . First of all,
this is possible by setting P, Q as the reflection of MC , MB over O. Next, for any other
placement of P, Q is fixed (since PQ = MC MB . Inverting with the circumcenter, O′
goes to the intersection of MC MB and PQ, so the locus of O′ is ( MC OMB ). However,
since MC MB ∩ PQ lies outside of MC MB , this means the locus of O′ must also be
within ( ABC ), so the only intersection of ( MC OMB ) with OI is O. Therefore, O′ = O.
Finally, this means

1
∠ BCTC = ∠QCA = 90 − ∠ ACMB = 90 − B
2
192
Solutions

so BTC = BC, and s − BC = BC so AB + AC = 3BC.

4 x −cos2 x 4 x −sin2 x 4 x +1−cos2 x 4 x +1−sin2 x


Solution. 26. Proof 2sin − 2cos = cos 2x ⇔ 2sin − 2cos =
4 x +sin2 x 4 x +cos2 x 4 x +sin2 x
 4 2 4 2x

2 cos 2x ⇔ 2sin − 2cos 1 − 2cos x+cos x−sin x−sin
= 2 cos 2x ⇔ 2sin =
4 2
2 cos 2x 2sin x+sin x 1 − 22 cos 2x = 2 cos 2x ⇒ 2 cos 2x 1 − 22 cos 2x ≥ 0 ⇔ cos 2x =
 
4 2
0 ⇒ 2sin x+sin x 1 − 22 cos 2x = 2 cos 2x ⇔ cos 2x = 0 ⇔ x ∈ ± π4 + πk, k ∈ Z
 

Solution. 27. Let f ( x ) = x2 − ( a2 + b2 + c2 + d2 + 1) x + ab + bc + cd + da, let roots of


f be x = r1 , r2 . Since f ∈ Z [ X ] is monic and has an integer root, we see that it must
necessarily have two integer roots. We see that r1 + r2 = a2 + b2 + c2 + d2 + 1 > 0 and
r1 r2 = ab + bc + cd + da > 0, therefore r1 , r2 > 0.
1
Now, a2 + b2 + c2 + d2 = ab + bc + cd + da + [( a − b)2 + (b − c)2 + (c − d)2 + (d −
2
1
2
a) ] and so this re-arranges to (1 − r1 )(r2 + 1) = [( a − b)2 + (b − c)2 + (c − d)2 +
2
(d − a)2 ] ≥ 0 . . . (♣), however r1 , r2 ̸= 0 as ( a, b, c, d) ∈ N and so r1 , r2 ≥ 1, implying
that (1 − r1 )(r2 − 1) ≤ 0 and using (♣), we have that (1 − r1 )(r2 − 1) = 0 or one
of the roots r1 or r2 is 1, let r1 = 1, then a2 + b2 + c2 + d2 = r1 + r2 = r1 + 1 =
ab + bc + cd + da + 1 =⇒ a2 + b2 + c2 + d2 = ab + bc + cd + da and so a = b = c = d
must be true, implying that x = r1 or r2 = ab + bc + cd + da = 4a2 = (2a)2 or 1 = 12
which is as desired

Solution. 28. If x > 1 then LHS > RHS, if 0 < x < 1 then LHS < RHS. so x = 1 is
the only solution

193
Solutions

Solution. 29. a + b − c = 2 and 2ab − c2 = 4 =⇒ ( a + b − c)2 = 2ab − c2 =⇒


a2 + b2 + 2c2 − 2c( a + b) = 0
a2 + b2 + 2c(c − a − b) = 0 =⇒ 2ab − 4c ≤ a2 + b2 − 4c = 0 =⇒ 2ab − 4c ≤ 0
2ab = 4 + c2 =⇒ 2ab − 4c = 4 + c2 − 4c = (c − 2)2 ≤ 0. So c − 2 = 0 =⇒ c = 2.
Therefore, a + b = 4 and 2ab = 8, ( a + b)2 = 16 and 4ab = 16 =⇒ ( a + b)2 =
4ab =⇒ a = b
Then we deduce that a = b = c = 2
So △ ABC is an equilateral triangle

Solution. 30. Let ab = 1010 and suppose a and b have no zeroes in their decimal
representation. One of a, b must be divisible by 2 and one must be divisible by 5, and
neither can be divisible by both. Therefore a = 210 and b = 510 . But then a = 1024,
which has a zero, contradiction.

n n −1 −1 (n−1)(nn−2 +nn−3 +···+1) nn−2 +nn−3 +···+1


Solution. 31. ( n −1)2
= ( n −1)2
= n −1 . I now show nn−2 +

nn−3 + · · · + 1 ≡ 0 (mod n − 1). We know n ≡ 1 (mod n − 1), so nn−2 + nn−3 + · · · +


1 ≡ n − 1 ≡ 0 (mod n − 1)■

Solution. 32. The equality is obvious at x = 2 Now using a is the longest side, For
x > 2 a x = a2 a x−2 = (b2 + c2 )( a x−2 ) > b x + c x

194
Solutions

Solution. 33. (a) x n + x n+1 is strictly increasing and continuous on R + .And x n + x n+1
goes from 0 to +∞ on R + .Then by intermediate value theorem, ∃!xn > 0 s.t. xnn +
xnn+1 = 1■
+1 n +2
(b) Obviously 0 < xn < 1.If xn+1 < xn ,then 1 = xnn + xnn+1 > xnn+ 1 + xn+1 which
is absurd.Thus xn+1 ≥ xn .Then { xn } is increasing and upper-bounded.Therefore ∃α
s.t. limn→+∞ xn = α.0 < α ≤ 1.If 0 < α < 1,then limn→+∞ ln xnn = limn→+∞ n ln xn =
−∞.Hence limn→+∞ xnn = 0.And 0 < xnn+1 < xnn ,by squeeze theorem, limn→+∞ xnn+1 =
0.Thus limn→+∞ xnn + xnn+1 = 0 which is absurd.Therefore α = 1■

p
Solution. 34. 1)Using power mean inequality (k p + (n − k ) p ) ≥ 2 n2 so 1 p + 2 p +
p
... + (n − 1) p + n p + (n − 1) p + ... + 2 p + 1 p ≥ 2n · n2 −→ ∞ as n −→ ∞
1 p +2 p +...+(n−1) p +n p +(n−1) p +...+2 p +1 p
2)For the second case it is easy to calculate n2
=1

Solution. 35. ∠ DFE = ∠ DAE by angles in the same segment property. ∠ ADF =
90◦ − ∠ DAF =⇒ ∠ MDF = 90◦ − ∠OAF = B using MD || AO ∠OAF = 90◦ − B
hence △ FMD ∼ △ ADB by angle equalities

n −r +1 r −1
Solution. 36. Rewrite the identity as ∑∞ n
d=1 ( d )(r −d) = ( r ) (1). Arbitrarily split n

objects into a group of n − r + 1 (Group 1) and r − 1 (Group 2) objects. In order to


choose r objects from the n total, we can either choose 1 object from Group 1 and r − 1
from Group 2 or 2 from Group 1 and r − 2 from Group 2, etc. Thusly we can count the
choosing of r objects from n in two ways, proving the equality of the L.H.S. and R.H.S.
of (1).
Note that d cannot be 0 because we cannot choose r objects from Group 2, which
only has r − 1 objects. Also when d > n − r + 1 or d − 1 > r − 1, the binomial product
vanishes (the sum is equally valid from d = 1 to min({n − r + 1, r })).

195
Solutions
1 1 1 1 ln(2) ln(2)
Solution. 37. let P = 2 2 · 4 4 · 8 8 · (2n ) 2n from here we get ln( P) = 2 + 2 +
 
3 3 n ln(2) 1 2 3 n
8 ln ( 2 ) + · · · + ln ( 2 ) + · · · + 2n ln ( P ) = ln ( 2 ) + + + · · · + 2n
8   2 4 8
n +1   n +1
ln( P) = ln(2) −2 21 (n + 1) − 2 12 + 2 < 2 ln(2)

p √ p √
Solution. 38. Let N = 3 13 + x + 3 13 − x
p√ p √
note that N > 3 x + 3 − x = 0
3 = (13 +
√ p
3
√ p 3
√ p √
cubing both sides yeilds:N x ) + 3 ( 13 + x )( 13 − x )( 3 13 + x +
p3
√ √
13 − x ) + (13 − x )

3 N 3 − 26
169 − x =
3N
3
−26 3
x = 169 − ( N 3N ) >0
It is easy to see that if N ≥ 5,then x < 0
therefore,N ∈ {1, 2, 3, 4} put the values N to get x = 196 as the only integer for N = 2

Solution. 39. Let P( x, y) be the assertion f (max( x, y) + min( f ( x ), f (y))) = x + y Let


a = f (0)
P( x, x ) =⇒ f ( x + f ( x )) = 2x Let x > y. If f ( x ) ≤ f (y), then P( x, y) =⇒
f ( x + f ( x )) = x + y = 2x and so x = y, impossible. So x > y =⇒ f ( x ) > f (y) and
P( x, y =⇒ : f ( x + f (y)) = x + y ∀ x > y (I)
So f ( x ) = x + y − f (y) ∀ x > f (y) Setting there y = 0, we get f ( x ) = x − a ∀ x > a
(II)
Let then y ∈ R and x > max(y, a − f (y)) : x > y implies f ( x + f (y)) = x + y
(see I above) x + f (y) > a implies f ( x + f (y)) = x + f (y) − a (see II above) And so
f (y) = y + a ∀y
Plugging this back in original equation, we get a = 0 and so f ( x ) = x ∀ x

196
Solutions

Solution. 40. We have ( a + d)(b + c) = ( ab + cd) + ( ac + bd) = 9 and ( a + d) + (b +


c) = 6. Therefore, we must have a + d = b + c = 3. Similarly, ( a + b)(c + d) = 5
and ( a + b) + (c + d) = 6. Therefore, { a + b, c + d} = {1, 5}. Also similarly, ( a +
c)(b + d) = ( ab + cd) + ( ad + bc) = 8 and ( a + c) + (b + d) = 6. Therefore, we must
have { a + c, b + d} = {2, 4}. Note that 2a = ( a + b) + ( a + c) − (b + c) ∈ {0, 2, 4, 6}.
Therefore, a ∈ {0, 1, 2, 3}. However, each of these gives us a unique solution (because
each value of a corresponds to a unique value of a + b and a + c), and therefore all the
solutions are ( a, b, c, d) = (0, 1, 2, 3), (1, 0, 3, 2), (2, 3, 0, 1), (3, 2, 1, 0)

sin 3x +sin 3y+sin 3z


Solution. 41. .Given tan( x + y + z) = cos 3x +cos 3y+cos 3z , then sin( x + y + z)(cos 3x +

cos 3y + cos 3z) = cos( x + y + z)(sin 3x + sin 3y + sin 3z),


sin(4x + y + z) + sin(−2x + y + z) + sin( x + 4y + z) + sin( x − 2y + z) + sin( x + y +
4z) + sin( x + y − 2z) = sin(4x + y + z) + sin(2x − y − z) + sin( x + 4y + z) + sin(− x +
2y − z) + sin( x + y + 4z) + sin(− x − y + 2z),
simplifying: sin(2x − y − z) + sin(− x + 2y − z) + sin(2z − x − y) = 0, [sin(2x −
y
y − z) + sin(− x + 2y − z)] + sin 2(z − 2x − 2 ) = 0,
y 3y y y y
2 sin( 2x + 2 − z) cos( 3x x x x
2 − 2 ) + 2 sin( z − 2 − 2 ) cos( z − 2 − 2 ) = 0, sin( 2 + 2 −
3y x y
z)[cos( 3x
2 − 2 ) − cos( z − 2 − 2 )] = 0.
3( x − y )
Solutions: x + y = 2z or 2x = y + z R = 1 + 2 cos 2 which gives 4 as desired
answer

Solution. 42. Let x = cos θ and y = sin θ. Then the second condition becomes
4 sin(θ ) cos(θ )cos(2θ ) = 1, which becomes 2 sin(2θ ) cos(2θ ) = 1,
√ which becomes

sin(4θ ) = 1. Thus, 4θ = π2 , so θ = π8 . Thus, x = cos π8 = 2+ 2

2 , so the an-
2 2
swer is 2 + 2 = 4 S2 :16x2 y2 2x2 − 1 = 1 =⇒ 16x2 (1 − x2 ) 2x2 − 1 = 1 =⇒
 

64x8 − 128x6 + 80x4 − 16x2 + 1 = 0 2


=⇒ 64x8√− 128x6 + 64x4 + 16x4 − 16x2 + 1 = 0 =⇒ 8x4 − 8x2 + 1 = 0 =⇒

2 −(−8)± (−8)2 −4·8·1 2± 2
x = = 4
2·8 √ √
So the largest root is x = 2+ 2
2

And again, a + b = 2 + 2 = 4.

197
Solutions

Solution. 43. We can rewrite f ( x ) as −( x − 2p)2 + 4p2 − p + 1.


The area of this triangle is equal to half the base times the height. Taking the base as
the side of the triangle on the x-axis, the base is the difference between the roots and
the height is the y-coordinate of the vertex.
The y-coordinate of the vertex is, from the above √ form, 4p2 − p + 1. Let this be q.
−4p± 16p2 −4p+4 p
From the quadratic formula, the roots are −2 = 2p ± 4p2 − p + 1, so
2 − p + 1 = 2√ q.
p
the base of the triangle, or the difference in the roots, is 2 4p
  √ √
Therefore, the area of the triangle is 21 (q)(2 q) = q q. This is an integer if and
only if q is the square of an integer.
Now let 4p2 − p + 1√= x2 , so that 4p2 − p + 1 − x2 = 0. From the quadratic formula,

2 1± (4x )2 −15
p = 1± 16x8
−15
= 8 .
2 2
Now let (4x ) − 15 = y , or (4x − y)(4x + y) = 15. Therefore, either 4x − y =
1, 4x + y = 15, or 4x − y = 3, 4x + y = 5. Both of these give integral values for x, so
there are two integral values for x. Each integral value for x gives two rational values
for p, so we have 4 rational values.
Namely, p = − 34 , 0, 41 , 1.

Solution. 44. Using L’hospital’s rule and differentiation under integral sign, we have:
1
lim A → ∞ A1 1 A A x dx = L’Hospital =
R
1 1 −1 1
lim A → ∞A A + 1 A A xx dx = 1 + lim A → ∞ A1 1 A Axx dx = L’Hospital
R R
1 1 −1 1
=1+ lim A → ∞ AAA + 1 A A xx2 dx = 1 + lim A → ∞ A1 1 A Ax2x dx = 1 + lim A→∞ A1 ·
R R
1
A− A A
ln A =1

Solution. 45. By adding all equations we get a3 + b3 + c3 + d3 = 0 which implies that


at least one of them is positive and at least one of them is negative.
Suppose a is positive. If b was positive this will imply c is positive which will imply
d being positive which can’t happen.
Then b is negative. From d3 + a = b we get that d is negative too. Since c3 = a − d
we have that c must be positive.
The other cases are the same thing, basically a and c share signs and b and d also
share signs, opposite to the sign of a and c.
Therefore, suppose a, c are positive and b, d are negative. For simplicity, we will set
a = x 2 , b = − y2 , c = z2 , d = − w2 .
198
Solutions

Our equations then become


 6

 x − y2 = z2
 − y6 + z2 = − w2



 z6 − w2 = x 2
− w6 + x 2 = − y2

Which after rearranging become


 6

 x = y2 + z2
 y6 = z2 + w2



 z6 = w2 + x 2
 6
w = x 2 + y2

Clearly, x6 + z6 = y6 + w6
Suppose x2 > z2 . From this we have x6 > z6 → y2 + z2 > w2 + x2 → y2 > w2 →
y6 > w6
However, x2 + y2 > z2 + w2 → w6 > y6 which is a contradiction.
Similarly, asumming z2 > x2 will give us a contradiction in the same way. Therefore
x 2 = z2 .
Using the exact same method we can prove y2 = w2
Since x6 + z6 = y6 + w6 we have x6 = y6
I will switch back to a, b, c, d keeping in mind a = c , b = d and a3 = −b3
We have the two equations (
a3 + b = a
b3 + a = b
Since a3 = −b3 we must have a = −b √ √
Therefore we must solve a3 = 2a. √ The√solutions
√ √to this √
equation
√ are √ 0,√ 2, − 2.
Hence ( a, b, c, d) = (0, 0, 0, 0), ( 2, − 2, 2, − 2), (− 2, 2, − 2, 2)

Solution. 46. Let ( x, y) := ( g(0), f ( g(0))): g( f ( g(0)) − f ( g(0))) = f ( g( f ( g(0)))) +


g(0) f ( g( f ( g(0)))) = 0 Let ( x, y) := ( g( f ( g(0))), 0): g( f ( g( f ( g(0))))) = f ( g(0)) +
g( f ( g(0))) (1) g(0) = f ( g(0)) + g( f ( g(0))) Let ( x, y) := ( g(0), 0): (2) g( f ( g(0))) =
f ( g(0)) + g(0) From (1) and then (2) we have: g(0) = f ( g(0)) + g( f ( g(0))) =
f ( g(0)) + f ( g(0)) + g(0) = 2 f ( g(0)) + g(0), so f ( g(0)) = 0. Let ( x, y) := (0, 0):
g( f (0)) = f ( g(0)) = 0 Let y := 0: g( f ( x )) = f ( g(0)) + x = x Let y := f ( x ):
g(0) = f ( g( f ( x ))) + x, but g( f ( x )) = x, so g(0) = f ( x ) + x, so f ( x ) = − x + g(0).
We use f ( x ) = − x + g(0) in g( f ( x )) = x and we get: g(− x + g(0)) = x Let
x := − x + g(0) in g(− x + g(0)) = x: g( x ) = − x + g(0) So f ( x ) = g( x ) = − x + g(0).
Check: LHS = g( f ( x ) − y) = g(− x + g(0) − y) = x + y − g(0) + g(0) = x + y
RHS = f ( g(y)) + x = f (−y + g(0)) + x = y − g(0) + g(0) + x = x + y. So functions
f ( x ) = g( x ) = − x + c for any c are solutions of this equation.
199
Solutions

Solution. 47. Case I: x = y then ( x, y) = (50, 50), (1, 1).


Case II: x < y then ( x, y) = (2, 225 ), (5, 510 ), (10, 105 ), (25, 252 ). These pairs are

x 50
found by using equation y = x50 = x x . Clearly, x must be a divisor of 50.
Case III: x > y then this means x50 > y50 so y x = x50 if x > 50. But from the
50
equation in Case II, if x > 50 then x x is not an integer so there is no solutions in this
condition.
∴ The all pairs of solutions are (1, 1), (2, 225 ), (5, 510 ), (10, 105 ), (25, 252 ), (50, 50).

Solution. 48. .Let the midpoint of BC be M, let AM and BE meet at H, and let BE
meet AC at N. Since AB = AC, we have that AM ⊥ BD and we already know
that BE ⊥ AD, so H is the orthocenter of △ ABD, so DH ⊥ AB. Yet, AC ⊥ AB, so
DH ∥ AC. Thus,
BC BC BC
HM MD MC − DC 2 − 3 6 1
= = = BC
= BC
=
AH DC DC 3 3
2

It follows that since H is on median AM, we have that H is the centroid of △ ABC.
Thus, N is the midpoint of AC. Now, notice that ∠ AEN = 90 = ∠ BAN, so △ AEN ∼
EN
△ BAN, which means that AN = AN 2 2
BN , so BN · EN = AN = CN . This gives that
EN NC
NC = BN , so △ ENC ∼ △CNB, so ∠ NCE = ∠ NBC. Furthermore, from similar
triangles AEN and BAN, we have that ∠EAN = ∠ ABN, so

∠ AEC = 180 − ∠EAC − ∠ECA = 180 − ∠ ABN − ∠CBN = 180 − 45 = 135

from which we conclude that ∠ DEC = 180 − ∠ AEC = 45 .

Solution. 49. We start by noting that equality holds for {n, n + 1, n + 2}. This tells us
that the inequality may be hard to prove, and it’s logical to try our hand at a simpler
analogue. We may construct a 2-variable version, where equality holds for {n, n + 1}.
Namely, this is   
1 1
a− b− ≤ ab − 2. (1)
a b
200
Solutions

Proof. After multiplying by ab and expanding, we arrive at

a2 b2 − a2 − b2 + 1 ≤ a2 b2 − 2ab,

which rearranges to 1 ≤ ( a − b)2 , which is true. Equality holds only for | a − b| = 1. ■


Using this inequality, we may easily establish the problem’s claim:
     
1 1 1 1
a− b− c− ≤ a− (bc − 2) ≤ abc − ( a + b + c),
a b c a

where the last estimate is equivalent to

bc 2
abc − 2a − + ≤ abc − ( a + b + c),
a a
2 bc ( a − b)( a − c)
≤ a−b−c+ = ,
a a a
which is clearly true if ( a − b)( a − c) ≥ 2. This holds if a = max{ a, b, c}, which we
may assume WLOG. ■

Solution. 50. WLOG a ≥ b and c ≥ d and b ≥ d abcd = 4( a + b)(c + d) ≤ 4( a + a)(c +


c) = 16ac ⇒ bd ≤ 16 ⇒ d2 ≤ bd ≤ 16 ⇒ d ≤ 4
2(c + d) = ab ⇒ 2c = ab − 2d 2( a + b) = cd ⇒ 4( a + b) = 2cd ⇒ 4( a + b) =
( ab − 2d)d ⇒ abd2 − 4ad − 4bd − 2d3 = 0 ⇒
( ad − 4)(bd − 4) = 2d3 + 16 and obviously we can put d = 1, 2, 3, 4 in the last equal-
ity,and find solutions... d = 1: ( a − 4)(b − 4) = 18 ⇒ b = 5, a = 22, c = 54 or
b = 6, a = 13, c = 38 or b = 7, a = 10, c = 34
d = 2: (2a − 4)(2b − 4) = 32 ⇒ ( a − 2)(b − 2) = 8 ⇒ b = 3, a = 10, c = 13 or
b = 4, a = 6, c = 10
d = 3: (3a − 4)(3b − 4) = 70 ⇒ b = 3, a = 6, c = 6
d = 4: (4a − 4)(4b − 4) = 144 ⇒ ( a − 1)(b − 1) = 9 ⇒ b = 4, a = 4, c = 4 therefore
all solutions are: {{ a, b}, {c, d}} = {{22, 5}, {54, 1}}, {{13, 6}, {38, 1}}, {{7, 10}, {34, 1}}
, {{10, 3}, {13, 2}}, {{6, 4}, {10, 2}}, {{6, 3}, {6, 3}}, {{4, 4}, {4, 4}}

201
About the Authors

Pranav M. Sawant
Pranav Milind Sawant is a Grade 12 student from Army Public School, Pune. He has been
studying Olympiad mathematics for the past four years and has done exceedingly well,
alongside consistently bagging single-digit ranks, in several prestigious Olympiads such as
the IOQM (Indian Olympiad Qualifier in Mathematics), INMO (Indian National Mathematics
Olympiad), SouthEast Asian Maths Olympiad, Asia International Math Olympiad, Purple
Comet and many others.
With a background in Competitive programming as well, Pranav loves to solve
Combinatorics, Game Theory and Number Theory problems. His research interests include
the likes of Number Theory, Stochastic Analysis, Mathematical Modelling, Game Theory and
Astronomy. He has also authored two research papers and has a provisional patent to his
name. You can find more about him in the links provided.

Personal Website: https://pranavsawant.in


Linkedin: https://www.linkedin.com/in/pranavsawant-2005/
ResearchGate: https://www.researchgate.net/profile/Pranav-Sawant-6
GoogleScholar:
https://scholar.google.com/citations?user=Io6UOIQAAAAJ&hl=en&authuser=2
Academia: https://independentresearcher.academia.edu/PranavSawant
Piyush K. Jha
Piyush Kumar Jha is a mathlete with multiple accolades in Mathematical Olympiads. He is
currently in grade 12 at Rishabh Public School, Delhi. He has been preparing for math
Olympiads for the past 3 years and coining math problems since he was in grade 10. With his
commitment and hard work, he qualified for the Indian National Math Olympiad (INMO) in
2020-21. He also has excellent performances in exams such as Limit, Purple Comet and
Sharygin Olympiad.

To share the knowledge he acquired over these years and give back to the community he
runs "Mathematical Society", an educational platform. His research interests include real
analysis, complex analysis, geometry, algebra, number theory and combinatorics.

To connect with Piyush or get hints and solutions for the Unsolved Challenges, he can be
contacted via his email ramanujanpiyush@gmail.com or his AOPS id #lifeismathematics.

Anshuman Shukla’s Links


Personal Website: https://anshuman.email
Linkedin: https://www.linkedin.com/in/anshuman-shukla-a90a93227/
ResearchGate: https://www.researchgate.net/profile/Anshuman-Shukla-17
Medium: https://medium.com/@anshuman.shukla07
Recommended Resources for Olympiad Mathematics

1) Handouts

● Evan Chen Handouts: https://web.evanchen.cc/olympiad.html


● Yufei Zhao Handouts: https://yufeizhao.com/olympiad.html
● Alexander Remorov Handouts:
https://alexanderrem.weebly.com/math-competitions.html
● Po-Shen Loh Handouts: https://math.cmu.edu/~ploh/olympiad.shtml

2) Books

● Mathematical Circles (Russian Experience) by Fomin, Genkin and Itenberg


● Excursion in Mathematics by Bhaskaracharya Pratishthana, Pune
● Inequalities (Little Mathematical Library) by Pavel Korovkin
● Functional Equation by B.J. Venkatchala
● The Art and Craft of Problem Solving by Paul Zeitz
● Mathematical Olympiad Challenges by Titu Andreescu
● Mathematical Olympiad Treasures by Titu Andreescu
● Putnam and Beyond by Gelca and Andreescu
● Lecture Notes on Mathematical Olympiad Courses by Xu Jiagu
● Euclidean Geometry in Mathematical Olympiads by Evan Chen
● Game Theory and Strategy by Philip D. Straffin
● The USSR Olympiad Problem Book by Shklarsky, Chentzov and Yaglom
● Lemmas in Olympiad Geometry by Adreescu, Korsky and Pohoata
● Inequalities: Theorems, Techniques and Selected Problems by Zdravko Cvetkovski
● Problems in Plane Geometry by Igor Sharygin
● The Mathematical Olympiad Handbook by A. Gardiner
● Problem Solving Strategies by Arthur Engel
● Principles and Techniques in Combinatorics by Chi Chuan
● Elementary Number Theory by David Burton
● Polynomials by E.J. Barbeau
● Elementary Number Theory by Waclaw Sierpinski
● Graph Theory by Frank Harary
● Introductory Combinatorics by Richard Brualdi
● Secrets in Inequalities Vil. I and II by Pham Kim Hung
● Functional Equations and How to Solve Them - (Springer) - Christopher G. Small.

You might also like